NCLEX Practice-Respiratory

Lakukan tugas rumah & ujian kamu dengan baik sekarang menggunakan Quizwiz!

During an admission assessment, a patient with asthma reports experiencing frequent coughing. What other symptoms does the nurse anticipate the patient will report? A Dyspnea B Low-grade fever C Hemoptysis D Weight gain

A Dyspnea The patient with asthma will most likely report periods of dyspnea during a flare-up of his asthma due to the narrowed air passages. Fever and hemoptysis may be indicative of infection. Weight gain is not a typical sign of asthma.

It is appropriate to teach patients to obtain sufficient rest to help decrease the frequency with which they contract upper respiratory infections. How does rest help prevent respiratory infections? A Rest assists in keeping the immune system healthy. B Rest allows the body to produce more red blood cells. C Rest reduces the amount of vitamin C that the body excretes. D Rest enhances the functioning of the cough reflex.

ANS A Proper rest and good nutrition help keep the immune system functioning properly, which will decrease the likelihood of contracting respiratory infections. White blood cells rather than red blood cells fight infection; rest does not influence the amount of vitamin C excreted; and rest has not have an effect on the cough reflex.

A patient tells the nurse, "I went to the dentist a few days ago. I was sure I had a bad toothache; my upper teeth hurt so much. But the dentist told me my teeth were fine and that I should see my physician." What do these symptoms most likely indicate? A Maxillary sinus infection B Otitis media C Acute pharyngitis D Epistaxis

ANS A Sinusitis can cause pressure from mucus and purulent drainage buildup to affect nearby anatomical structures; in this patient's case, the maxilla. Significant pain and discomfort are often the result.

The patient tells the LPN/LVN that she has been hoarse for the past 2½ weeks. Which response by the nurse is most appropriate? A "You should see your primary health care provider." B "Use a high-humidity vaporizer two or three times a day." C "Try to talk as little as possible." D "Gargle with warm, slightly salted water."

ANS A Hoarseness for an extended period of time should be investigated by the primary health care provider. Running a humidifier, talking as little as possible, and gargling with warm salted water are good interventions if the sore throat is found to be due to viral or bacterial infection, but the hoarseness could be due to a more serious cause.

How does exhaling affect the pH of the blood? A Exhaling causes the pH to increase (more alkaline). B Exhaling causes the pH to lower (more acidic). C Exhaling causes the pH to be positive. D Exhaling causes the pH to be negative.

ANS B Exhaling releases carbon dioxide, which makes the blood from the respiratory tract more acidic. This has the effect of raising the pH of blood, making it more alkaline. Retaining carbon dioxide would lower the pH of the blood by making it more acidic.

The nurse wants to obtain information regarding the patient's oxygen (O2) saturation level. What is the least invasive method of obtaining this information? a Perfusion scan b Pulse oximetry c Arterial blood gases (ABGs) d Pulmonary function tests

ANS B Pulse oximetry is a noninvasive method for obtaining the patient's O2 saturation. It is obtained by placing an oxygen sensor on a patient's finger, earlobe, or pinna of the ear; O2 saturation, or percentage of hemoglobin carrying O2, is obtained via a digital readout. Perfusion scans are contrast media studies of pulmonary blood supply. Although a highly accurate method of evaluating arterial oxygen content (PaO2), ABG is invasive since it involves an arterial puncture (usually at the radial artery). Pulmonary tests are noninvasive measurements of lung capacities, but they are not used as a means of obtaining arterial O2 content.

A patient is asked to give the LPN/LVN a sputum specimen. When is the best time for the nurse to obtain the specimen? A In the middle of the day, right after lunch B Immediately after drinking hot fluids C In the morning, before mouth care is done D Right before bedtime, after mouth care is done

ANS C A sputum specimen is most easily obtained first thing in the morning since respiratory secretions tend to pool at night while the patient is supine and sleeping. Mouth care should be given after the specimen has been obtained.

To defend against exposure to foreign particles, the mucous membrane of the respiratory tract contains tiny, hair-like projections. What are these called? A Alveoli B Surfactants C Cilia D Chemoreceptors

ANS C Cilia are the hair-like projections that catch foreign particles before these particles advance farther into the respiratory tract and cause infection.

To defend against exposure to foreign particles, the mucous membrane of the respiratory tract contains tiny, hair-like projections. What are these called? a Alveoli b Surfactants c Cilia d Chemoreceptors

ANS C Cilia are the hair-like projections that catch foreign particles before these particles advance farther into the respiratory tract and cause infection.

Which finding in a female patient should indicate to the LPN/LVN that the patient is likely to have a respiratory problem? A A fine macular-papular rash over the anterior of her chest B Inverted breast nipples C Clubbing of the fingers D Inability to rotate the shoulder joint

ANS C Clubbing of the fingers may be seen in patients with chronic respiratory or heart disease. Clubbing is characterized by the fingers being wider than normal at the distal end, similar in shape to a club. Also, there is marked rounded curvature of the fingernails. The other options do not indicate respiratory problems.

The nurse is performing an admission assessment on a patient who is scheduled for several diagnostic respiratory procedures. Which symptom reported by the patient would make the nurse suspect the patient may have laryngeal cancer? A Difficulty swallowing B Lump in the neck C Persistent hoarseness D Anemia

ANS C Laryngitis lasting longer than 2 weeks is considered ominous and should be followed up with a physician. Anemia is an issue that usually results from the effects of chemotherapy on bone marrow. Difficulty swallowing may indicate a problem within the esophagus. A lump in the neck may be an enlarged lymph node.

Which nursing intervention ensures adequate ventilating exchange after surgery? A. Remove the airway only when client is fully conscious B. Assess for hypoventilation by auscultating the lungs C. Position client laterally with the neck extended D. Maintain humidified oxygen via nasal canula

ANS C Positioning the client laterally with the neck extended does not obstruct the airway so that drainage of secretions and oxygen and carbon dioxide exchange can occur.

Where are the chemoreceptors that regulate breathing located in the body? a Carotid artery and aorta b Jugular vein and left atrium c Cerebellum and pons d Coronary sinus and alveoli

ANS C Regulatory chemoreceptors are found in the brainstem.

Common patient complaints associated with respiratory disorders include: A cough, pain, dyspnea. B cough, headache, palpitations. C dyspnea, fatigue, weakness. D pain, confusion, edema.

ANS C Shortness of breath or difficulty breathing can lead to fatigue and weakness because of the amount of energy necessary to breathe. Although many patients with respiratory disorders complain of coughing and dyspnea, pain is uncommon. Headache and palpitations are unusual for respiratory patients, unless they occur as adverse effects of medications used in treatment (notably aminophylline and theophylline). Confusion may result from severe hypoxia; however, pain is unusual and edema generally results from a cardiovascular, renal, or liver dysfunction.

The nurse is assessing an elderly patient who has been admitted with a diagnosis of aspiration pneumonia. As the nurse auscultates the patient's lungs, diminished breath sounds are heard on the right side. Why is the patient more likely to aspirate foreign objects into the right bronchus rather than the left? a The right bronchus is shorter and more horizontal. b The right bronchus is narrower and shorter. c The right bronchus angles off to the right. d The right bronchus is larger and wider.

ANS C The right bronchus branches more sharply from the trachea, making it easier for aspirated objects, such as food, to become lodged there. All of the descriptions comparing the right bronchus with the left bronchus are anatomically incorrect.

A client with pneumonia is receiving supplemental oxygen, 2 L/min via nasal cannula. The client's history includes chronic obstructive pulmonary disease (COPD) and coronary artery disease. Because of these findings, the nurse closely monitors the oxygen flow and the client's respiratory status. Which complication may arise if the client receives a high oxygen concentration? A. Apnea B. Anginal pain C. Respiratory alkalosis D. Metabolic acidosis

ANS: A Hypoxia is the main breathing stimulus for a client with COPD. Excessive oxygen administration may lead to apnea by removing that stimulus. Anginal pain results from a reduced myocardial oxygen supply. A client with COPD may have anginal pain from generalized vasoconstriction secondary to hypoxia; however, administering oxygen at any concentration dilates blood vessels, easing anginal pain. Respiratory alkalosis results from alveolar hyperventilation, not excessive oxygen administration. In a client with COPD, high oxygen concentrations decrease the ventilatory drive, leading to respiratory acidosis, not alkalosis. High oxygen concentrations don't cause metabolic acidosis.

The nurse is caring for four clients on a stepdown intensive care unit. The client at the highest risk for developing nosocomial pneumonia is the one who: A. has a respiratory infection B. is intubated and on a ventilator C. has pleural chest tubes D. is receiving feedings through a jejunostomy tube

ANS: B When clients are on mechanical ventilation, the artificial airway impairs the gag and cough reflexes that help keep organisms out of the lower respiratory tract. The artificial airway also prevents the upper respiratory system from humidifying and heating air to enhance mucociliary clearance. Manipulations of the artificial airway sometimes allow secretions into the lower airways. Whit standard procedures the other choices wouldn't be at high risk.

In a client with enteritis and frequent diarrhea, the nurse should anticipate an acidbase imbalance of: A. respiratory acidosis B. respiratory alkalosis C. metabolic acidosis D. metabolic alkalosis

ANS: C Diarrhea causes a bicarbonate deficit. With loss of the relative alkalinity of the lower GI tract, the relative acidity of the upper GI tract predominates leading to metabolic acidosis. Diarrhea doesn't lead to respiratory acid-base imbalances, such as respiratory acidosis and respiratory alkalosis. Loss of acid, which occurs with severe vomiting, may lead to metabolic alkalosis.

The nurse is teaching a client with chronic bronchitis about breathing exercises. Which of the following should the nurse include in the teaching? A. Make inhalation longer than exhalation. B. Exhale through an open mouth. C. Use diaphragmatic breathing. D. Use chest breathing.

ANS: C In chronic bronchitis, the diaphragmatic is flat and weak. Diaphragmatic breathing helps to strengthen the diaphragm and maximizes ventilation. Exhalation should longer than inhalation to prevent collapse of the bronchioles. The client with chronic bronchitis should exhale through pursed lips to prolong exhalation, keep the bronchioles from collapsing, and prevent air trapping. Diaphragmatic breathing—not chest breathing—increases lung expansion.

A nurse is caring for a young client with cystic fibrosis (CF) who has frequent episodes of dyspnea. Which intervention from the plan of care should the nurse perform? a) Reinforce family teaching on the performance of chest physiotherapy. b) Suction the client's upper airway every four hours. c) Reinforce teaching about the importance of following a fluid restriction. d) Administer oxygen as needed for dyspnea.

a) Reinforce family teaching on the performance of chest physiotherapy. Routine scheduling of airway clearance using chest physiotherapy is an essential intervention for clients with CF. Suctioning the upper airway will not help dyspnea due to thick secretions in the lungs. Fluid restrictions are not correct treatment for a client with CF. Fluid is required to thin thick secretions. Oxygen will not help to relieve dyspnea caused by thick secretions in the lungs.

A 68-year-old client with end-stage chronic obstructive pulmonary disease (COPD) has discharge orders that include home oxygen therapy. The client exhibits anxiety about becoming dependent on supplemental oxygen. How can a nurse help allay the client's anxiety? a) Remain with the client during an education session taught by the respiratory therapist, and reinforce teaching after the session. b) Arrange for the client to meet the respiratory therapists who provide assistance with home oxygen therapy. c) Notify the physician of the client's concerns about home oxygen use. d) Send the client to the respiratory therapy department for instruction about home oxygen use.

a) Remain with the client during an education session taught by the respiratory therapist, and reinforce teaching after the session. Respiratory therapy is an area of health care that requires specialized training in managing respiratory disorders. The nurse should remain with the client while he receives instruction about home oxygen use from the respiratory therapist. After the session, the nurse should reinforce teaching and have the client perform a return demonstration of equipment use. If the client remains anxious after these interventions, the physician should be notified of the client's concerns.

The nurse is caring for a client experiencing an acute asthma attack. The client stops wheezing and breath sounds are not audible. The nurse understands this change occurred for which reason? a) The airways are so swollen that no air can get through b) The swelling has decreased c) Crackles have replaced wheezes d) The attack is over

a) The airways are so swollen that no air can get through During an acute attack, wheezing may stop and breath sounds may become inaudible because the airways are so swollen that air can't get through. If the attack is over and swelling has decreased, there would be no more wheezing and breath sounds would be clearly audible. Crackles don't replace wheezes during an acute asthma attack.

A client with pulmonary embolism has received a thrombolytic medication. Which teaching point should the nurse reinforce with this client and his family? a) The medication was given to break apart the blood clot blocking the pulmonary artery b) The medication is taken orally and will thin the blood c) The medication will help the client to breathe by dilating bronchial tubes d) The medication will prevent future clots from forming

a) The medication was given to break apart the blood clot blocking the pulmonary artery A thrombolytic medication is given I.V. to break apart or dissolve blood clots. It isn't given orally, doesn't prevent future clots from forming, and has no effect on the bronchial tubes. (

Which strategy would be best to include in the care plan for a child with acute epiglottitis? a) Maintain respiratory isolation for 48 hours. b) Administer I.V. antibiotic therapy. c) Maintain the child in semi-Fowler position. d) Encourage oral fluids for hydration.

b) Administer I.V. antibiotic therapy. The etiologic agent for epiglottitis is usually bacterial; therefore, the treatment consists of I.V. antibiotic therapy. The child shouldn't be allowed anything by mouth during the initial phases of the infection to prevent aspiration. The child should be placed in Fowler position or any position that provides the most comfort and security. Respiratory isolation isn't required.

Which statement best describes what happens to the alveoli in acute respiratory distress syndrome (ARDS)? a) Alveoli increase perfusion b) Alveolar spaces are filled with fluid c) Alveoli improve gaseous exchange d) Alveoli are overexpanded

b) Alveolar spaces are filled with fluid In ARDS, the alveolar membranes are more permeable and the spaces are fluid-filled. The fluid interferes with gas exchange and reduces perfusion.

A chest X-ray shows a client's lungs to be clear. His tuberculin Mantoux skin test is positive, with 10 mm of induration. His previous test was negative. These test results are possible because of which reason? a) He is a "seroconverter," meaning the TB has gotten to his bloodstream b) He is a "tuberculin converter," which means he has been infected with TB since his last skin test c) He had tuberculosis (TB) in the past and no longer has it d) He was successfully treated for TB but skin tests always stay positive

b) He is a "tuberculin converter," which means he has been infected with TB since his last skin test He is a "tuberculin converter," which means he has been infected with TB since his last skin test

Using which part of the hands is appropriate when performing chest compressions on a child between ages 1 and 8? a) Index and middle fingers b) Heel of one hand c) Heels of both hands d) Thumbs of both hands

b) Heel of one hand The heel of one hand is recommended for performing chest compressions on children between ages 1 and 8. Two hands are used for adult cardiopulmonary resuscitation. Chest thrusts administered with the middle and third fingers, and in some cases the thumbs of each hand, are used on infants younger than age 1.

The nurse is discussing pneumonia with the parents of a child diagnosed with the condition. Which definition best describes pneumonia? a) Severe infection of the bronchioles b) Inflammation of the pulmonary parenchyma c) Acute viral infection with maximum effect at the bronchiolar level d) Inflammation of the large airways

b) Inflammation of the pulmonary parenchyma Pneumonia is an inflammation of the pulmonary parenchyma. Bronchitis is inflammation of the large airways. Bronchiolitis is a severe infection of the bronchioles. Bronchiolitis and respiratory syncytial virus are types of acute viral infection with maximum effect at the bronchiolar level.

Which of the following would be appropriate for a client with arterial blood gas (ABG) values of pH 7.5, PaCO2 26 mm Hg, O2 saturation 96%, HCO3- 24 mEq/L, and PaO2 94 mm Hg? a) Administer prescribed supplemental oxygen. b) Instruct the client to breathe into a paper bag. c) Administer a prescribed decongestant. d) Offer the client fluids frequently.

b) Instruct the client to breathe into a paper bag. The ABG results reveal respiratory alkalosis. The best intervention to raise the PaCO2 level would be to have the client breathe into a paper bag. All of the other options — such as administering a decongestant, offering fluids frequently, and administering supplemental oxygen — wouldn't raise the lowered PaCO2 level.

A client has just undergone a bronchoscopy. Which nursing interventions are appropriate after this procedure? Select all that apply. a) Assist the client to resume food and fluids when his or her voice returns. b) Monitor the client's vital signs. c) Keep the client flat for at least 2 hours. d) Withhold food and fluids until the client's gag reflex returns. e) Assess for hemoptysis and frank bleeding. f) Provide sips of water to moisten the client's mouth.

b) Monitor the client's vital signs. d) Withhold food and fluids until the client's gag reflex returns. e) Assess for hemoptysis and frank bleeding. To prevent aspiration, the client should not receive food or fluids until the gag reflex returns. Although a small amount of blood in the sputum is expected if a biopsy was performed, frank bleeding indicates hemorrhage and should be reported to the physician immediately. Vital signs should be monitored after the procedure, because a vasovagal response may cause bradycardia, laryngospasm can affect respirations, and fever may develop within 24 hours of the procedure. To reduce the risk of aspiration, the client should be placed in a semi-Fowler's or side-lying position after the procedure until the gag reflex returns. The client does not lose the voice after a bronchoscopy, so voice should not be used as a gauge for resuming food and fluid intake.

Which intervention should be done before a chest tube is removed? a) Sedate the client; the physician will remove the tube without warning the client b) Provide the results of the most recent chest X-ray for the physician to review before removal c) Make sure that the physician orders arterial blood gas analysis before removal d) Disconnect the drainage system from the chest tube before the tube's removal

b) Provide the results of the most recent chest X-ray for the physician to review before removal A chest X-ray should be done before chest tube removal to ensure that the client's lung has remained expanded after suction was discontinued. Pulse oximetry would be sufficient and is the more commonly used method to track oxygenation. Disconnecting the drainage system before the chest tube is removed could cause tension pneumothorax. Client cooperation is desirable; if the client can hold his breath while the chest tube is removed, there's less chance that air will be drawn back into the pleural space durng removal.

In the client with terminal lung cancer, the focus of nursing care is on which nursing intervention? a) Providing emotional support b) Providing pain control c) Preparing the client's will d) Providing nutritional support

b) Providing pain control The client with terminal lung cancer may have extreme pleuritic pain and should be treated to reduce his discomfort. Preparing the client and his family for the impending death is also important but shouldn't be the primary focus until pain is under control. Nutritional support may be provided, but as the terminal phase advances, the client's nutritional needs greatly decrease. Nursing care doesn't focus on helping the client prepare a will.

Which result is the primary goal of surgical resection for lung cancer? a) Remove as much of the tumor as possible, without removing any alveoli b) Remove the tumor and as little surrounding tissue as possible c) Remove the tumor and all surrounding tissue d) Remove all the tumor and any collapsed alveoli in the same region

b) Remove the tumor and as little surrounding tissue as possible The goal of surgical resection is to remove the cancerous lung tissue that has tumor in it while preserving as much surrounding tissue as possible. It may be necessary to remove alveoli and bronchioles, but care is taken to remove only what's absolutely necessary.

A client has a benign lung tumor. How is the tumor treated? a) Left alone unless symptoms are present b) Removed, involving the least amount of tissue possible c) With chemotherapy only d) With radiation only

b) Removed, involving the least amount of tissue possible The tumor is removed to prevent further compression of lung tissue as the tumor grows, which could lead to respiratory decompensation. If for some reason the tumor can't be removed, then chemotherapy or radiation may be used to try to shrink it.

A client with a large pulmonary embolism will have an arterial blood gas analysis performed to determine the extent of hypoxia. Which acid-base disorder may be present? a) Metabolic alkalosis b) Respiratory alkalosis c) Respiratory acidosis d) Metabolic acidosis

b) Respiratory alkalosis A client with a large pulmonary embolism will have a large region of lung tissue unavailable for perfusion. This causes the client to hyperventilate and blow off large amounts of carbon dioxide, which crosses the unaffected alveolar-capillary membrane more readily than does oxygen, resulting in respiratory alkalosis.

A client with interstitial lung disease is prescribed prednisone to control inflammation. During client teaching, the nurse stresses the importance of taking prednisone exactly as prescribed and cautions against discontinuing the drug abruptly. A client who discontinues prednisone abruptly may experience: a) hyperglycemia and glycosuria. b) acute adrenocortical insufficiency. c) GI bleeding. d) restlessness and seizures.

b) acute adrenocortical insufficiency. Administration of a corticosteroid such as prednisone suppresses the body's natural cortisol secretion, which may take weeks or months to normalize after drug discontinuation. Abruptly discontinuing such therapy may cause the serum cortisol level to drop low enough to trigger acute adrenocortical insufficiency. Hyperglycemia, glycosuria, GI bleeding, restlessness, and seizures are common adverse effects of corticosteroid therapy, not its sudden cessation.

A dark-skinned client with asthma seeks emergency care for acute respiratory distress. Because of this client's dark skin, the nurse should assess for cyanosis by inspecting the: a) lips. b) mucous membranes. c) nail beds. d) earlobes.

b) mucous membranes. Skin color doesn't affect the mucous membranes. The lips, nail beds, and earlobes are less reliable indicators of cyanosis because they're affected by skin color.

During the insertion of a rigid scope for bronchoscopy, a client experiences a vasovagal response. Which physiological response should the nurse monitor this client for? a) Client to experience bronchodilation b) Decrease in the client's gastric secretions c) A drop in the client's heart rate d) Client's pupils to dilate

c) A drop in the client's heart rate During a bronchoscopy, a vasovagal response may be caused by stimulating the pharynx, and it, in turn, may cause stimulation of the vagus nerve. The client may, therefore, experience a sudden drop in heart rate leading to syncope. Stimulation of the vagus nerve doesn't lead to pupillary dilation or bronchodilation. Stimulation of the vagus nerve increases gastric secretions.

An elderly client with pneumonia commonly exhibits which symptom first? a) Pleuritic chest pain and cough b) Fever and chills c) Altered mental status and dehydration d) Hemoptysis and dyspnea

c) Altered mental status and dehydration Fever, chills, hemoptysis, dyspnea, cough, and pleuritic chest pain are the common symptoms of pneumonia, but elderly clients may first exhibit only an altered mental status and dehydration due to a blunted immune response.

Which measure would be ordered first for a client who recently had a central venous access device inserted and who now appears short of breath and anxious? a) Electrocardiogram b) Laboratory tests c) Chest X-ray d) Sedation

c) Chest X-ray Inserting an I.V. catheter in the subclavian vein can result in a pneumothorax, so a chest X-ray should be done. If it's negative, then other tests should be done, but they aren't appropriate as the first intervention

Which treatment would the nurse expect for a client with spontaneous pneumothorax? a) Hyperbaric chamber b) Antibiotics c) Chest tube placement d) Bronchodilators

c) Chest tube placement The only way to reexpand the lung is to place a chest tube on the right side so the air in the pleural space can be removed and the lung reexpanded. Antibiotics and bronchodilators would have no effect on lung reexpansion, nor would the hyperbaric chamber

Nurse Oliver is caring for a client immediately after removal of the endotracheal tube. The nurse reports which of the following signs immediately if experienced by the client? a. Stridor b. Occasional pink-tinged sputum c. A few basilar lung crackles on the right d. Respiratory rate 24 breaths/min

Answer A. The nurse reports stridor to the physician immediately. This is a high-pitched, coarse sound that is heard with the stethoscope over the trachea. Stridor indicates airway edema and places the client at risk for airway obstruction. Options B, C, and D are not signs that require immediate notification of the physician.

While the chest tube is removed the nurse should instruct the patient to: a. Exhale deeply b. Inhale deeply c. Lie at the abdomen d. Hyperextend the neck

Answer: A. As the chest tube is removed the client should be instructed to exhale deeply and do valsalva maneuver. The client is placed in a semi-fowler's position before the chest tube is removed.

A nurse is about to perform a Mantoux test. The test is done by using which route? a. Intradermal b. Intramuscular c. Subcutaneous d. IM with the use of Z-track method

Answer: A. Mantoux tests are done intradermally.

A client is on chest tube. A three-way bottle system is used. The nurse expects that the suction bottle will normally have which of the following characteristics. a. Intermittent bubbling b. Continuous bubbling c. No bubbling d. None of the above

Answer: A. Suction bottle will have continuous suctioning while the water seal bottle will have an intermittent suctioning.

Before a tonsillectomy is performed, which of the following data is very crucial for the nurse to assess? a. Degree of pain b. URTI c. Drainage on the ears d. Respiration pattern

Answer: B. The should assess for Upper respiratory tract infection (URTI). Coughing and sneezing postoperatively due to URTI may cause bleeding.

The physician is going to remove the chest tube from a client. The nurse should least likely prepare which of the following item? a. Sterile gauze b. Suture removal kit c. Empty bottles d. Adhesive tape

Answer: C. In removal of the chest tube the nurse should prepare the following: • Petrolatum gauze • Suture removal kit • Sterile gauze • Adhesive tape

The nurse assesses a patient with emphysema and notes a barrel chest. What is the reason for this patient's chest anomaly? A Collapse of distal alveoli B Hyperinflation of the lungs C Long-term, chronic hypoxia D Use of accessory muscles

B Hyperinflation of the lungs A patient with emphysema develops a barrel chest as a result of the trapping of air in the lungs, causing them to hyperinflate. Collapsed alveoli do not cause a barrel chest.

The nurse notes that the respiratory symptoms of the patient with chronic obstructive pulmonary disease (COPD) are affected by his position. Which would most help to alleviate the patient's dyspnea? A Lying supine with one pillow B Standing or sitting upright C Side-lying with his head elevated D Lying with his head elevated

B Standing or sitting upright A patient with COPD experiences increased difficulty breathing when lying down. Respiratory effort is improved by standing, sitting upright, or positioning the bed in high Fowler's position.

What would be the primary goal of therapy for a client with pulmonary edema and heart failure? A. Enhance comfort B. Increase cardiac output C. Improve respiratory status D. Peripheral edema decreased

B. Increase cardiac output The primary goal of therapy for the client with pulmonary edema or heart failure is increasing cardiac output. Pulmonary edema is an acute medical emergency requiring immediate intervention.

In the elderly, which is an obvious sign of atypical pneumonia? A Diplopia B Nystagmus C Confusion D Elevated temperature

C Confusion Atypical pneumonia may be overlooked in any patient because the signs and symptoms of the infection include a normal or subnormal temperature, normal breath sounds with only occasional crackles and wheezes, and no pain, dry cough, or feeling of extreme fatigue as usually seen in patients with pneumonia. The elderly patient often presents with confusion due to hypoxia. Diagnosis is made by chest x-ray.

The term "pink puffer" refers to the client with which condition? a) Acute respiratory distress syndrome (ARDS) b) Asthma c) Emphysema d) Chronic obstructive bronchitis

c) Emphysema Because of the large amount of energy it takes to breathe, clients with emphysema are usually cachectic. They're pink and they usually breathe through pursed lips, hence the term "puffer." Clients with asthma don't have any particular characteristics. Clients with chronic obstructive bronchitis are bloated and cyanotic in appearance, and clients with ARDS are usually acutely short of breath.

Administration of which childhood vaccination assists in decreasing a child's incidence of developing epiglottitis? a) Diphtheria vaccine b) Measles vaccine c) Haemophilus influenzae type B (Hib) vaccine d) Inactivated polio vaccine (IPV)

c) Haemophilus influenzae type B (Hib) vaccine Epiglottitis is caused by the bacterial agent H. influenzae. The American Academy of Pediatrics recommends that, beginning at age 2 months, children receive the Hib conjugate vaccine. A decline in the incidence of epiglottitis has been seen as a result of this vaccination regimen. The diphtheria vaccine, measles vaccine, and IPV are preventive for those diseases, not epiglottitis.

The nurse is caring for a client with chest trauma. Which nursing diagnosis takes highest priority? a) Anxiety b) Decreased cardiac output c) Impaired gas exchange d) Ineffective tissue perfusion: cardiopulmonary

c) Impaired gas exchange For a client with chest trauma, a diagnosis of Impaired gas exchange takes priority because adequate gas exchange is essential for survival. Although the other options are possible nursing diagnoses for this client, they take lower priority.

A client who underwent surgery 12 hours ago has difficulty breathing. He has petechiae over his chest and reports acute chest pain. What action should a nurse take first? a) Administer a heparin bolus and begin an infusion at 500 units/hour. b) Administer analgesics, as ordered. c) Initiate oxygen therapy. d) Perform nasopharyngeal suctioning.

c) Initiate oxygen therapy. The client's signs and symptoms suggest a pulmonary embolism. Therefore, maintaining respiratory function takes priority. The nurse should first initiate oxygen therapy and then notify the physician immediately. The physician will most likely prescribe an anticoagulant such as heparin or an antithrombolytic to dissolve the thrombus. Analgesics can be administered to decrease pain and anxiety, but oxygen administration takes priority. Suctioning typically isn't necessary with a pulmonary embolism.

Which activity is recommended to prevent foreign body aspiration during meals? a) Give children toys to play with. b) Allow children to watch television. c) Insist that children are seated. d) Allow children to eat in a separate room.

c) Insist that children are seated. Children should remain seated while eating. The risk of aspiration increases if children are running, jumping, or talking with food in their mouth. Toys and television are a dangerous distraction to toddlers and young children and should be avoided. Children need constant supervision and should be monitored while eating snacks and meals.

A client who was hospitalized for pulmonary embolism is being discharged on warfarin therapy. Which teaching by the nurse about warfarin therapy is correct? a) It will reduce blood pressure and prevent venous stasis. b) Coagulation studies to monitor bleeding times will be necessary every 6 months. c) It inhibits the formation of blood clots. d) It's given to continue to reduce the size of the pulmonary embolism.

c) It inhibits the formation of blood clots. Warfarin inhibits clot formation by interfering with clotting factors that are dependent on vitamin K. Coagulation studies will be performed every 2 to 4 weeks while the client is receiving warfarin.

Which condition could lead to acute respiratory distress syndrome (ARDS)? a) Right meniscus injury b) Receiving conscious sedation c) Massive trauma d) Appendicitis

c) Massive trauma In a client with massive trauma, the tissues lining the alveoli and pulmonary capillaries are injured directly or indirectly, in-creasing the permeability of protein and fluid and leading to the development of hypoxemia and ARDS. Appendicitis, unless it causes overwhelming sepsis, won't lead to ARDS. Injuries to the meniscus and conscious sedation don't lead to ARDS.

A nurse is working in a community health center during the flu season. There is a shortage of flu shots. Which group of clients should have priority in getting the available vaccines? a) High school students b) Members of the city basketball team c) Older clients in a nursing home d) Group of seniors living in a senior citizen apartment complex

c) Older clients in a nursing home Elderly clients living in a nursing home should have priority over the other groups. Most elderly nursing home clients have comorbidities that compromise their health and therefore are the group with the highest risk of contracting the flu. Seniors in a senior citizen apartment complex are presumed to be fairly in good health and have a lower risk than those in the nursing home. High school students and those in a basketball team have a lower risk than the rest of the groups listed.

If a client requires a pneumonectomy, what fills the area of the thoracic cavity? a) The space remains filled with air only b) The surgeon fills the space with a gel c) Serous fluid fills the space and consolidates the region d) The tissue from the other lung grows over to the other side

c) Serous fluid fills the space and consolidates the region Serous fluid fills the space and eventually consolidates, preventing extensive mediastinal shift of the heart and remaining lung. There's no gel that can be placed in the pleural space. The tissue from the other lung can't cross the mediastinum, although a temporary mediastinal shift exists until the space is filled. Air can't be left in the space.

A client admitted to the health care facility with acute bronchitis is receiving supplemental oxygen via nasal cannula. When monitoring this client, the nurse suddenly hears a high-pitched whistling sound. What is the likely cause of this sound? a) The water level in the humidifier reservoir is too low. b) The oxygen concentration is above 40%. c) The oxygen tubing is pinched. d) The client has a nasal obstruction.

c) The oxygen tubing is pinched. Pinching of the tubing used to deliver oxygen causes a high-pitched whistling sound. When the water level in the humidifier reservoir is too low, the oxygen tubing appears dry but doesn't make noise. A client with a nasal obstruction becomes more uncomfortable with nasal prongs in place and doesn't experience relief from oxygen therapy; the client's protests, not an abnormal sound, would alert the nurse to this problem. A nasal cannula can't deliver oxygen concentrations above 40%

The nurse knows to monitor a child with a diagnosis of pertussis for the development of which sign or symptom? a) Inspiratory stridor b) Barking cough c) Whooping cough d) Abrupt high fever

c) Whooping cough Pertussis is characterized by consistent short, rapid coughs followed by a sudden inspiration with a high-pitched whooping sound. A barking cough and inspiratory stridor are noted with croup. Pertussis usually is accompanied by a low-grade fever.

A client with severe acute respiratory syndrome privately informs a nurse that he doesn't want to be placed on a ventilator if his condition worsens. The client's wife and children have repeatedly expressed their desire that everything be done for the client. The most appropriate action by the nurse would be to: a) inform the family of the client's wishes. b) assure the family that everything possible will be done. c) support the client's decision. d) assure the client that everything possible will be done.

c) support the client's decision. The nurse is obligated to act as a client advocate. The nurse shouldn't discuss the issue with the client's family unless the client gives permission. Options 2 and 4 oppose the client's wishes and don't demonstrate client advocacy.

Which instruction is appropriate to give a patient with CAL/COPD? A "When lifting something heavy, try holding your breath." B "Try not to do abdominal breathing when you are walking." C "Use an antihistamine if you feel a cold coming on." D "During activity or exertion, exhale through pursed lips."

D "During activity or exertion, exhale through pursed lips." Exhaling through pursed lips is frequently used by COPD patients since it helps promote better exhalation of carbon dioxide and prevents air trapping.

Which symptom would a nurse most likely observe first in a client with an acute pulmonary embolism? a) Nonproductive cough b) Distended jugular veins c) Bradycardia d) Dyspnea

d) Dyspnea Dyspnea is usually the first symptom of pulmonary embolus because the thrombus prevents gas exchange in the pulmonary arterial bed. If the embolus is large enough, the client may then develop right ventricular failure with such symptoms as distended jugular veins, tachycardia, and circulatory collapse. He may also have hemoptysis.

A client has hypoxemia of pulmonary origin. What portion of arterial blood gas results is most useful in distinguishing acute respiratory distress syndrome from acute respiratory failure? a) Partial pressure of arterial carbon dioxide (PaCO2) b) pH c) Bicarbonate (HCO3-) d) Partial pressure of arterial oxygen (PaO2)

d) Partial pressure of arterial oxygen (PaO2) In acute respiratory failure, administering supplemental oxygen elevates the PaO2. In acute respiratory distress syndrome, elevation of the PaO2 requires positive end-expiratory pressure. In both situations, the PaCO2 is elevated and the pH and HCO3- are depressed.

When a client has a pulmonary embolism, he may develop chest pain caused by which condition? a) Costochondritis b) Referred pain from the pelvis c) Myocardial infarction d) Pleuritic pain

d) Pleuritic pain Pleuritic pain is caused by the inflammatory reaction of the lung parenchyma to the pulmonary embolism. The pain isn't associated with myocardial infarction, costochondritis, or referred pain from the pelvis to the chest.

Which organism is the most common causative agent for bacterial pneumonia? a) Parainfluenza virus b) Respiratory syncytial virus (RSV) c) Mycoplasma d) Pneumococci

d) Pneumococci Pneumococcal pneumonia is the most common causative agent accounting for about 90% of bacterial pneumonia. Mycoplasma is a causative agent for primary atypical pneumonia. Parainfluenza virus and RSV account for viral pneumonia.

An elderly client is admitted to an acute care facility with influenza. The nurse monitors the client closely for which most common complication of influenza? a) Pulmonary edema b) Septicemia c) Meningitis d) Pneumonia

d) Pneumonia Pneumonia is the most common complication of influenza. It may be either primary influenza viral pneumonia or pneumonia secondary to a bacterial infection. Other complications of influenza include myositis, exacerbation of chronic obstructive pulmonary disease, and Reye's syndrome. Myocarditis, pericarditis, transverse myelitis, and encephalitis are rare complications of influenza. Although septicemia may arise when any infection becomes overwhelming, it rarely results from influenza. Meningitis and pulmonary edema aren't associated with influenza.

Warning signs and symptoms of lung cancer include persistent cough, bloody sputum, dyspnea, and which other symptom? a) Generalized weakness b) Dizziness c) Hypotension d) Recurrent pleural effusions

d) Recurrent pleural effusions Recurring episodes of pleural effusions can be caused by the tumor and should be investigated. Dizziness, hypotension, and generalized weakness aren't typically considered warning signals but may occur in advanced stages of cancer.

A client with pneumonia has a nursing diagnosis of Ineffective airway clearance related to increased secretions and ineffective cough. Which intervention would facilitate effective coughing? a) Lying in semi-Fowler position b) Using thoracic breathing c) Inhaling and exhaling from pursed lips d) Sipping water, hot tea, or coffee

d) Sipping water, hot tea, or coffee Sips of water, hot tea, or coffee may stimulate coughing. The best position is sitting in a chair with the knees flexed and the feet placed firmly on the floor. The client should inhale through the nose and exhale through pursed lips. Diaphragmatic, not thoracic, breathing helps to facilitate coughing. (less)

A patient taking aminophylline tells the nurse that he is going to begin a smoking cessation program when he is discharged from the hospital. Why should the nurse tell this patient to notify his physician if his smoking pattern changes? A The patient will need his aminophylline dosage adjusted. B The patient will require an increase in antitussive medication. C The patient will no longer require annual influenza immunization. D The patient will not derive as much benefit from inhaler use.

A The patient will need his aminophylline dosage adjusted. The length of the action of aminophylline is decreased by smoking. Therefore, changes in smoking patterns should be discussed with the physician or other health care provider because this may affect the dosage of aminophylline needed. COPD patients are managed with expectorants, not antitussives. Annual influenza vaccine is recommended for all patients with lung disease. The patient should experience increased benefits from inhaler use following smoking cessation.

Which statement is true about crackles? A. They're grating sounds. B. They're high-pitched, musical squeaks. C. They're low-pitched noises that sound like snoring. D. They may be fine, medium, or course.

ANS: D Crackles result from air moving through airways that contain fluid. Heard during inspiration and expiration, crackles are discrete sounds that vary in pitch and intensity. They're classified as fine, medium, or coarse. Pleural friction rubs have a distinctive grating sound. As the name indicates, these breath sounds result when inflamed pleurae rub together. Continuous, highpitched, musical squeaks, called wheezes, result when air moves rapidly through airways narrowed by asthma or infection or when an airway is partially obstructed by a tumor or foreign body. Wheezes, like gurgles, occur on expiration and sometimes on inspiration. Loud, coarse, low-pitched sounds resembling snoring are called gurgles. These sounds develop when thick secretions partially obstruct airflow through the large upper airways.

Miriam, a college student with acute rhinitis sees the campus nurse because of excessive nasal drainage. The nurse asks the patient about the color of the drainage. In a acute rhinitis, nasal drainage normally is A. Yellow B. Green C. Clear D. Gray

Answer C. Normally, nasal drainage in acute rhinitis is clear. Yellow or green drainage indicates spread of the infection to the sinuses. Gray drainage may indicate a secondary infection.

A community health nurse is conducting an educational session with community members regarding tuberculosis. The nurse tells the group that one of the first symptoms associated with tuberculosis is: a. Dyspnea b. Chest pain c. A bloody, productive cough d. A cough with the expectoration of mucoid sputum

Answer D. One of the first pulmonary symptoms is a slight cough with the expectoration of mucoid sputum. Options A, B, and C are late symptoms and signify cavitation and extensive lung involvement.

Nurse Paul is assisting a physician with the removal of a chest tube. The nurse should instruct the client to: a. Exhale slowly. b. Stay very still. c. Inhale and exhale quickly. d. Perform the Valsalva maneuver.

Answer D. When the chest tube is removed, the client is asked to perform the Valsalva maneuver (take a deep breath, exhale, and bear down). The tube is quickly withdrawn, and an airtight dressing is taped in place. An alternative instruction is to ask the client to take a deep breath and hold the breath while the tube is removed. Options A, B, and C are incorrect client instructions.

A client with sinusitis had undergone Caldwell-Luc Surgery. The nurse should instruct the client to do which of the following after the procedure? a. Chew on the unaffected side only. b. The client can wear dentures 5 days after. c. Sneezing should be avoided for a week after the surgery d. All of the above

Answer: A. After a Caldwell-Luc Operation (Radical Antrum Surgery) the following should be instructed to the client: • Do not chew on the affected side • Caution with oral hygiene to prevent trauma of incision • Do not wear dentures for 10 days • Do not blow nose for 2 weeks after the removal of the packing • Avoid sneezing for two weeks after surgery

The accumulation of fluids in the pleural space is called: a. Pleural effusion b. Hemothorax c. Hydrothorax d. Pyothorax

Answer: A. The strategy used in this item is the "umbrella effect." Hemothorax (blood), hydrothorax (water) and pyothirax (pus; also called empyema) are all types of pleural effusion. The three choices are under pleural effusion (umbrella effect), thus the correct answer is A.

Which intervention is least likely done for sinutis? a. Increase fluid intake b. Cold wet packs c. Hot wet packs d. Rest

Answer: B. Management for sinusitis: • Rest • Increase fluids • Hot wet packs • Codeine • Amoxicillin and other anti-infectives • Nasal decongestants • Irrigation of maxillary sinuses with warm NSS

A client is brought to the ER with complaints of stuffy nose, headache, persistent cough, fever and post-nasal drip. Pain is complained by the client above the eyebrows. The diagnosis is sinusitis. Which of the following sinuses is affected? a. Maxillary b. Frontal c. Ethmoid d. Sphenoid

Answer: B. Because the pain is felt above the eyebrows the affected sinus is the frontal sinus. Pain assessment in sinusitis • Maxillary: cheek and upper teeth • Frontal: above eyebrows • Ethmoid: in and around the eyes • Sphenoid: behind eye, occiput, top of the head

For a female patient with chronic obstructive pulmonary disease, which nursing intervention would help maintain a patent airway? a. Restricting fluid intake to 1,000 ml per day b. Enforcing absolute bed rest c. Teaching the patient how to perform controlled coughing d. Administering prescribe sedatives regularly and in large amounts

Answer C. Controlled coughing helps maintain a patent airway by helping to mobilize and remove secretions. A moderate fluid intake (usually 2 L or more daily) and moderate activity help liquefy and mobilize secretions. Bed rest and sedatives may limit the patient's ability to maintain a patent airway, causing a high risk for infection from pooled secretions.

The morning weight for a patient indicates a weight gain of 5 lb in 6 days. His intake has been within his usual range. The weight gain may indicate the development of which complication of COPD? A Polycythemia B Cor pulmonale C Left ventricular failure D Compensated acidosis

B Cor pulmonale Cor pulmonale is the development of right ventricular failure as a result of chronic lung disease, leading to fluid retention manifested by weight gain. A patient with left ventricular failure would present with pulmonary edema before obvious weight gain. There is no indication of polycythemia or compensated acidosis.

Which treatment goal is best for the client with status asthmaticus? a) To determine the cause of the attack b) To improve exercise tolerance c) To avoid intubation d) To reduce secretions

c) To avoid intubation

A nurse is monitoring the progress of a client with acute respiratory distress syndrome (ARDS). Which data best indicate that the client's condition is improving? a) Arterial blood gas (ABG) values are normal b) The client's blood pressure has stabilized c) The sputum and sensitivity culture shows no growth in bacteria d) The bronchoscopy results are negative

a) Arterial blood gas (ABG) values are normal Normal ABG values would indicate that the client's oxygenation has improved. ARDS is characterized by hypoxia, so the bronchoscopy and sputum culture results have no bearing on the improvement of ARDS. Increased blood pressure isn't relative to the client's respiratory condition.

When auscultating the chest of a client with pneumonia, the nurse would expect to hear which sounds over areas of consolida-tion? a) Bronchial b) Bronchovesicular c) Vesicular d) Tubular

a) Bronchial Chest auscultation reveals bronchial breath sounds over areas of consolidation. Bronchovesicular breath sounds are normal over midlobe lung regions, tubular sounds are commonly heard over large airways, and vesicular breath sounds are commonly heard in the bases of the lung fields

Which laboratory test value is elevated in clients who smoke and therefore can't be used as a general indicator of cancer? a) Carcinoembryonic antigen level b) Alkaline phosphatase level c) Acid phosphatase level d) Serum calcitonin level

a) Carcinoembryonic antigen level Because the level of carcinoembryonic antigen is elevated in clients who smoke, it can't be used as a general indicator of cancer. However, the carcinoembryonic antigen level is helpful in monitoring cancer treatment because it usually falls to normal within 1 month if treatment is successful. An elevated acid phosphatase level may indicate prostate cancer. An elevated alkaline phosphatase level may reflect bone metastasis. An elevated serum calcitonin level usually signals thyroid cancer.

A college student with acute rhinitis sees the campus nurse because of excessive nasal drainage. While assessing the student, the nurse expects to find which color of nasal drainage? a) Clear b) Yellow c) Gray d) Green

a) Clear Normally, nasal drainage in acute rhinitis is clear. Yellow or green drainage indicates spread of the infection to the sinuses. Gray drainage may indicate a secondary infection.

Which characteristic distinguishes status asthmaticus from asthma? a) Constant attacks unrelieved by bronchodilators b) Increased response to bronchodilators c) Several attacks per month d) Less than 6 attacks per year

a) Constant attacks unrelieved by bronchodilators Status asthmaticus can best be described as constant attacks unrelieved by bronchodilators. Moderate asthma is characterized by several attacks per month. Mild asthma is less than 6 attacks per year. Little or no response to bronchodilators occurs in severe asthma

A client has undergone a left hemicolectomy for bowel cancer. Which activities prevent the occurrence of postoperative pneumonia in this client? a) Coughing, breathing deeply, frequent repositioning, and using an incentive spirometer b) Coughing, breathing deeply, maintaining bed rest, and using an incentive spirometer c) Administering pain medications, frequent repositioning, and limiting fluid intake d) Administering oxygen, coughing, breathing deeply, and maintaining bed rest

a) Coughing, breathing deeply, frequent repositioning, and using an incentive spirometer Activities that help to prevent the occurrence of postoperative pneumonia are coughing, breathing deeply, frequent repositioning, medicating the client for pain, and using an incentive spirometer. Limiting fluids and lying still will increase the risk of pneumonia

Emergency treatment of a client in status asthmaticus includes which drug class? a) Inhaled beta-adrenergic agents b) I.V. beta-adrenergic agents c) Inhaled corticosteroids d) Oral corticosteroids

a) Inhaled beta-adrenergic agents Inhaled beta-adrenergic agents help promote bronchodilation, which improves oxygenation. I.V. beta-adrenergic agents can be used but have to be monitored because of their greater systemic effects. They're typically used when the inhaled beta-adrenergic agents don't work. Corticosteroids are slow acting, so their use won't reduce hypoxia in the acute phase.

A client with a newly inserted tracheostomy tube begins to cough and dislodges the tube. In addition to notifying the health care provider, what is the nurse's appropriate action? a) Mask ventilate the client with 100% oxygen. b) Attempt to reinsert the tracheostomy tube. c) Assess the client's oxygen saturation. d) Cover the stoma with sterile gauze.

a) Mask ventilate the client with 100% oxygen. The tracheostomy tube should not be reinserted because a new tracheostomy stoma can close, causing the airway to occlude. The appropriate action for the nurse is to use mask ventilation on the client with 100% oxygen and prepare for intubation. Covering the stoma will prevent adequate oxygenation. Assessing oxygen saturation is not the priority.

The goal of oxygen therapy for a client with a pulmonary embolism is to obtain which value? a) PaO2 above 60 mm Hg b) PaCO2 above 40 mm Hg c) PaO2 below 60 mm Hg d) PaCO2 below 40 mm Hg

a) PaO2 above 60 mm Hg The goal of oxygen therapy for a client with a pulmonary embolism is to have a PaO2 greater than 60 mm Hg on an FIO2 of 40% or less. The normal range of the PaCO2 is 35 to 45 mm Hg. In the absence of other pathologic states, it should reach normal levels before the PaO2 does on room air because carbon dioxide crosses the alveolar-capillary membrane with greater ease

A client admitted to the facility for treatment for tuberculosis receives instructions about the disease. Which statement made by the client indicates the need for further instruction? a) "This disease may come back later if I am under stress." b) "I will stay in isolation for at least 6 weeks." c) "I will have to take the medication for up to a year." d) "I will always have a positive test for tuberculosis."

b) "I will stay in isolation for at least 6 weeks." The client needs to be in isolation for 2 weeks, not 6, while receiving antitubercular drug therapy. After 2 weeks of antitubercular therapy, the client is no longer considered contagious. The client needs to receive the drugs for 9 months to a year. He will be positive when tested, and if he's sick or under some stress he could have a relapse of the disease

The nurse is assisting the physician with removing a client's chest tube. Which intervention is essential to prevent complications following chest tube removal? a) Change the gauze dressing every 4 hours. b) Listen to lung sounds at least every 4 hours. c) Monitor blood gasses every 2 hours. d) Monitor subcutaneous emphysema every 8 hours.

b) Listen to lung sounds at least every 4 hours. The nurse should assess respiratory status at least every 4 hours by auscultating lung sounds. Frequent assessment is necessary to monitor respiratory status and the effect of chest tube after removal. Subcutaneous emphysema will cease when the chest tube is removed. It is unnecessary to monitor blood gas results every 2 hours.

A client with respiratory acidosis is admitted to the intensive care unit for close observation. The nurse should stay alert for which complication associated with respiratory acidosis? a) Hyperglycemia b) Shock c) Seizures d) Stroke

b) Shock Complications of respiratory acidosis include shock and cardiac arrest. Stroke and hyperglycemia aren't associated with respiratory acidosis. Seizures may complicate respiratory alkalosis, not respiratory acidosis.

The nurse prepares to perform postural drainage. Which method would the nurse use to determine the best position for facilitating drainage of the lungs? a) Chest X-ray b) Arterial blood gas (ABG) levels c) Auscultation d) Inspection

c) Auscultation Breath sounds should be auscultated before doing postural drainage to determine the areas that need draining. After the areas are identified, the nurse can position the client appropriately. Inspection, chest X-rays, and ABG levels are all parameters that give good information about respiratory function, but they aren't necessary to determine lung areas requiring postural drainage.

The term "blue bloater" refers to which condition? a) Emphysema b) Acute respiratory distress syndrome (ARDS) c) Chronic obstructive bronchitis d) Asthma

c) Chronic obstructive bronchitis Clients with chronic obstructive bronchitis appear bloated; they have large barrel chests and peripheral edema, cyanotic nail beds and, at times, circumoral cyanosis. Clients with asthma don't exhibit characteristics of chronic disease. Clients with emphysema appear pink and cachectic, and clients with ARDS are acutely short of breath and frequently need intubation for mechanical ventilation and large amounts of oxygen.

The physician determines that a client has been exposed to someone with tuberculosis. The nurse expects the physician to order which of the following? a) Nothing, until signs of active disease arise b) Isolation until 24 hours after antitubercular therapy begins c) Daily doses of isoniazid, 300 mg for 6 months to 1 year d) Daily oral doses of isoniazid and rifampin for 6 months to 2 years

c) Daily doses of isoniazid, 300 mg for 6 months to 1 year All clients exposed to persons with tuberculosis should receive prophylactic isoniazid in daily doses of 300 mg for 6 months to 1 year to avoid the deleterious effects of the latent mycobacterium. Daily oral doses of isoniazid and rifampin for 6 months to 2 years are appropriate for a client with active tuberculosis. Isolation for 2 to 4 weeks is warranted only for a client with active tuberculosis. (less)

The nurse is collecting data on a client who comes to the clinic for care. Which findings, collected by the nurse during an assessment, in this client suggest bacterial pneumonia? a) Nonproductive cough and normal temperature b) Hemoptysis and dysuria c) Dyspnea and wheezing d) Sore throat and abdominal pain

c) Dyspnea and wheezing In a client with bacterial pneumonia, retained secretions cause dyspnea, and respiratory tract inflammation causes wheezing. Bacterial pneumonia also produces a productive cough and fever, rather than a nonproductive cough and normal temperature. Sore throat occurs in pharyngitis, not bacterial pneumonia. Abdominal pain is characteristic of a gastrointestinal disorder, unlike chest pain, which can reflect a respiratory infection such as pneumonia. Hemoptysis and dysuria aren't associated with pneumonia.

Which precaution is recommended when caring for children with respiratory infections such as croup? a) Place the child in isolation. b) Keep siblings in the same room. c) Enforce hand washing. d) Teach children to use tissues.

c) Enforce hand washing. Hand washing helps prevent the spread of infections. Ill children should be placed in separate bedrooms if possible but don't need to be isolated. Teaching children to use tissues properly is important, but the key is tissue disposal and hand washing after use.

When assessing a child for increased laryngotracheal edema and early signs of impending airway obstruction, the nurse should observe for which warning sign? a) Decreased heart and respiratory rates and a high peak flow rate b) Increased temperature c) Increased heart and respiratory rates, retractions, and restlessness d) Decreased blood pressure

c) Increased heart and respiratory rates, retractions, and restlessness Increased heart and respiratory rates, retractions, and restlessness are classic indicators of hypoxia. The heart and respiratory rates increase to enable increased oxygenation. Accessory breathing muscles are used, causing retractions in substernal, suprasternal, and intercostal areas. Reduced oxygen to the brain causes restlessness initially and altered level of consciousness later. A decrease in heart and respiratory rates would be a late ominous sign of decompensation. Peak flow rate is a test used in asthma. A decrease, not increase, in peak flow is diagnostic of disease. A drop in blood pressure would be a late sign of hypoxia. An increase in temperature is more indicative of infection or inflammation than respiratory distress.

A client in the emergency department is diagnosed with a communicable disease. When complications of the disease are discovered, the client is admitted to the hospital and placed in respiratory isolation. Which infection warrants respiratory isolation? a) Cholera b) Chickenpox c) Measles d) Impetigo

c) Measles Measles warrants respiratory isolation, which aims to prevent disease transmission primarily over short distances through the air (droplet transmission). Other infections necessitating respiratory isolation include epiglottitis or pneumonia caused by Haemophilus influenzae, erythema infectiosum, meningitis caused by H. influenzae or meningococci, meningococcal pneumonia, meningococcemia, mumps, and pertussis. Chickenpox calls for strict isolation; impetigo, contact isolation; and cholera, enteric isolation.

The nurse assesses a client's respiratory status. Which observation indicates that the client is experiencing difficulty breathing? a) Controlled breathing b) Diaphragmatic breathing c) Use of accessory muscles d) Pursed-lip breathing

c) Use of accessory muscles The use of accessory muscles for respiration indicates the client is having difficulty breathing. Diaphragmatic and pursed-lip breathing are two controlled breathing techniques that help the client conserve energy.

When caring for a client with acute respiratory failure, the nurse should expect to focus on resolving which of the following problems? a) Hyperventilation, hypertension, and hypocapnia b) Hypotension, hyperoxemia, and hypercapnia c) Hyperoxemia, hypocapnia, and hyperventilation d) Hypercapnia, hypoventilation, and hypoxemia

d) Hypercapnia, hypoventilation, and hypoxemia The cardinal physiologic abnormalities of acute respiratory failure are hypoventilation, hypoxemia, and hypercapnia. The nurse should focus on resolving these problems.

Which intervention is most appropriate when suctioning thick secretions through a tracheostomy tube? a) Hypoventilate the child before suctioning. b) Insert the catheter 1 to 2 cm below the tracheostomy tube. c) Repeat the suctioning process for two intervals. d) Inject a small amount of normal saline solution into the tube before suctioning.

d) Inject a small amount of normal saline solution into the tube before suctioning. Injecting a small amount of normal saline solution helps to loosen secretions for easier aspiration, but should not be done routinely. Preservative-free normal saline solution should be used. The child should be hyperventilated before and after suctioning to prevent hypoxia. The suctioning process should be repeated until the trachea is clear. The catheter should be inserted 0.5 cm beyond the tracheostomy tube. If the catheter is inserted too far, it will irritate the carina and may cause blood-tinged secretions.

Which intervention is most important for a child with atelectasis? a) Increase I.V. fluids. b) Administer oxygen. c) Obtain arterial blood gas (ABG) levels. d) Perform chest physiotherapy.

d) Perform chest physiotherapy. Chest physiotherapy and incentive spirometry help to enhance the clearance of mucus and open the alveoli. Although not the most important intervention, giving I.V. and oral fluids is recommended to help liquefy and thin secretions. Administration of oxygen won't provide enough pressure to open the alveoli. Obtaining ABG levels isn't necessary unless the client has signs of hypoxemia.

Which complication may be seen in a child receiving mechanical ventilation? a) High cardiac output b) Polycythemia c) Hypovolemia d) Pneumothorax

d) Pneumothorax Mechanical ventilation can cause barotrauma, as occurs with pneumothorax. A child receiving mechanical ventilation must be carefully monitored. Mechanical ventilation decreases, not increases, cardiac output. Polycythemia is the result of chronic hypoxia, not mechanical ventilation. Mechanical ventilation can cause fluid overload, not dehydration

A definitive diagnosis of lung cancer is obtained by which evaluation? a) Chest X-ray b) Bronchoscopy c) Computerized tomography of the chest d) Surgical biopsy

d) Surgical biopsy Only surgical biopsy with cytologic examination of the cells can give a definitive diagnosis of the type of cancer. Bronchoscopy gives positive results in only 30% of the cases. Chest X-ray and computerized tomography can identify location but don't diagnose the type of cancer.

A client is receiving oxygen by way of a nasal cannula at a rate of 2 L/minute. How should the oxygen flow meter be set? a) The top of the ball should sit below the line marked "2." b) The bottom of the ball should sit on top of the line marked "2." c) Any part of the ball should touch the line marked "2." d) The line marked "2" should cut the ball in half

d) The line marked "2" should cut the ball in half The oxygen flow rate is set by centering the indicator on the line marked "2."

The nurse observes constant bubbling in the water-seal chamber of a closed chest drainage system. What should the nurse conclude? a) The client has a pneumothorax. b) The system is functioning normally. c) The chest tube is obstructed. d) The system has an air leak.

d) The system has an air leak. Constant bubbling in the chamber indicates an air leak and requires immediate intervention. The client with a pneumothorax will have intermittent bubbling in the water-seal chamber. Clients without a pneumothorax should have no evidence of bubbling in the chamber. If the tube is obstructed, the nurse should notice that the fluid has stopped fluctuating in the water-seal chamber.

The nurse has just completed teaching about postoperative activity to a client who is going to have a cataract surgery. The nurse knows the teaching has been effective if the client: A. coughs and deep breathes postoperatively B. ties his own shoes C. asks his wife to pick up his shirt from the floor after he drops it. D. States that he doesn't need to wear an eyepatch or guard to bed

ANS: C Bending to pick up something from the floor would increase intraocular pressure, as would bending to tie his shoes. The client needs to wear eye protection to bed to prevent accidental injury during sleep.

The nurse is caring for a patient who has sustained a major chest injury during a motor vehicle accident. What are the priority interventions for this patient? (Select all that apply.) A Maintenance of fluid and electrolyte balance B Maintenance of an airway C Assurance of adequate ventilation D Treatment of circulatory problems E Achieving pain control

B Maintenance of an airway C Assurance of adequate ventilation Thoracic trauma is a major cause of accidental death with the major complications of chest trauma involving either the lungs and air passages or the heart and major blood vessels, or both. Fluid and electrolyte balance and pain control are not high-priority interventions for this patient

A patient with COPD asks the nurse to turn his oxygen up from 3 L/min via nasal cannula to 5 L/min. The nurse explains to the patient that she cannot turn his oxygen up this high. What is the reason the oxygen cannot be increased to 5 L/min? A Higher concentrations may result in a severe headache. B Hypercapnic drive is necessary for breathing. C Higher levels will be required later for arterial blood gases (ABGs). D Hypoxic drive is needed for breathing.

D Hypoxic drive is needed for breathing. Respiratory effort is stimulated by hypoxemia in patients with COPD. Higher oxygen levels would eliminate the stimulus to breathe. Low oxygen levels, not high carbon dioxide levels, are the stimulus for breathing for a patient with COPD.

A client is diagnosed with active tuberculosis and started on triple antibiotic therapy. What signs and symptoms would the client show if therapy is inadequate? a) Nonproductive cough b) Positive acid-fast bacilli in a sputum sample after 2 months of treatment c) Improved chest X-ray d) Decreased shortness of breath

b) Positive acid-fast bacilli in a sputum sample after 2 months of treatment Continuing to have acid-fast bacilli in the sputum after 2 months indicates continued infection. The other choices all indicate improvement.

A nurse is caring for a male client with emphysema who is receiving oxygen. The nurse assesses the oxygen flow rate to ensure that it does not exceed: a. 1 L/min b. 2 L/min c. 6 L/min d. 10 L/min

Answer B. Oxygen is used cautiously and should not exceed 2 L/min. Because of the long-standing hypercapnia that occurs in emphysema, the respiratory drive is triggered by low oxygen levels rather than increased carbon dioxide levels, as is the case in a normal respiratory system.

On arrival at the intensive care unit, a critically ill female client suffers respiratory arrest and is placed on mechanical ventilation. The physician orders pulse oximetry to monitor the client's arterial oxygen saturation (SaO2) noninvasively. Which vital sign abnormality may alter pulse oximetry values? a. Fever b. Tachypnea c. Tachycardia d. Hypotension

Answer D. Hypotension, hypothermia, and vasoconstriction may alter pulse oximetry values by reducing arterial blood flow. Likewise, movement of the finger to which the oximeter is applied may interfere with interpretation of SaO2. All of these conditions limit the usefulness of pulse oximetry. Fever, tachypnea, and tachycardia don't affect pulse oximetry values directly.

Bronchodilators include the following apart from: a. Theophyline b. Terbutaline c. Metaproterenol d. Dipenhydramine

Answer: D. Benadryl (Dipenhydramine) is an antihistamine not bronchodilator.

The nurse is caring for a client with asthma. A review of the client's data collection indicates to the nurse that further teaching is required if what factor is collected? a) Client washes bedding in warm water b) Client warms up well before exercising c) Client incorporates vitamins A, C, B, and zinc in the diet d) Client's spouse no longer smokes in the home

a) Client washes bedding in warm water The client's bedding should be washed in hot water to destroy bed mites. Vitamins A, C, B, and zinc are important for immune function and should be incorporated in the diet. Research has shown that secondhand cigarette smoke contributes to asthma so it is good for the spouse to smoke outside the home. Exercise can trigger an acute asthma attack, so the client should warm up before exercising

Which strategy is recommended when caring for an infant with bronchopulmonary dysplasia? a) Place the infant in an open crib. b) Place the infant on a set schedule. c) Decrease oxygen during feedings. d) Provide frequent playful stimuli.

b) Place the infant on a set schedule. Timing care activities with rest periods to avoid fatigue and to decrease respiratory effort is essential. Early stimulation activities are recommended but the infant will have limited tolerance for them because of the illness. Oxygen is usually increased during feedings to help decrease respiratory and energy requirements. Thermoregulation is important because both hypothermia and hyperthermia will in crease oxygen consumption and may increase oxygen requirements. These infants are usually maintained on warmer beds or in Isolettes.

When an unconscious client has been diagnosed with probable drug overdose complicated by alcohol ingestion, the nurse knows that the priority intervention is to: a) draw blood for a drug screen b) continue close monitoring of vital signs c) administer I.V. naloxone d) administer I.V. fluids

c) administer I.V. naloxone If the client took opioids, giving naloxone could reverse the effects and awaken the client. I.V. fluids will most likely be administered, and he'll be closely monitored over a period of several hours to several days. A drug screen should be drawn in the emergency department, but results may not come back for several hours.

The student nurse is caring for a patient with a restrictive respiratory disease. Which description demonstrates student knowledge of the disease? A "The disease is characterized by decreased lung expansion." B "The disease is characterized by increased lung volumes." C "The disease is characterized by narrowed tracheobronchial tree openings." D "The disease is characterized by an obstruction in the lungs."

A "The disease is characterized by decreased lung expansion." Restrictive pulmonary disorders may be caused by decreased elasticity or compliance of the lungs, decreased ability of the chest wall to expand, or disorders of the central nervous system.

The nurse knows to observe for and report which abnormal breathing pattern that is most likely to occur in patients with increased intracranial pressure: 1 Cheyne- Strokes respirations 2 Kussmaul's respirations 3 Biot's respirations 4 Apneustic respirations

ANS 3 Wheezing is commonly heard in asthma due to the narrowing of airways. Fine crackles can be heard in patients who have atelectasis, fibrosis, pneumonia, or early congestive heart failure. Stridor can be heard when there is partial obstruction of the upper air passages. Pleural friction rub occurs when irritated visceral and parietal pleura rub against each other.

Narcotic analgesics are often ordered for patients with respiratory disorders, even though a common side effect of these drugs is respiratory depression. Which rationale would explain narcotic use in these situations? A Narcotics decrease oxygen demand. B Narcotics do not produce dependency. C Narcotics depress the cough reflex. D Narcotics decrease respiratory secretions.

ANS A Morphine, in particular, decreases anxiety, thereby reducing oxygen demands of the body.

Risk factors for development of a respiratory disorder include: (Select all that apply.) A family history of respiratory disorders. B heavy ethyl alcohol use. C group living conditions. D tobacco use. E contact sports.

ANS A, B, C, D All of these are known risk factors for development of respiratory disorders. There are no data to support a correlation between contact sports and respiratory disorders

The nurse is caring for a patient who has had a tonsillectomy. Which observation may indicate bleeding at the operative site? A The patient complains of throat pain. B The patient refuses to lie on his side. C The patient swallows frequently. D The patient seems frightened when moving his neck.

ANS C As blood runs down the throat, the patient will frequently swallow. Throat pain is expected and does not necessarily indicate bleeding. Refusing to lie on his side or being frightened when moving his neck does not indicate bleeding.

The nurse assesses a patient and notices nasal flaring. What does the nurse suspect as the reason for this finding? A Severe respiratory distress B Pneumonia C Hypoxia D Hypoxemia

ANS C Nasal flaring is a sign of inadequate oxygen in body tissues. Nasal flaring is an early sign of respiratory compromise. Nasal flaring is not a sign of a specific disease process. Signs of low blood oxygen levels manifest more acutely than those of hypoxia and include cyanosis.

The nurse is caring for a patient who has just had a thoracentesis performed. How should the nurse position the patient after this procedure? a Flat, prone b Flat, supine c Side-lying, on the affected side d Side-lying, on the unaffected side

ANS C Unless specifically ordered otherwise, this is the appropriate position in the immediate post-thoracentesis period. It allows the mattress to be used to apply pressure to the aspiration site, minimizing leakage and bleeding. Flat, while prone or supine, does not allow for full chest expansion. Unless otherwise specified by the physician, lying on the side on which the procedure was performed is preferable.

A nurse teaches a male client about the use of a respiratory inhaler. Which action by the client indicates a need for further teaching? a. Inhales the mist and quickly exhales b. Removes the cap and shakes the inhaler well before use c. Presses the canister down with the finger as he breathes in d. Waits 1 to 2 minutes between puffs if more than one puff has been prescribed

Answer A. The client should be instructed to hold his or her breath for at least 10 to 15 seconds before exhaling the mist. Options B, C, and D are accurate instructions regarding the use of the inhaler.

Dr. Jones prescribes albuterol sulfate (Proventil) for a patient with newly diagnose asthma. When teaching the patient about this drug, the nurse should explain that it may cause: a. Nasal congestion b. Nervousness c. Lethargy d. Hyperkalemia

Answer B. Albuterol may cause nervousness. The inhaled form of the drug may cause dryness and irritation of the nose and throat, not nasal congestion; insomnia, not lethargy; and hypokalemia (with high doses), not hyperkalemia. Otther adverse effects of albuterol include tremor, dizziness, headache, tachycardia, palpitations, hypertension, heartburn, nausea, vomiting and muscle cramps.

A male client with Guillain-Barré syndrome develops respiratory acidosis as a result of reduced alveolar ventilation. Which combination of arterial blood gas (ABG) values confirms respiratory acidosis? a. pH, 5.0; PaCO2 30 mm Hg b. pH, 7.40; PaCO2 35 mm Hg c. pH, 7.35; PaCO2 40 mm Hg d. pH, 7.25; PaCO2 50 mm Hg

Answer D. In respiratory acidosis, ABG analysis reveals an arterial pH below 7.35 and partial pressure of arterial carbon dioxide (PaCO2) above 45 mm Hg. Therefore, the combination of a pH value of 7.25 and a PaCO2 value of 50 mm Hg confirms respiratory acidosis. A pH value of 5.0 with a PaCO2 value of 30 mm Hg indicates respiratory alkalosis. Options B and C represent normal ABG values, reflecting normal gas exchange in the lungs.

A nurse is assisting a physician with the removal of a chest tube. The nurse should instruct the client to: a. Exhale slowly b. Stay very still c. Inhale and exhale quickly d. Perform the Valsalva maneuver

Answer D. When the chest tube is removed, the client is asked to perform the Valsalva maneuver (take a deep breath, exhale, and bear down). The tube is quickly withdrawn, and an airtight dressing is taped in place. An alternative instruction is to ask the client to take a deep breath and hold the breath while the tube is removed. Options A, B, and C are incorrect client instructions.

Mr. Lorenzo is schedule for a bronchography. Before the procedure the nurse least likely performs which of the following? a. Assist the client in a side-lying position b. Checking for allergies c. Instructing the client to be on NPO for 6-8 hours d. Administer atropine sulfate

Answer: A. Nursing interventions before bronchogram: • Secure written consent • Check for allergies (seafoods and iodine or anesthesia) • NPO 6-8 hours • Pre-op meds: atropine sulfate and valium, topical anesthesia sprayed followed by local anesthetic into the larynx • Have oxygen and antispasmodic agents ready Nursing Interventions after bronchogram • Side-lying position • NPO until cough and gag reflex returns • Cough and deep breathe clients • Low grade fever common

A client with COPD is instructed to follow what diet? a. High carbohydrate, low calorie and high protein diet b. High protein, high calorie and low carbohydrate diet c. High carbohydrate, low protein and high calorie diet d. High protein, high carbohydrate and high caloric diet

Answer: B. Diet for COPD: High calorie, high protein and low carbohydrate diet • High caloric diet provides source of energy. • High protein diet helps maintain integrity of alveolar walls. • Low carbohydrate diet limits carbon dioxide production (natural end product). The client with COPD has difficulty exhaling carbon dioxide

The nurse is assessing the puncture site of a client who has received a purified protein derivative test. Which finding indicates a need for further evaluation? a) 3-mm induration b) 15-mm induration c) Reddened area d) Blister

b) 15-mm induration A 10-mm induration strongly suggests a positive response in this tuberculosis screening test; a 15-mm induration clearly requires further evaluation. The other options aren't positive reactions to the test and require no further evaluation.

What's the best time to administer a nebulizer treatment to a child with croup? a) During playtime b) During naptime c) After the child eats d) After the parents leave

b) During naptime The nurse should administer nebulizer treatments at prescribed intervals. Administering treatment during naptime allows for as little disruption as possible. Administering treatment during playtime disrupts the child's daily pattern. A child should be given a treatment before eating so the airway will be open and the work of eating will be decreased. Parents are usually helpful when administering treatments. The child can sit on the parent's lap to help decrease anxiety or fear.

A 2-year-old child with status asthmaticus is admitted to the pediatric unit and begins to receive continuous treatment with albuterol, given by nebulizer. The nurse should observe for which adverse reaction? a) Tachypnea b) Bradycardia c) Lethargy d) Tachycardia

d) Tachycardia Albuterol is a rapid-acting bronchodilator. Common adverse effects include tachycardia, nervousness, tremors, insomnia, irritability, and headache.

In what structure does the actual gas exchange during respiration occur? a Lungs b Trachea c Bronchi d Alveoli

ANS D Oxygen and carbon dioxide are exchanged through the walls of the alveoli. The trachea and bronchi are air passageways, but no gas exchange occurs in these structures.

A patient with tuberculosis has just been prescribed isoniazid (INH). What side effects should the nurse teach the patient to be aware of? A Peripheral neuritis B Hyperuricemia C Blindness D Deafness

A Peripheral neuritis The patient should be alert to the side effects of peripheral neuritis, hypersensitivity, and jaundice when taking INH. These side effects should be reported to the primary care provider.

The nurse is preparing to administer aminophylline to a patient with asthma. Laboratory results indicate the patient's serum aminophylline level is 17 mcg/mL. What is the most appropriate nursing action? A Administer the medication. B Hold the medication until the next serum aminophylline level is drawn. C Contact the physician immediately with the lab result. D Administer half the dose of the ordered aminophylline.

A Administer the medication. The therapeutic range for aminophylline is 10 to 20 mcg/mL. The patient's level is clearly within the therapeutic range so the medication should be administered.

What is the normal pH range for arterial blood? a. 7 to 7.49 b. 7.35 to 7.45 c. 7.50 to 7.60 d. 7.55 to 7.65

Answer B. A pH less than 7.35 is indicative of acidosis; a pH above 7.45 indicates alkalosis.

Which child has an increased risk of sudden infant death syndrome (SIDS)? a) Premature infant with low birth weight b) Firstborn child c) Infant hospitalized for fever d) A healthy 2-year-old

a) Premature infant with low birth weight Premature infants, especially those with low birth weight, have an increased risk for SIDS. Infants with apnea, central nervous system disorders, or respiratory disorders also have a higher risk of SIDS. Peak age for SIDS is 2 to 4 months. Hospitalization for fever doesn't affect risk for SIDS. There's an increased risk of SIDS in subsequent siblings of two or more SIDS victims.

Nursing management of a client with a pulmonary embolism focuses on which action? a) Monitoring the oxygen delivery device b) Monitoring for other sources of clots c) Assessing oxygenation status d) Determining whether the client requires another ventilation-perfusion scan

c) Assessing oxygenation status Nursing management of a client with a pulmonary embolism focuses on assessing oxygenation status and ensuring treatment is adequate. If the client's status begins to deteriorate, it's the nurse's responsibility to contact the physician and attempt to improve oxygenation. Monitoring for other clot sources and ensuring the oxygen delivery device is working properly are other nursing responsibilities, but they aren't the focus of care.

A client recovering from a pulmonary embolism is receiving warfarin. To counteract a warfarin overdose, the nurse would administer: a) protamine sulfate. b) heparin. c) vitamin K (phytonadione). d) vitamin C.

c) vitamin K (phytonadione).

A client with pneumonia has just finished dinner. The nurse must calculate the client's fluid intake before taking the tray from the room. The client had 6 oz of soup, 4 oz of milk, and 8 oz of juice. How many milliliters of fluid should the nurse record on the client's intake record? Record your answer using a whole number.

540mL The nurse should know that 1 oz equals 30 mL. Therefore, 18 oz multiplied by 30 equals 540 mL.

The nurse will be assisting the physician with the removal of a patient's chest tube. Which of these measures should be taken 30 to 60 minutes before removing the patient's chest tube? A Administer an analgesic. B Administer an antibiotic. C Check the patient's intake and output. D Check the patient's oxygen saturation level.

A Administer an analgesic. Chest tube removal is often painful, so the nurse should premedicate the patient with an ordered analgesic. The intake and output, as well as the oxygen saturation level, is part of the ongoing care of a patient with chest tubes. Antibiotics are most likely also part of the patient's ongoing care.

The nurse is caring for a patient with viral pneumonia. Which intervention(s) should the nurse include in the care plan? (Select all that apply.) A Providing adequate rest periods B Providing oral hygiene before and after meals C Maintaining adequate fluid intake D Administering organism-specific antibiotics E Monitoring vital signs and respiratory status

A Providing adequate rest periods B Providing oral hygiene before and after meals C Maintaining adequate fluid intake E Monitoring vital signs and respiratory status Antibiotics are only effective for bacterial, not viral, infections. Antibiotics would only be given for secondary bacterial infections.

On initial assessment of a patient diagnosed with an acute exacerbation of chronic obstructive pulmonary disease (COPD), the nurse is likely ti find which signs (select all that apply): 1 tensing of the shoulder muscles 2 unable to tolerate sitting up 3 flaring of the nostrils 4 complete sentences with no effort 5 Sternal retraction

ANS 1,3,5 During an acute exacerbation of chronic obstructive pulmonary disease (COPD), the patient is likely to be sitting and hunched forward in an attempt to open the thoracic cavity. Other signs of respiratory distress include nasal flaring, use of accessory muscles, and gasping for breath between words.

For a patient undergoing mechanical ventilation, which nursing intervention would reduce the insensible fluid loss: 1 suction frequently 2 provide humidified air 3 increase respiratory rate setting 4 repositioning every 2 hours

ANS 2 Humidification provides moisture into the air as the patient inhales. Frequent suctioning and increasing the respiratory rate will actually increase fluid loss. Repositioning is appropriate but unrelated to fluid loss.

The nurse attends to the nutritional needs of the patient with chronic respiratory disease by providing oral care. What is the best rationale for this nursing action: 1 Low energy states diminish appetite 2 Respiratory secretions leave a bad taste 3 Chewing is believed to induce coughing spells 4 Nasal congestion reduces the flavor of food

ANS 2 Mouth care is especially needed before meals, when the taste or odor of the sputum may adversely affect appetite. Low energy, coughing, and nasal congestion may be present, but are not relieved by oral hygiene

The nurse is observing a nursing student who is suctioning a patient for excess secretions. The nurse would intervene immediately if the student: 1 preoxygenated the patient with 100% oxygen for 2 minutes 2 applied suction while inserting the catheter 3 set the wall suction gauge at 80 mm Hg 4 monitored the patient frequently for tachycardia or bradycardia

ANS 2 Suction should not be applied during insertion of the catheter. The other options are included in the correct technique.

The patient is instructed to take in a deep breath and slowly exhale while coughing three times in succession. What is the appropriate rationale for these instructions 1 clears the nasal passages 2 mobilizes secretions and stimulates expectoration 3 increases the likelihood of atelectasis 4 Decreases insensible fluid losses

ANS 2 The first cough mobilizes secretions and the next two bring secretions up to be expectorated. The procedure does not clear the nasal passages or decrease the fluid loss. The nurse would never intentionally instruct a patient in a procedure that would increase the risk for atelectasis (collapse of alveoli).

While obtaining sputum for culture and sensitivity, the nurse notes that the specimen is thick, tenacious, and ropey. This finding is most likely to be present in which disorder: 1 pneumococcal pneumonia 2 pulmonary edema 3 chronci bronchitis 4 tuberculosis

ANS 3

Immediately after the physician incubates a patient who is in respiratory arrest, the nurse notes that there are no breath sounds in the left lung fields, but breath sounds are heard on the right side. The likely explanation is that the endotracheal tube: 1 was placed in the esophagus 2 punctured the left lung 3 was inserted too far 4 is malfunctioning

ANS 3 The right mainstem bronchus is straighter than the left; therefore, the endotracheal (ET) tube will go to the right if inserted past the bifurcation of the mainstem bronchus. If the ET tube is in the esophagus, the stomach will rise when a breath is delivered via Ambu bag, but no breath sounds will be heard. It is unlikely for the endotracheal tube to puncture the lung, and if there was a tube malfunction, sounds would be heard (or not heard) bilaterally.

The nurse is caring for several patients who are scheduled for diagnostic testing for respiratory disorders. The patient who needs postprocedural care that includes frequent vital signs is the patient who had: 1 capnography 2 a D dimer test 3 a ventilation and perfusion scan 4 bronchoscopy

ANS 4 Patient will be sedated; a local anesthetic is given for a bronchoscopy and the scope passes down the throat and into the bronchial tree; therefore, signs of hemorrhage return of gag reflex, and vitals must be checked. D-dimer and capnography are relatively noninvasive, requiring a blood draw and a nasal cannula with monitoring probe, respectively. Ventilation and perfusion scan requires IV administration and inhalation of radioactive material.

The nurse is caring for a patient who has asthma. Which lung sound would the nurse expect to hear when auscultating the patient's lung fields: 1 Fine crackles 2 Stridor 3 Pleural friction rub 4 Wheezes

ANS 4 Wheezing is commonly heard in asthma due to the narrowing of airways. Fine crackles can be heard in patients who have atelectasis, fibrosis, pneumonia, or early congestive heart failure. Stridor can be heard when there is partial obstruction of the upper air passages. Pleural friction rub occurs when irritated visceral and parietal pleura rub against each other.

The nurse is suctioning a patient who is unable to expectorate respiratory secretions from his tracheotomy. How can the nurse avoid the serious consequences of removing oxygen when suctioning this patient? A Do not suction the patient for more than 10 to 15 seconds. B Avoid giving the patient oxygen just prior to suctioning. C Keep suction pressure between 110 and 120 mm Hg. D Apply suction as the catheter is advanced into the trachea.

ANS A Limiting suctioning to 10 to 15 seconds per suction attempt will help prevent the removal of too much oxygen from the patient. The patient should be preoxygenated before suctioning, and the pressure should be between 80 and 100mm Hg. Suction should only be applied while the catheter is removed.

Paul is admitted to the hospital due to metabolic acidosis caused by Diabetic ketoacidosis (DKA). The nurse prepares which of the following medications as an initial treatment for this problem? A. Regular insulin B. Potassium C. Sodium bicarbonate D. Calcium gluconate

ANS A Metabolic acidosis is anaerobic metabolism caused by lack of ability of the body to use circulating glucose. Administration of insulin corrects this problem.

The nurse is assessing an elderly patient who has been admitted with a diagnosis of aspiration pneumonia. As the nurse auscultates the patient's lungs, diminished breath sounds are heard on the right side. Why is the patient more likely to aspirate foreign objects into the right bronchus rather than the left? A The right bronchus is shorter and more horizontal. B The right bronchus is narrower and shorter. C The right bronchus angles off to the right. D The right bronchus is larger and wider.

ANS C The right bronchus branches more sharply from the trachea, making it easier for aspirated objects, such as food, to become lodged there. All of the descriptions comparing the right bronchus with the left bronchus are anatomically incorrect.

A patient arrives in the emergency room holding a towel up to his face, which is saturated with bright red drainage. He tells you he was accidentally "hit with a piece of wood" approximately 30 minutes ago. What is the immediate nursing priority? A Assess the patient's airway patency. B Put on gloves. C Remove the towel. D Apply an ice pack and pressure to the site.

ANS A Airway patency is always the priority. The patient is able to speak and be understood, so he has at least a partial airway. Gloves can be donned at the same time airway patency is being assessed. Removing the towel at this time may remove the pressure that is stopping a larger hemorrhage. Reinforce the area with gauze sponges and notify the physician of the situation. The specific injury has not been determined yet; applying ice and/or pressure may do more damage to the injured area.

How does exhaling affect the pH of the blood? a Exhaling causes the pH to increase (more alkaline). b Exhaling causes the pH to lower (more acidic). c Exhaling causes the pH to be positive. d Exhaling causes the pH to be negative.

ANS A Exhaling releases carbon dioxide, which makes the blood from the respiratory tract more acidic. This has the effect of raising the pH of blood, making it more alkaline. Retaining carbon dioxide would lower the pH of the blood by making it more acidic.

The patient tells the LPN/LVN that she has been hoarse for the past 2½ weeks. Which response by the nurse is most appropriate? A "You should see your primary health care provider." B "Use a high-humidity vaporizer two or three times a day." C "Try to talk as little as possible." D "Gargle with warm, slightly salted water."

ANS A Hoarseness for an extended period of time should be investigated by the primary health care provider. Running a humidifier, talking as little as possible, and gargling with warm salted water are good interventions if the sore throat is found to be due to viral or bacterial infection, but the hoarseness could be due to a more serious cause.

A patient who has a severe upper respiratory infection is placed on antibiotics. Why is it important for the LPN/LVN to remind the patient to finish taking all of the antibiotic medication prescribed? A Taking the entire antibiotic prevents the development of infections that are resistant to the antibiotic. B Taking the entire antibiotic decreases the need for extra fluids. C Taking the entire antibiotic reduces the need for avoidance of others with upper respiratory infections. D Taking the entire antibiotic lessens the chance of getting an upper respiratory infection in the future.

ANS A It is important to remind patients to take every dose of the prescription antibiotic in order to adequately kill all causative microorganisms. Disease-resistant strains of bacteria occur when only a portion of the prescribed dose of antibiotic is taken.

A patient is to have a bronchoscopy. The LPN/ LVN should expect which finding in the postprocedure period? A Blood-tinged sputum B Elevated blood pressure C Difficulty breathing D Elevated temperature

ANS A It is normal to find a small amount of blood-tinged sputum due to the irritation of the respiratory mucosa during the procedure. Elevated blood pressure, elevated temperature, and dyspnea are indications that complications may be occurring postprocedure.

The nurse is suctioning a patient who is unable to expectorate respiratory secretions from his tracheotomy. How can the nurse avoid the serious consequences of removing oxygen when suctioning this patient? A Do not suction the patient for more than 10 to 15 seconds. B Avoid giving the patient oxygen just prior to suctioning. C Keep suction pressure between 110 and 120 mm Hg. D Apply suction as the catheter is advanced into the trachea.

ANS A Limiting suctioning to 10 to 15 seconds per suction attempt will help prevent the removal of too much oxygen from the patient. The patient should be preoxygenated before suctioning, and the pressure should be between 80 and 100mm Hg. Suction should only be applied while the catheter is removed.

It is appropriate to teach patients to obtain sufficient rest to help decrease the frequency with which they contract upper respiratory infections. How does rest help prevent respiratory infections? a Rest assists in keeping the immune system healthy. b Rest allows the body to produce more red blood cells. c Rest reduces the amount of vitamin C that the body excretes. d Rest enhances the functioning of the cough reflex.

ANS A Proper rest and good nutrition help keep the immune system functioning properly, which will decrease the likelihood of contracting respiratory infections. White blood cells rather than red blood cells fight infection; rest does not influence the amount of vitamin C excreted; and rest has not have an effect on the cough reflex.

The LPN/LVN is to assist a patient with a partial laryngectomy with eating. The nurse should understand that it is best to begin by giving which of these types of food? A Semisolid B Fibrous C Liquids D Sweet

ANS A Soft or semisolid foods will be the easiest consistency for the patient to eat initially. Liquids may lead to aspiration, fibrous foods will be difficult to swallow, and most sweet foods lack the nutrients necessary for the healing process.

What is the function of the chemoreceptors in the carotid and aortic bodies? A These chemoreceptors are sensitive to changes in blood carbon dioxide levels. B These chemoreceptors regulate respiratory depth and rhythm. C These chemoreceptors are sensitive to increases in blood oxygen levels. D These chemoreceptors send impulses to the myocardium.

ANS A These receptors monitor arterial carbon dioxide levels. The regulatory centers for respiratory rate and depth are located in the medulla oblongata. Chemoreceptors sensitive to oxygen levels are located in the medulla oblongata. These chemoreceptors communicate directly with the medulla oblongata.

What is the function of the chemoreceptors in the carotid and aortic bodies? a These chemoreceptors are sensitive to changes in blood carbon dioxide levels. b These chemoreceptors regulate respiratory depth and rhythm. c These chemoreceptors are sensitive to increases in blood oxygen levels. d These chemoreceptors send impulses to the myocardium.

ANS A These receptors monitor arterial carbon dioxide levels. The regulatory centers for respiratory rate and depth are located in the medulla oblongata. Chemoreceptors sensitive to oxygen levels are located in the medulla oblongata. These chemoreceptors communicate directly with the medulla oblongata.

A female client is suspected of having a pulmonary embolus. A nurse assesses the client, knowing that which of the following is a common clinical manifestation of pulmonary embolism? a. Dyspnea b. Bradypnea c. Bradycardia d. Decreased respiratory

Answer A. The common clinical manifestations of pulmonary embolism are tachypnea, tachycardia, dyspnea, and chest pain.

During the rehabilitation period, a patient who has had a laryngectomy seems very depressed and tells the LPN/LVN that he does not know how he will manage. Which action is likely to be most helpful? A Arrange to have the patient meet someone who has had the surgery and is managing well. B Try to distract the patient by providing activities he enjoys. C Encourage the patient to think about the things he has achieved in his lifetime. D Ask the patient's significant others to talk with the patient about his feelings.

ANS A Depression is not uncommon when the patient has undergone a drastic lifestyle change as a result of this type of surgery. Having the patient meet with another person with the same condition will be most helpful because this person will know what the patient is experiencing and can offer hope and suggestions for how to manage the condition.

A patient suspected of having a pulmonary abnormality is to have pulmonary angiography. Which nursing interventions should be included in this patient's plan of care? (Select all that apply.) A Explain to the patient that he may feel a warm flush as the dye is injected during the procedure. B Check the pressure dressing for signs of hemorrhage after the procedure. C Monitor vital signs after the procedure. D Provide mouth care every 1 to 2 hours after the procedure. E Assess the sputum for blood.

ANS A, B, C During a pulmonary angiography a radiopaque dye is injected via a catheter into the right side of the heart or the pulmonary artery. Radiographs are taken and fluoroscopy is used. The patient may feel a warm flushing when the dye is injected, and the insertion site for the catheter must be monitored for bleeding. Vital signs are necessary postprocedure to monitor the patient for complications. Mouth care and sputum assessment are not necessary postprocedure since procedure does not invade the respiratory system.

Risk factors for development of a respiratory disorder include: (Select all that apply.) a family history of respiratory disorders. b heavy ethyl alcohol use. c group living conditions. d tobacco use. e contact sports.

ANS A, B, C, D All of these are known risk factors for development of respiratory disorders. There are no data to support a correlation between contact sports and respiratory disorders.

The nurse is caring for a patient following a total laryngectomy. What interventions should the nurse anticipate will be needed? (Select all that apply.) A Perform tracheostomy care. B Maintain aspiration precautions. C Administer gastric tube feedings. D Develop an alternate communication method. E Maintain neutropenic precautions.

ANS A, B, D The patient will first have a laryngectomy tube and then a standard tracheostomy because of postoperative edema that could cause airway obstruction. Aspiration will be a risk initially. Because laryngectomy patients lose the ability to speak, devising an alternative means of communication with the patient's input is a priority. Gastric tube feedings are generally not necessary following laryngectomy. Total parenteral nutrition (TPN) may be provided until the patient is able to safely take adequate food and fluids by mouth. Neutropenia is not generally expected after laryngectomy. Standard infection control measures should be sufficient to prevent infection.

The nurse is teaching the patient about ways he can prevent inflammation in the respiratory tract. Which patient statement(s) demonstrates knowledge of this subject? (Select all that apply.) A I need to stop smoking. B I should be sure to practice good handwashing. C I should take an aspirin every day. D It's a good idea to stay away from crowds during cold and flu season. E I need to get enough rest and eat a balanced diet.

ANS A, B, D, E An aspirin helps maintain good circulation and prevents clots; it does not decrease inflammation. The other comments demonstrate that the patient understands how to decrease inflammation in the respiratory tract. An additional way is by avoiding known allergens

The nurse is teaching the patient about ways he can prevent inflammation in the respiratory tract. Which patient statement(s) demonstrates knowledge of this subject? (Select all that apply.) a I need to stop smoking. b I should be sure to practice good handwashing. c I should take an aspirin every day. d It's a good idea to stay away from crowds during cold and flu season. e I need to get enough rest and eat a balanced diet.

ANS A, B, D, E An aspirin helps maintain good circulation and prevents clots; it does not decrease inflammation. The other comments demonstrate that the patient understands how to decrease inflammation in the respiratory tract. An additional way is by avoiding known allergens.

A patient is to have direct laryngoscopy for removal of polyps. Which of these instructions should the LPN/LVN be sure the patient has been given? A "You will be in a very well-lit room for the procedure." B "Do not eat or drink for 6 to 8 hours before the procedure." C "If you have discomfort after the procedure, warm compresses will be applied." D "You will be told to do coughing and deep-breathing exercises shortly after the procedure."

ANS B A direct laryngoscopy allows visualization of the larynx. The patient should not eat or drink 6 to 8 hours prior to the procedure in order to avoid any nausea or vomiting. Advise the patient that the room will be darkened, and an ice collar may be applied after the procedure to decrease swelling. The patient should also be told that a mild sore throat and hoarseness may occur. Coughing will be discouraged postprocedure to reduce trauma to the area.

The nurse is giving a presentation on vaccinations. Which group of individuals should be encouraged to have an annual vaccination against influenza? A 6- to 12-year-old children B Identified high-risk individuals C People with allergies to feathers D Adults in rural areas

ANS B Certain groups such as the elderly and the chronically ill are considered high-risk individuals and should receive immunization against influenza since this is an effective means of reducing the incidence of respiratory disease. Physicians, nurses, and others involved in providing health care should also be immunized.

George who has undergone thoracic surgery has chest tube connected to a water-seal drainage system attached to suction. Presence of excessive bubbling is identified in water-seal chamber, the nurse should... A. "Strip" the chest tube catheter B. Check the system for air leaks C. Recognize the system is functioning correctly D. Decrease the amount of suction pressure

ANS B Excessive bubbling indicates an air leak which must be eliminated to permit lung expansion.

A male client with pneumonia develops respiratory failure and has a partial pressure of arterial oxygen of 55 mm Hg. He's placed on mechanical ventilation with a fraction of inspired oxygen (FIO2) of 0.9. The nursing goal should be to reduce the FIO2 to no greater than: a. 0.21 b. 0.35 c. 0.5 d. 0.7

Answer C. An FO2 greater than 0.5 for as little as 16 to 24 hours can be toxic and can lead to decreased gas diffusion and surfactant activity. The ideal oxygen source is room air F IO 2 0.18 to 0.21.

A patient presents at the emergency room complaining of severe sore throat "that's so bad I can hardly swallow. It feels like there's a huge lump in my throat." She is diagnosed with severe pharyngitis. What should the nurse include in patient teaching regarding this condition? A Decrease the humidity in her environment. B Increase her fluid intake to 2500 mL a day. C Limit the amount of fruit juice in her diet. D Refrain from taking hot baths or showers.

ANS B Increasing her fluid intake will thin any secretions that develop and keep her from becoming dehydrated. Decreased humidity will thicken secretions and create more difficulty swallowing. Because of the vitamin C content, fruit juices will help the immune system and ability to fight the infection. Hot baths or showers pose no problem for this patient.

Mrs. Chua a 78 year old client is admitted with the diagnosis of mild chronic heart failure. The nurse expects to hear when listening to client's lungs indicative of chronic heart failure would be: A. Stridor B. Crackles C. Wheezes D. Friction rubs

ANS B Left sided heart failure causes fluid accumulation in the capillary network of the lung. Fluid eventually enters alveolar spaces and causes crackling sounds at the end of inspiration.

The nurse wants to obtain information regarding the patient's oxygen (O2) saturation level. What is the least invasive method of obtaining this information? A Perfusion scan B Pulse oximetry C Arterial blood gases (ABGs) D Pulmonary function tests

ANS B Pulse oximetry is a noninvasive method for obtaining the patient's O2 saturation. It is obtained by placing an oxygen sensor on a patient's finger, earlobe, or pinna of the ear; O2 saturation, or percentage of hemoglobin carrying O2, is obtained via a digital readout. Perfusion scans are contrast media studies of pulmonary blood supply. Although a highly accurate method of evaluating arterial oxygen content (PaO2), ABG is invasive since it involves an arterial puncture (usually at the radial artery). Pulmonary tests are noninvasive measurements of lung capacities, but they are not used as a means of obtaining arterial O2 content.

A patient with a sore throat is to have a throat culture to establish whether the infection is being caused by Streptococcus pyogenes. If it is a streptococcal infection and the patient is not treated, what may the patient be at risk for? A Cystitis B Glomerulonephritis C Glaucoma D Hepatitis

ANS B Streptococcus pyogenes can invade the kidney or heart if the infection is left untreated, causing glomerulonephritis or rheumatic fever.

A patient has a pulse oximeter device attached to her left fingertip. Which of these actions should be included in the patient's care? A Ask the patient to keep her left hand exposed to the light. B Report readings that are persistently below normal to the physician. C Remind the patient to keep her left hand at the level of her heart. D Ensure that the left hand is kept dry at all times.

ANS B The pulse oximeter is used to monitor the patient's oxygen saturation level. Persistent low oxygen saturation should be reported to the physician. There is no special care of the finger being used for pulse oximetry other than ensuring no fingernail polish or artificial nails are present on the finger.

The nurse is caring for a patient with epistaxis who is to be taken for x-rays of the skull and face. What nursing interventions will be necessary? (Select all that apply.) A Administering a narcotic analgesic B Compressing the bleeding nostril against the septum and applying ice C Monitoring airway patency D Encouraging the patient to take sips of water E Keeping the patient's head elevated at 30 to 45 degrees

ANS B, C Airway patency is always the priority; compression of the nostril and application of ice to the area are standard actions to control epistaxis. Elevating the patient's head 30 to 45 degrees helps to ensure that drainage from the epistaxis will flow downward into the gastrointestinal (GI) tract, not the respiratory tract. In general, analgesia and oral intake are held until a diagnosis can be made, as well as a determination whether surgery will be necessary.

The student nurse is preparing a presentation on ways to prevent throat cancer. Which factor increases the risk of developing throat cancer? A Eating a high-fat diet B Eating very spicy foods C Drinking alcohol excessively D Overuse of nonsteroidal anti-inflammatory drugs

ANS C Approximately 90% of throat cancer occurs in people who both smoke and immoderately drink alcohol and is four times more common among men.

What does the term hypoxemia indicate? A An excess of oxygen in the blood B An excess of carbon dioxide in the blood C A deficiency of oxygen in the blood D A deficiency of carbon dioxide in the blood

ANS C Hypoxemia means that there is an insufficient amount of oxygen in the blood.

What does the term hypoxemia indicate? a An excess of oxygen in the blood b An excess of carbon dioxide in the blood c A deficiency of oxygen in the blood d A deficiency of carbon dioxide in the blood

ANS C Hypoxemia means that there is an insufficient amount of oxygen in the blood.

Where are the chemoreceptors that regulate breathing located in the body? A Carotid artery and aorta B Jugular vein and left atrium C Cerebellum and pons D Coronary sinus and alveoli

ANS C Regulatory chemoreceptors are found in the brainstem.

Which is a function of the central nervous system? A It controls voluntary respiration only. B It controls involuntary respiration only. C It controls both voluntary and involuntary respiration. D It does not alter respirations.

ANS C The central nervous system controls both involuntary and voluntary respiration via the pons and the medulla.

For which of these reasons is it particularly important for elderly people to receive influenza immunizations? A They cannot tolerate changes in temperature. B They tend to live alone. C They are more susceptible to upper respiratory infections. D They tend not to seek medical assistance soon enough.

ANS C The elderly tend to have a weaker immune system so it is important for them to receive an influenza immunization to help protect them from developing this infection, which can lead to secondary infections.

The nurse is caring for a patient who has had a stroke that has resulted in a weakened cough reflex. What is this patient most at risk for? A Epiglottitis B Esophageal varicosities C Aspiration D Paralysis of the vocal cords

ANS C The patient with a weakened cough reflex is most at risk for aspiration of food or fluids, which often leads to aspiration pneumonia.

Developing respiratory distress in an adult patient include which signs? A Bradycardia, widening pulse pressure, decreasing level of consciousness B Bradycardia, shortness of breath, hypertension C Tachycardia, tachypnea, fatigue D Confusion, decreased urine output, bradycardia

ANS C These are classic signs of inadequate oxygenation. Heart rate increases in an attempt to distribute available oxygen; respiratory rate increases in an attempt to inhale more oxygen; fatigue results from increased energy expenditure. Blood pressure may increase due to increased work of breathing, but bradycardia is not a sign of increasing respiratory distress in an adult. Mental status may decrease, but urine output is not relevant in this case and bradycardia is not associated with increasing dyspnea in an adult.

Developing respiratory distress in an adult patient include which signs? a Bradycardia, widening pulse pressure, decreasing level of consciousness b Bradycardia, shortness of breath, hypertension c Tachycardia, tachypnea, fatigue d Confusion, decreased urine output, bradycardia

ANS C These are classic signs of inadequate oxygenation. Heart rate increases in an attempt to distribute available oxygen; respiratory rate increases in an attempt to inhale more oxygen; fatigue results from increased energy expenditure. Blood pressure may increase due to increased work of breathing, but bradycardia is not a sign of increasing respiratory distress in an adult. Mental status may decrease, but urine output is not relevant in this case and bradycardia is not associated with increasing dyspnea in an adult.

The nurse is caring for a patient who has just had a thoracentesis performed. How should the nurse position the patient after this procedure? A Flat, prone B Flat, supine C Side-lying, on the affected side D Side-lying, on the unaffected side

ANS C Unless specifically ordered otherwise, this is the appropriate position in the immediate post-thoracentesis period. It allows the mattress to be used to apply pressure to the aspiration site, minimizing leakage and bleeding. Flat, while prone or supine, does not allow for full chest expansion. Unless otherwise specified by the physician, lying on the side on which the procedure was performed is preferable.

The student nurse is preparing a presentation on ways to prevent throat cancer. Which factor increases the risk of developing throat cancer? A Eating a high-fat diet B Eating very spicy foods C Drinking alcohol excessively D Overuse of nonsteroidal anti-inflammatory drugs

ANS C Approximately 90% of throat cancer occurs in people who both smoke and immoderately drink alcohol and is four times more common among men.

Which of the following compilations should the nurse carefully monitors a client with acute pancreatitis. A. Myocardial Infarction B. Cirrhosis C. Peptic ulcer D. Pneumonia

ANS D A client with acute pancreatitis is prone to complications associated with respiratory system.

Which of the following complications associated with tracheostomy tube? A. Increased cardiac output B. Acute respiratory distress syndrome (ARDS) C. Increased blood pressure D. Damage to laryngeal nerves

ANS D Tracheostomy tube has several potential complications including bleeding, infection and laryngeal nerve damage.

In what structure does the actual gas exchange during respiration occur? A Lungs B Trachea C Bronchi D Alveoli

ANS D Oxygen and carbon dioxide are exchanged through the walls of the alveoli. The trachea and bronchi are air passageways, but no gas exchange occurs in these structures.

Which of the following antituberculosis drugs can damage the 8th cranial nerve? A. Isoniazid (INH) B. Paraoaminosalicylic acid (PAS) C. Ethambutol hydrochloride (myambutol) D. Streptomycin

ANS D Streptomycin is an aminoglycoside and damage on the 8th cranial nerve (ototoxicity) is a common side effect of aminoglycosides.

A patient's nose begins to bleed. Which action should the LPN/LVN take? A Encourage the patient to blow his nose at frequent intervals. B Tell the patient to keep swallowing. C Apply warm compresses to the nose and face. D Have the patient apply direct pressure by pinching his nose for 10 to 15 minutes.

ANS D The patient should lean forward and apply pressure by pinching his nose for 10 to 15 minutes. Blowing the nose will increase bleeding, as will applying heat. Swallowing may lead to nausea and vomiting from blood entering the stomach.

The nurse is caring for a patient who has recently had a supraglottic laryngectomy. Which nursing diagnosis is most appropriate for this patient? A Risk for meningitis related to loss of larynx B Activity intolerance due to hypoxemia C Fatigue due to hypoxemia D Impaired verbal communication related to loss of larynx

ANS D The patient with a recent laryngectomy will not be able to speak. It will be important for the nurse to assist the patient in an alternative method of communication during the initial postoperative period and to find a new style of speech.

The nurse is caring for a patient following a bronchoscopy. What is an important postprocedure nursing intervention for this patient? A Be sure there is an immediate chest x-ray taken. B Place the patient in the semi-Fowler's position. C Encourage an increased fluid intake. D Check the gag reflex before offering fluids.

ANS D The throat will be sprayed with a local anesthetic preprocedure to numb the gag reflex; therefore, it is important to ensure the gag reflex has returned postprocedure before offering fluids in order to prevent aspiration. There is no need for a chest x-ray postprocedure unless there are complications. The patient should be placed on his or her side to prevent aspiration until the gag reflex has returned.

The nurse is planning the care for a patient who has had a laryngectomy. During the immediate postoperative period, how often will the nurse need to suction the tracheostomy of this patient? A Every 30 minutes B Every 60 minutes C Every 2 hours D Every 4 hours and PRN

ANS D This is the usual patient course for the first several days after surgery. It is imperative that the nurse assess the patient frequently, in case the secretions increase or become tenacious. Suctioning every 30 minutes is usually unnecessary, but patients should be assessed regularly for increased secretions. Every hour or 2 hours is acceptable, but every 4 hours and PRN is the better answer because it allows for individual patient needs

A client is admitted to the health care facility with active tuberculosis. The nurse should include which intervention in the plan of care? A. Putting on a mask when entering the client's room. B. Instructing the client to wear a mask at all times C. Wearing a gown and gloves when providing direct care D. Keeping the door to the client's room open to observe the client

ANS: A Because tuberculosis is transmitted by droplet nuclei from the respiratory tract, the nurse should put on a mask when entering the client's room. Having the client wear a mask at all the times would hinder sputum expectoration and make the mask moist from respirations. If no contact with the client's blood or body fluids is anticipated, the nurse need not wear a gown or gloves when providing direct care. A client with tuberculosis should be in a room with laminar air flow, and the door should be closed at all times.

A nurse is caring for a client who has a tracheostomy and temperature of 39º C. which intervention will most likely lower the client's arterial blood oxygen saturation? A. Endotracheal suctioning B. Encouragement of coughing C. Use of cooling blanket D. Incentive spirometry

ANS: A Endotracheal suctioning secretions as well as gases from the airway and lowers the arterial oxygen saturation (SaO2) level. Coughing and incentive spirometry improve oxygenation and should raise or maintain oxygen saturation. Because of superficial vasoconstriction, using a cooling blanket can lower peripheral oxygen saturation readings, but SaO2 levels wouldn't be affected.

Which statement about fluid replacement is accurate for a client with hyperosmolar hyperglycemic nonketotic syndrome (HHNS)? A. administer 2 to 3 L of IV fluid rapidly B. administer 6 L of IV fluid over the first 24 hours C. administer a dextrose solution containing normal saline solution D. administer IV fluid slowly to prevent circulatory overload and collapse

ANS: A Regardless of the client's medical history, rapid fluid resuscitation is critical for maintaining cardiovascular integrity. Profound intravascular depletion requires aggressive fluid replacement. A typical fluid resuscitation protocol is 6 L of fluid over the first 12 hours, with more fluid to follow over the next 24 hours. Various fluids can be used, depending on the degree of hypovolemia. Commonly prescribed fluids include dextran (in case of hypovolemic shock), isotonic normal saline solution and, when the client is stabilized, hypotonic half-normal saline solution.

For the first 72 hours after thyroidectomy surgery, the nurse would assess the client for Chvostek's sign and Trousseau's sign because they indicate which of the following? A. hypocalcemia B. hypercalcemia C. hypokalemia D. Hyperkalemia

ANS: A The client who has undergone a thyroidectomy is t risk for developing hypocalcemia from inadvertent removal or damage to the parathyroid gland. The client with hypocalcemia will exhibit a positive Chvostek's sign (facial muscle contraction when the facial nerve in front of the ear is tapped) and a positive Trousseau's sign (carpal spasm when a blood pressure cuff is inflated for few minutes). These signs aren't present with hypercalcemia, hypokalemia, or Hyperkalemia.

Before weaning a client from a ventilator, which assessment parameter is most important for the nurse to review? A. fluid intake for the last 24 hours B. baseline arterial blood gas (ABG) levels C. prior outcomes of weaning D. electrocardiogram (ECG) results

ANS: B Before weaning a client from mechanical ventilation, it's most important to have a baseline ABG levels. During the weaning process, ABG levels will be checked to assess how the client is tolerating the procedure. Other assessment parameters are less critical. Measuring fluid volume intake and output is always important when a client is being mechanically ventilated. Prior attempts at weaning and ECG results are documented on the client's record, and the nurse can refer to them before the weaning process begins.

A client is chronically short of breath and yet has normal lung ventilation, clear lungs, and an arterial oxygen saturation (SaO2) 96% or better. The client most likely has: A. poor peripheral perfusion B. a possible Hematologic problem C. a psychosomatic disorder D. left-sided heart failure

ANS: B SaO2 is the degree to which hemoglobin is saturated with oxygen. It doesn't indicate the client's overall Hgb adequacy. Thus, an individual with a subnormal Hgb level could have normal SaO2 and still be short of breath. In this case, the nurse could assume that the client has a Hematologic problem. Poor peripheral perfusion would cause subnormal SaO2. There isn't enough data to assume that the client's problem is psychosomatic. If the problem were left-sided heart failure, the client would exhibit pulmonary crackles.

A client is admitted with a diagnosis of meningitis caused by Neisseria meningitides. The nurse should institute which type of isolation precautions? A. Contact precautions B. Droplet precautions C. Airborne precautions D. Standard precautions

ANS: B This client requires droplet precautions because the organism can be transmitted through airborne droplets when the client coughs, sneezes, or doesn't cover his mouth. Airborne precautions would be instituted for a client infected with tuberculosis. Standard precautions would be instituted for a client when contact with body substances is likely. Contact precautions would be instituted for a client infected with an organism that is transmitted through skin-to-skin contact.

A client comes to the emergency department with chest pain, dyspnea, and an irregular heartbeat. An electrocardiogram shows a heart rate of 110 beats/minute (sinus tachycardia) with frequent premature ventricular contractions. Shortly after admission, the client has ventricular tachycardia and becomes unresponsive. After successful resuscitation, the client is taken to the intensive care unit. Which nursing diagnosis is appropriate at this time? A. Deficient knowledge related to interventions used to treat acute illness B. Impaired physical mobility related to complete bed rest C. Social isolation related to restricted visiting hours in the intensive care unit D. Anxiety related to the threat of death

ANS: D Anxiety related to the threat of death is an appropriate nursing diagnosis because the client's anxiety can adversely affect hear rate and rhythm by stimulating the autonomic nervous system. Also, because the client required resuscitation, the threat of death is a real and immediate concern. Unless anxiety is dealt with first, the client's emotional state will impede learning. Client teaching should be limited to clear concise explanations that reduce anxiety and promote cooperation. An anxious client has difficulty learning, so the deficient knowledge would continue despite attempts teaching. Impaired physical mobility and social isolation are necessitated by the client's critical condition; therefore, they aren't considered problems warranting nursing diagnoses.

After a cerebrovascular accident, a 75 yr old client is admitted to the health care facility. The client has left-sided weakness and an absent gag reflex. He's incontinent and has a tarry stool. His blood pressure is 90/50 mm Hg, and his hemoglobin is 10 g/dl. Which of the following is a priority for this client? a. checking stools for occult blood b. performing range-of-motion exercises to the left side c. keeping skin clean and dry d. elevating the head of the bed to 30 degrees

ANS: D Because the client's gag reflex is absent, elevating the head of the bed to 30 degrees helps minimize the client's risk of aspiration. Checking the stools, performing ROM exercises, and keeping the skin clean and dry are important, but preventing aspiration through positioning is the priority.

A woman whose husband was recently diagnosed with active pulmonary tuberculosis (TB) is a tuberculin skin test converter. Management of her care would include: A. scheduling her for annual tuberculin skin testing B. placing her in quarantine until sputum cultures are negative C. gathering a list of persons with whom she has had recent contact D. advising her to begin prophylactic therapy with isoniazid (INH)

Ans. D Individuals who are tuberculin skin test converters should begin a 6-month regimen of an antitubercular drug such as INH, and they should never have another skin test. After an individual has a positive tuberculin skin test, subsequent skin tests will cause severe skin reactions but won't provide new information about the client's TB status. The client doesn't have active TB, so can't transmit, or spread, the bacteria. Therefore, she shouldn't be quarantined or asked for information about recent contacts.

A male client has been admitted with chest trauma after a motor vehicle accident and has undergone subsequent intubation. A nurse checks the client when the high-pressure alarm on the ventilator sounds, and notes that the client has absence of breathe sounds in right upper lobe of the lung. The nurse immediately assesses for other signs of: a. Right pneumothorax b. Pulmonary embolism c. Displaced endotracheal tube d. Acute respiratory distress syndrome

Answer A. Pneumothorax is characterized by restlessness, tachycardia, dyspnea, pain with respiration, asymmetrical chest expansion, and diminished or absent breath sounds on the affected side. Pneumothorax can cause increased airway pressure because of resistance to lung inflation. Acute respiratory distress syndrome and pulmonary embolism are not characterized by absent breath sounds. An endotracheal tube that is inserted too far can cause absent breath sounds, but the lack of breath sounds most likely would be on the left side because of the degree of curvature of the right and left main stem bronchi.

Nurse Joana is teaching a client with emphysema how to perform pursed-lip breathing. The client asks the nurse to explain the purpose of this breathing technique. Which explanation should the nurse provide? a. It helps prevent early airway collapse. b. It increases inspiratory muscle strength c. It decreases use of accessory breathing muscles. d. It prolongs the inspiratory phase of respiration.

Answer A. Pursed-lip breathing helps prevent early airway collapse. Learning this technique helps the client control respiration during periods of excitement, anxiety, exercise, and respiratory distress. To increase inspiratory muscle strength and endurance, the client may need to learn inspiratory resistive breathing. To decrease accessory muscle use and thus reduce the work of breathing, the client may need to learn diaphragmatic (abdominal) breathing. In pursed-lip breathing, the client mimics a normal inspiratory-expiratory (I:E) ratio of 1:2. (A client with emphysema may have an I:E ratio as high as 1:4.)

A nurse is assessing a male client with chronic airflow limitations and notes that the client has a "barrel chest." The nurse interprets that this client has which of the following forms of chronic airflow limitations? a. Emphysema b. Bronchial asthma c. Chronic obstructive bronchitis d. Bronchial asthma and bronchitis

Answer A. The client with emphysema has hyperinflation of the alveoli and flattening of the diaphragm. These lead to increased anteroposterior diameter, referred to as "barrel chest." The client also has dyspnea with prolonged expiration and has hyperresonant lungs to percussion.

Gina, a home health nurse is visiting a home care client with advanced lung cancer. Upon assessing the client, the nurse discovers wheezing, bradycardia, and a respiratory rate of 10 breaths/minute. These signs are associated with which condition? a. Hypoxia b. Delirium c. Hyperventilation d. Semiconsciousness

Answer A. As the respiratory center in the brain becomes depressed, hypoxia occurs, producing wheezing, bradycardia, and a decreased respiratory rate. Delirium is a state of mental confusion characterized by disorientation to time and place. Hyperventilation (respiratory rate greater than that metabolically necessary for gas exchange) is marked by an increased respiratory rate or tidal volume, or both. Semiconsciousness is a state of impaired consciousness characterized by limited motor and verbal responses and decreased orientation.

A female patient suffers adult respiratory distress syndrome as a consequence of shock. The patient's condition deteriorates rapidly, and endotracheal intubation and mechanical ventilation are initiated. When the high pressure alarm on the mechanical ventilator, alarm sounds, the nurse starts to check for the cause. Which condition triggers the high pressure alarm? a. Kinking of the ventilator tubing b. A disconnected ventilator tube c. An endotracheal cuff leak d. A change in the oxygen concentration without resetting the oxygen level alarm

Answer A. Conditions that trigger the high pressure alarm include kinking of the ventilator tubing, bronchospasm or pulmonary embolus, mucus plugging, water in the tube, coughing or biting on endotracheal tube, and the patient's being out of breathing rhythm with the ventilator. A disconnected ventilator tube or an endotracheal cuff leak would trigger the low pressure alarm. Changing the oxygen concentration without resetting the oxygen level alarm would tigger the oxygen alarm.

Nurse Kim is caring for a client with a pneumothorax and who has had a chest tube inserted notes continuous gentle bubbling in the suction control chamber. What action is appropriate? a. Do nothing, because this is an expected finding. b. Immediately clamp the chest tube and notify the physician. c. Check for an air leak because the bubbling should be intermittent. d. Increase the suction pressure so that bubbling becomes vigorous.

Answer A. Continuous gentle bubbling should be noted in the suction control chamber. Option B is incorrect. Chest tubes should only be clamped to check for an air leak or when changing drainage devices (according to agency policy). Option C is incorrect. Bubbling should be continuous and not intermittent. Option D is incorrect because bubbling should be gentle. Increasing the suction pressure only increases the rate of evaporation of water in the drainage system.

Nurse Lei caring for a client with a pneumothorax and who has had a chest tube inserted notes continues gentle bubbling in the suction control chamber. What action is appropriate? a. Do nothing, because this is an expected finding b. Immediately clamp the chest tube and notify the physician c. Check for an air leak because the bubbling should be intermittent d. Increase the suction pressure so that the bubbling becomes vigorous

Answer A. Continuous gentle bubbling should be noted in the suction control chamber. Option b is incorrect. Chest tubes should only be clamped to check for an air leak or when changing drainage devices (according to agency policy). Option c is incorrect. Bubbling should be continuous and not intermittent. Option d is incorrect because bubbling should be gentle. Increasing the suction pressure only increases the rate of evaporation of water in the drainage system.

Nurse Julia is caring for a client who has a tracheostomy and temperature of 103° F (39.4° C). Which of the following interventions will most likely lower the client's arterial blood oxygen saturation? a. Endotracheal suctioning b. Encouragement of coughing c. Use of cooling blanket d. Incentive spirometry

Answer A. Endotracheal suctioning removes secretions as well as gases from the airway and lowers the arterial oxygen saturation (SaO2) level. Coughing and incentive spirometry improves oxygenation and should raise or maintain oxygen saturation. Because of superficial vasoconstriction, using a cooling blanket can lower peripheral oxygen saturation readings, but SaO2 levels wouldn't be affected.

The nurse is caring for a male client with a chest tube. If the chest drainage system is accidentally disconnected, what should the nurse plan to do? a. Place the end of the chest tube in a container of sterile saline. b. Apply an occlusive dressing and notify the physician. c. Clamp the chest tube immediately. d. Secure the chest tube with tape.

Answer A. If a chest drainage system is disconnected, the nurse may place the end of the chest tube in a container of sterile saline or water to prevent air from entering the chest tube, thereby preventing negative respiratory pressure. The nurse should apply an occlusive dressing if the chest tube is pulled out — not if the system is disconnected. The nurse shouldn't clamp the chest tube because clamping increases the risk of tension pneumothorax. The nurse should tape the chest tube securely to prevent it from being disconnected, rather than taping it after it has been disconnected.

Nurse Murphy administers albuterol (Proventil), as prescribed, to a client with emphysema. Which finding indicates that the drug is producing a therapeutic effect? a. Respiratory rate of 22 breaths/minute b. Dilated and reactive pupils c. Urine output of 40 ml/hour d. Heart rate of 100 beats/minute

Answer A. In a client with emphysema, albuterol is used as a bronchodilator. A respiratory rate of 22 breaths/minute indicates that the drug has achieved its therapeutic effect because fewer respirations are required to achieve oxygenation. Albuterol has no effect on pupil reaction or urine output. It may cause a change in the heart rate, but this is an adverse, not therapeutic, effect.

An unconscious male client is admitted to an emergency room. Arterial blood gas measurements reveal a pH of 7.30, a low bicarbonate level, a normal carbon dioxide level, a normal oxygen level, and an elevated potassium level. These results indicate the presence of: a. Metabolic acidosis b. Respiratory acidosis c. Overcompensated respiratory acidosis d. Combined respiratory and metabolic acidosis

Answer A. In an acidotic condition, the pH would be low, indicating the acidosis. In addition, a low bicarbonate level along with the low pH would indicate a metabolic state. Therefore, options B, C, and D are incorrect.

A nurse is caring for a male client immediately after removal of the endotracheal tube. The nurse reports which of the following signs immediately if experienced by the client? a. Stridor b. Occasional pink-tinged sputum c. A few basilar lung crackles on the right d. Respiratory rate of 24 breaths/min

Answer A. The nurse reports stridor to the physician immediately. This is a high-pitched, coarse sound that is heard with the stethoscope over the trachea. Stridor indicates airway edema and places the client at risk for airway obstruction. Options B, C, and D are not signs that require immediate notification of the physician.

For a male client who has a chest tube connected to a closed water-seal drainage system, the nurse should include which action in the plan of care? a. Measuring and documenting the drainage in the collection chamber b. Maintaining continuous bubbling in the water-seal chamber c. Keeping the collection chamber at chest level d. Stripping the chest tube every hour

Answer A. The nurse should measure and document the amount of chest tube drainage regularly to detect abnormal drainage patterns, such as may occur with a hemorrhage (if excessive) or a blockage (if decreased). Continuous bubbling in the water-seal chamber indicates a leak in the closed chest drainage system, which must be corrected. The nurse should keep the collection chamber below chest level to allow fluids to drain into it. The nurse should not strip chest tubes because doing so may traumatize the tissue or dislodge the tube.

A male client who takes theophylline for chronic obstructive pulmonary disease is seen in the urgent care center for respiratory distress. Once the client is stabilized, the nurse begins discharge teaching. The nurse would be especially vigilant to include information about complying with medication therapy if the client's baseline theophylline level was: a. 10 mcg/mL b. 12 mcg/mL c. 15 mcg/mL d. 18mcg/mL

Answer A. The therapeutic range for the serum theophylline level is 10 to 20 mcg/mL. If the level is below the therapeutic range, the client may experience frequent exacerbations of the disorder. Although all the options identify values within the therapeutic range, option A is the option that reflects a need for compliance with medication.

The amount of air inspired and expired with each breath is called: a. tidal volume. b. residual volume. c. vital capacity. d. dead-space volume.

Answer A. Tidal volume is the amount of air inspired and expired with each breath. Residual volume is the amount of air remaining in the lungs after forcibly exhaling. Vital capacity is the maximum amount of air that can be moved out of the lungs after maximal inspiration and expiration. Dead-space volume is the amount of air remaining in the upper airways that never reaches the alveoli. In pathologic conditions, dead space may also exist in the lower airways.

A male client is to receive I.V. vancomycin (Vancocin). When preparing to administer this drug, the nurse should keep in mind that: a. vancomycin should be infused over 60 to 90 minutes in a large volume of fluid. b. vancomycin may cause irreversible neutropenia. c. vancomycin should be administered rapidly in a large volume of fluid. d. vancomycin should be administered over 1 to 2 minutes as an I.V. bolus.

Answer A. To avoid a hypotensive reaction from rapid I.V. administration, the nurse should infuse vancomycin slowly, over 60 to 90 minutes, in a large volume of fluid. Although neutropenia may occur in approximately 5% to 10% of clients receiving vancomycin, this adverse effect reverses rapidly when the drug is discontinued.

A nurse is caring for a female client diagnosed with tuberculosis. Which assessment, if made by the nurse, is inconsistent with the usual clinical presentation of tuberculosis and may indicate the development of a concurrent problem? a. Cough b. High-grade fever c. Chills and night sweats d. Anorexia and weight loss

Answer B. The client with tuberculosis usually experiences cough (productive or nonproductive), fatigue, anorexia, weight loss, dyspnea, hemoptysis, chest discomfort or pain, chills and sweats (which may occur at night), and a low-grade fever.

Rhea, confused and short breath, is brought to the emergency department by a family member. The medical history reveals chronic bronchitis and hypertension. To learn more about the current respiratory problem, the doctor orders a chest x-ray and arterial blood gas (ABG) analysis. When reviewing the ABG report, the nurses sees many abbreviations. What does a lowercase "a" in ABG value present? a. Acid-base balance b. Arterial Blood c. Arterial oxygen saturation d. Alveoli

Answer B. A lowercase "a" in an ABG value represents arterial blood. For instance, the abbreviation PaO2 refers to the partial pressure of oxygen in arterial blood. The pH value reflects the acid base balance in arterial blood. Sa02 indicates arterial oxygen saturation. An uppercase "A" represents alveolar conditions: for example, PA02 indicates the partial pressure of oxygen in the alveoli.

A male client abruptly sits up in bed, reports having difficulty breathing and has an arterial oxygen saturation of 88%. Which mode of oxygen delivery would most likely reverse the manifestations? a. Simple mask b. Non-rebreather mask c. Face tent d. Nasal cannula

Answer B. A non-rebreather mask can deliver levels of the fraction of inspired oxygen (FIO2) as high as 100%. Other modes — simple mask, face tent and nasal cannula — deliver lower levels of FIO2.

When caring for a male patient who has just had a total laryngectomy, the nurse should plan to: a. Encourage oral feeding as soon as possible b. Develop an alternative communication method c. Keep the tracheostomy cuff fully inflated d. Keep the patient flat in bed

Answer B. A patient with a laryngectomy cannot speak, yet still needs to communicate. Therefore, the nurse should plan to develop an alternative communication method. After a laryngectomy, edema interferes with the ability to swallow and necessitates tube (enteral) feedings. To prevent injury to the tracheal mucosa, the nurse should deflate the tracheostomy cuff or use the minimal leak technique. To decrease edema, the nurse should place the patient in semi-fowler's position.

Nurse Mickey is administering a purified protein derivative (PPD) test to a homeless client. Which of the following statements concerning PPD testing is true? a. A positive reaction indicates that the client has active tuberculosis (TB). b. A positive reaction indicates that the client has been exposed to the disease. c. A negative reaction always excludes the diagnosis of TB. d. The PPD can be read within 12 hours after the injection.

Answer B. A positive reaction means the client has been exposed to TB; it isn't conclusive of the presence of active disease. A positive reaction consists of palpable swelling and induration of 5 to 15 mm. It can be read 48 to 72 hours after the injection. In clients with positive reactions, further studies are usually done to rule out active disease. In immunosuppressed clients, a negative reaction doesn't exclude the presence of active disease.

A female client with interstitial lung disease is prescribed prednisone (Deltasone) to control inflammation. During client teaching, the nurse stresses the importance of taking prednisone exactly as prescribed and cautions against discontinuing the drug abruptly. A client who discontinues prednisone abruptly may experience: a. hyperglycemia and glycosuria. b. acute adrenocortical insufficiency. c. GI bleeding. d. restlessness and seizures.

Answer B. Administration of a corticosteroid such as prednisone suppresses the body's natural cortisol secretion, which may take weeks or months to normalize after drug discontinuation. Abruptly discontinuing such therapy may cause the serum cortisol level to drop low enough to trigger acute adrenocortical insufficiency. Hyperglycemia, glycosuria, GI bleeding, restlessness, and seizures are common adverse effects of corticosteroid therapy, not its sudden cessation.

Before weaning a male client from a ventilator, which assessment parameter is most important for the nurse to review? a. Fluid intake for the last 24 hours b. Baseline arterial blood gas (ABG) levels c. Prior outcomes of weaning d. Electrocardiogram (ECG) results

Answer B. Before weaning a client from mechanical ventilation, it's most important to have baseline ABG levels. During the weaning process, ABG levels will be checked to assess how the client is tolerating the procedure. Other assessment parameters are less critical. Measuring fluid volume intake and output is always important when a client is being mechanically ventilated. Prior attempts at weaning and ECG results are documented on the client's record, and the nurse can refer to them before the weaning process begins.

A nurse is caring for a male client hospitalized with acute exacerbation of chronic obstructive pulmonary disease. Which of the following would the nurse expect to note on assessment of this client? a. Hypocapnia b. A hyperinflated chest noted on the chest x-ray c. Increase oxygen saturation with exercise d. A widened diaphragm noted on the chest x-ray

Answer B. Clinical manifestations of chronic obstructive pulmonary disease (COPD) include hypoxemia, hypercapnia, dyspnea on exertion and at rest, oxygen desaturation with exercise, and the use of accessory muscles of respiration. Chest x-rays reveal a hyperinflated chest and a flattened diaphragm if the disease is advanced.

Nurse Reese is caring for a client hospitalized with acute exacerbation of chronic obstructive pulmonary disease. Which of the following would the nurse expect to note on assessment of this client? a. Hypocapnia b. A hyperinflated chest noted on the chest x-ray c. Increased oxygen saturation with exercise d. A widened diaphragm noted on the chest x-ray

Answer B. Clinical manifestations of chronic obstructive pulmonary disease (COPD) include hypoxemia, hypercapnia, dyspnea on exertion and at rest, oxygen desaturation with exercise, and the use of accessory muscles of respiration. Chest x-rays reveal a hyperinflated chest and a flattened diaphragm if the disease is advanced.

Nurse Eve formulates a nursing diagnosis of Activity intolerance related to inadequate oxygenation and dyspnea for a client with chronic bronchitis. To minimize this problem, the nurse instructs the client to avoid conditions that increase oxygen demands. Such conditions include: a. drinking more than 1,500 ml of fluid daily. b. being overweight. c. eating a high-protein snack at bedtime. d. eating more than three large meals a day.

Answer B. Conditions that increase oxygen demands include obesity, smoking, exposure to temperature extremes, and stress. A client with chronic bronchitis should drink at least 2,000 ml of fluid daily to thin mucus secretions; restricting fluid intake may be harmful. The nurse should encourage the client to eat a high-protein snack at bedtime because protein digestion produces an amino acid with sedating effects that may ease the insomnia associated with chronic bronchitis. Eating more than three large meals a day may cause fullness, making breathing uncomfortable and difficult; however, it doesn't increase oxygen demands. To help maintain adequate nutritional intake, the client with chronic bronchitis should eat small, frequent meals (up to six a day).

Nurse Ryan caring for a client with a chest tube turns the client to the side, and the chest tube accidentally disconnects. The initial nursing action is to: a. Call the physician b. Place the tube in bottle of sterile water c. Immediately replace the chest tube system d. Place a sterile dressing over the disconnection site

Answer B. If the chest drainage system is disconnected, the end of the tube is placed in a bottle of sterile water held below the level of the chest. The system is replaced if it breaks or cracks or if the collection chamber is full. Placing a sterile dressing over the disconnection site will not prevent complications resulting from the disconnection. The physician may need to be notified, but this is not the initial action.

The nurse caring for a male client with a chest tube turns the client to the side, and the chest tube accidentally disconnects. The initial nursing action is to: a. Call the physician. b. Place the tube in a bottle of sterile water. c. Immediately replace the chest tube system. d. Place the sterile dressing over the disconnection site.

Answer B. If the chest drainage system is disconnected, the end of the tube is placed in a bottle of sterile water held below the level of the chest. The system is replaced if it breaks or cracks or if the collection chamber is full. Placing a sterile dressing over the disconnection site will not prevent complications resulting from the disconnection. The physician may need to be notified, but this is not the initial action.

While changing the tapes on a tracheostomy tube, the male client coughs and the tube is dislodged. The initial nursing action is to: a. Call the physician to reinsert the tube. b. Grasp the retention sutures to spread the opening. c. Call the respiratory therapy department to reinsert the tracheotomy. d. Cover the tracheostomy site with a sterile dressing to prevent infection

Answer B. If the tube is dislodged accidentally, the initial nursing action is to grasp the retention sutures and spread the opening. If agency policy permits, the nurse then attempts immediately to replace the tube. Covering the tracheostomy site will block the airway. Options 1 and 3 will delay treatment in this emergency situation.

While changing the tapes on a tracheostomy tube, the male client coughs and tube is dislodged. The initial nursing action is to: a. Call the physician to reinsert the tube b. Grasp the retention sutures to spread the opening c. Call the respiratory therapy department to reinsert the tracheotomy d. Cover the tracheostomy site with a sterile dressing to prevent infection

Answer B. If the tube is dislodged accidentally, the initial nursing action is to grasp the retention sutures and spread the opening. If agency policy permits, the nurse then attempts immediately to replace the tube. Covering the tracheostomy site will block the airway. Options A and C will delay treatment in this emergency situation

On auscultation, which finding suggests a right pneumothorax? a. Bilateral inspiratory and expiratory crackles b. Absence of breaths sound in the right thorax c. Inspiratory wheezes in the right thorax d. Bilateral pleural friction rub.

Answer B. In pneumothorax, the alveoli are deflated and no air exchange occurs in the lungs. Therefore, breath sounds in the affected lung field are absent. None of the other options are associated with pneumothorax. Bilateral crackles may result from pulmonary congestion, inspiratory wheezes may signal asthma, and a pleural friction rub may indicate pleural inflammation.

A female client has a tracheostomy but doesn't require continuous mechanical ventilation. When weaning the client from the tracheostomy tube, the nurse initially should plug the opening in the tube for: a. 15 to 60 seconds. b. 5 to 20 minutes. c. 30 to 40 minutes. d. 45 to 60 minutes.

Answer B. Initially, the nurse should plug the opening in the tracheostomy tube for 5 to 20 minutes, and then gradually lengthen this interval according to the client's respiratory status. A client who doesn't require continuous mechanical ventilation already is breathing without assistance, at least for short periods; therefore, plugging the opening of the tube for only 15 to 60 seconds wouldn't be long enough to reveal the client's true tolerance to the procedure. Plugging the opening for more than 20 minutes would increase the risk of acute respiratory distress because the client requires an adjustment period to start breathing normally.

The nurse is caring for a male client who recently underwent a tracheostomy. The first priority when caring for a client with a tracheostomy is: a. helping him communicate. b. keeping his airway patent. c. encouraging him to perform activities of daily living. d. preventing him from developing an infection.

Answer B. Maintaining a patent airway is the most basic and critical human need. All other interventions are important to the client's well-being but not as important as having sufficient oxygen to breathe.

A male elderly client is admitted to an acute care facility with influenza. The nurse monitors the client closely for complications. What is the most common complication of influenza? a. Septicemia b. Pneumonia c. Meningitis d. Pulmonary edema

Answer B. Pneumonia is the most common complication of influenza. It may be either primary influenza viral pneumonia or pneumonia secondary to a bacterial infection. Other complications of influenza include myositis, exacerbation of chronic obstructive pulmonary disease, and Reye's syndrome. Myocarditis, pericarditis, transverse myelitis, and encephalitis are rare complications of influenza. Although septicemia may arise when any infection becomes overwhelming, it rarely results from influenza. Meningitis and pulmonary edema aren't associated with influenza.

A black male client with asthma seeks emergency care for acute respiratory distress. Because of this client's dark skin, the nurse should assess for cyanosis by inspecting the: a. lips. b. mucous membranes. c. nail beds. d. earlobes.

Answer B. Skin color doesn't affect the mucous membranes. The lips, nail beds, and earlobes are less reliable indicators of cyanosis because they're affected by skin color.

Which of the following would be most appropriate for a male client with an arterial blood gas (ABG) of pH 7.5, PaCO2 26 mm Hg, O2 saturation 96%, HCO3 24 mEq/L, and PaO2 94 mm Hg? a. Administer a prescribed decongestant. b. Instruct the client to breathe into a paper bag. c. Offer the client fluids frequently. d. Administer prescribed supplemental oxygen.

Answer B. The ABG results reveal respiratory alkalosis. The best intervention to raise the PaCO2 level would be to have the client breathe into a paper bag. All of the other options — such as administering a decongestant, offering fluids frequently, and administering supplemental oxygen — wouldn't raise the lowered PaCO2 level.

An oxygen delivery system is prescribed for a male client with chronic obstructive pulmonary disease to deliver a precise oxygen concentration. Which of the following types of oxygen delivery systems would the nurse anticipate to be prescribed? a. Face tent b. Venturi mask c. Aerosol mask d. Tracheostomy collar

Answer B. The Venturi mask delivers the most accurate oxygen concentration. It is the best oxygen delivery system for the client with chronic airflow limitation because it delivers a precise oxygen concentration. The face tent, aerosol mask, and tracheostomy collar are also high-flow oxygen delivery systems but most often are used to administer high humidity.

A nurse is caring for a male client with acute respiratory distress syndrome. Which of the following would the nurse expect to note in the client? a. Pallor b. Low arterial PaO2 c. Elevated arterial PaO2 d. Decreased respiratory rate

Answer B. The earliest clinical sign of acute respiratory distress syndrome is an increased respiratory rate. Breathing becomes labored, and the client may exhibit air hunger, retractions, and cyanosis. Arterial blood gas analysis reveals increasing hypoxemia, with a PaO2 lower than 60 mm Hg.

A male patient has a sucking stab wound to the chest. Which action should the nurse take first? a. Drawing blood for a hematocrit and hemoglobin level b. Applying a dressing over the wound and taping it on three sides c. Preparing a chest tube insertion tray d. Preparing to start an I.V. line

Answer B. The nurse immediately should apply a dressing over the stab wound and tape it on three sides to allow air to escape and to prevent tension pneumothorax (which is more life-threatening than an open chest wound). Only after covering and taping the wound should the nurse draw blood for laboratory tests, assist with chest tube insertion, and start an I.V. line.

A male adult client with cystic fibrosis is admitted to an acute care facility with an acute respiratory infection. Prescribed respiratory treatment includes chest physiotherapy. When should the nurse perform this procedure? a. Immediately before a meal b. At least 2 hours after a meal c. When bronchospasms occur d. When secretions have mobilized

Answer B. The nurse should perform chest physiotherapy at least 2 hours after a meal to reduce the risk of vomiting and aspiration. Performing it immediately before a meal may tire the client and impair the ability to eat. Percussion and vibration, components of chest physiotherapy, may worsen bronchospasms; therefore, the procedure is contraindicated in clients with bronchospasms. Secretions that have mobilized (especially when suction equipment isn't available) are a contraindication for postural drainage, another component of chest physiotherapy.

A nurse has assisted a physician with the insertion of a chest tube. The nurse monitors the adult client and notes fluctuation of the fluid level in the water seal chamber after the tube is inserted. Based on this assessment, which action would be appropriate? a. Inform the physician. b. Continue to monitor the client. c. Reinforce the occlusive dressing. d. Encourage the client to deep-breathe.

Answer B. The presence of fluctuation of the fluid level in the water seal chamber indicates a patent drainage system. With normal breathing, the water level rises with inspiration and falls with expiration. Fluctuation stops if the tube is obstructed, if a dependent loop exists, if the suction is not working properly, or if the lung has reexpanded. Options A, C, and D are incorrect.

Nurse Maureen has assisted a physician with the insertion of a chest tube. The nurse monitors the client and notes fluctuation of the fluid level in the water seal chamber after the tube is inserted. Based on this assessment, which action would be appropriate? a. Inform the physician b. Continue to monitor the client c. Reinforce the occlusive dressing d. Encourage the client to deep-breathe

Answer B. The presence of fluctuation of the fluid level in the water seal chamber indicates a patent drainage system. With normal breathing, the water level rises with inspiration and falls with expiration. Fluctuation stops if the tube is obstructed, if a dependent loop exists, if the suction is not working properly, or if the lung has reexpanded. Options A, C, and D are incorrect.

An emergency room nurse is assessing a female client who has sustained a blunt injury to the chest wall. Which of these signs would indicate the presence of a pneumothorax in this client? a. A low respiratory b. Diminished breathe sounds c. The presence of a barrel chest d. A sucking sound at the site of injury

Answer B. This client has sustained a blunt or a closed chest injury. Basic symptoms of a closed pneumothorax are shortness of breath and chest pain. A larger pneumothorax may cause tachypnea, cyanosis, diminished breath sounds, and subcutaneous emphysema. Hyperresonance also may occur on the affected side. A sucking sound at the site of injury would be noted with an open chest injury.

An emergency room nurse is assessing a male client who has sustained a blunt injury to the chest wall. Which of these signs would indicate the presence of a pneumothorax in this client? a. A low respiratory rate b. Diminished breath sounds c. The presence of a barrel chest d. A sucking sound at the site of injury

Answer B. This client has sustained a blunt or a closed chest injury. Basic symptoms of a closed pneumothorax are shortness of breath and chest pain. A larger pneumothorax may cause tachypnea, cyanosis, diminished breath sounds, and subcutaneous emphysema. Hyperresonance also may occur on the affected side. A sucking sound at the site of injury would be noted with an open chest injury.

Nurse Hannah is preparing to obtain a sputum specimen from a client. Which of the following nursing actions will facilitate obtaining the specimen? a. Limiting fluids b. Having the clients take three deep breaths c. Asking the client to split into the collection container d. Asking the client to obtain the specimen after eating

Answer B. To obtain a sputum specimen, the client should rinse the mouth to reduce contamination, breathe deeply, and then cough into a sputum specimen container. The client should be encouraged to cough and not spit so as to obtain sputum. Sputum can be thinned by fluids or by a respiratory treatment such as inhalation of nebulized saline or water. The optimal time to obtain a specimen is on arising in the morning.

The nursing instructor asks a nursing student to describe the route of transmission of tuberculosis. The instructor concludes that the student understands this information if the student states that the tuberculosis is transmitted by: a. Hand and mouth b. The airborne route c. The fecal-oral route d. Blood and body fluids

Answer B. Tuberculosis is an infectious disease caused by the bacillus Mycobacterium tuberculosis and is spread primarily by the airborne route. Options A, C, and D are incorrect.

A nurse performs an admission assessment on a female client with a diagnosis of tuberculosis. The nurse reviews the result of which diagnosis test that will confirm this diagnosis? a. Bronchoscopy b. Sputum culture c. Chest x-ray d. Tuberculin skin test

Answer B. Tuberculosis is definitively diagnosed through culture and isolation of Mycobacterium tuberculosis. A presumptive diagnosis is made based on a tuberculin skin test, a sputum smear that is positive for acid-fast bacteria, a chest x-ray, and histological evidence of granulomatous disease on biopsy.

A nurse performs an admission assessment on a female client with a diagnosis of tuberculosis. The nurse reviews the results of which diagnostic test that will confirm this diagnosis? a. Bronchoscopy b. Sputum culture c. Chest x-ray d. Tuberculin skin test

Answer B. Tuberculosis is definitively diagnosed through culture and isolation of Mycobacterium tuberculosis. A presumptive diagnosis is made based on a tuberculin skin test, a sputum smear that is positive for acid-fast bacteria, a chest x-ray, and histological evidence of granulomatous disease on biopsy.

A female client with chest injury has suffered flail chest. A nurse assesses the client for which most distinctive sign of flail chest? a. Cyanosis b. Hypotension c. Paradoxical chest movement d. Dyspnea, especially on exhalation

Answer C. Flail chest results from fracture of two or more ribs in at least two places each. This results in a "floating" section of ribs. Because this section is unattached to the rest of the bony rib cage, this segment results in paradoxical chest movement. This means that the force of inspiration pulls the fractured segment inward, while the rest of the chest expands. Similarly, during exhalation, the segment balloons outward while the rest of the chest moves inward. This is a telltale sign of flail chest.

A male patient's X-ray result reveals bilateral white-outs, indicating adult respiratory distress syndrome (ARDS). This syndrome results from: a. Cardiogenic pulmonary edema b. Respiratory alkalosis c. Increased pulmonary capillary permeability d. Renal failure

Answer C. ARDS results from increased pulmonary capillary permeability, which leads to noncardiogenic pulmonary edema. In cardiogenic pulmonary edema, pulmonary congestion occurs secondary to heart failure. In the initial stage of ARDS, respiratory alkalosis may arise secondary to hyperventilation; however, it does not cause ARDS. Renal failure does not cause ARDS, either.

For a male client with chronic obstructive pulmonary disease, which nursing intervention would help maintain a patent airway? a. Restricting fluid intake to 1,000 ml/day b. Enforcing absolute bed rest c. Teaching the client how to perform controlled coughing d. Administering prescribed sedatives regularly and in large amounts

Answer C. Controlled coughing helps maintain a patent airway by helping to mobilize and remove secretions. A moderate fluid intake (usually 2 L or more daily) and moderate activity help liquefy and mobilize secretions. Bed rest and sedatives may limit the client's ability to maintain a patent airway, causing a high risk of infection from pooled secretions.

A nurse is suctioning fluids from a female client through an endotracheal tube. During the suctioning procedure, the nurse notes on the monitor that the heart rate is decreasing. Which of the following is the appropriate nursing intervention? a. Continue to suction. b. Notify the physician immediately. c. Stop the procedure and reoxygenate the client. d. Ensure that the suction is limited to 15 seconds.

Answer C. During suctioning, the nurse should monitor the client closely for side effects, including hypoxemia, cardiac irregularities such as a decrease in heart rate resulting from vagal stimulation, mucosal trauma, hypotension, and paroxysmal coughing. If side effects develop, especially cardiac irregularities, the procedure is stopped and the client is reoxygenated.

Before seeing a newly assigned female client with respiratory alkalosis, the nurse quickly reviews the client's medical history. Which condition is a predisposing factor for respiratory alkalosis? a. Myasthenia gravis b. Type 1 diabetes mellitus c. Extreme anxiety d. Narcotic overdose

Answer C. Extreme anxiety may lead to respiratory alkalosis by causing hyperventilation, which results in excessive carbon dioxide (CO2) loss. Other conditions that may set the stage for respiratory alkalosis include fever, heart failure, and injury to the brain's respiratory center, overventilation with a mechanical ventilator, pulmonary embolism, and early salicylate intoxication. Type 1 diabetes mellitus may lead to diabetic ketoacidosis; the deep, rapid respirations occurring in this disorder (Kussmaul's respirations) don't cause excessive CO2 loss. Myasthenia gravis and narcotic overdose suppress the respiratory drive, causing CO2 retention, not CO2 loss; this may lead to respiratory acidosis, not alkalosis.

A nurse is suctioning fluids from a male client via a tracheostomy tube. When suctioning, the nurse must limit the suctioning time to a maximum of: a. 1 minute b. 5 seconds c. 10 seconds d. 30 seconds

Answer C. Hypoxemia can be caused by prolonged suctioning, which stimulates the pacemaker cells in the heart. A vasovagal response may occur, causing bradycardia. The nurse must preoxygenate the client before suctioning and limit the suctioning pass to 10 seconds.

A nurse is caring for a female client after a bronchoscope and biopsy. Which of the following signs, if noted in the client, should be reported immediately to the physicians? a. Dry cough b. Hematuria c. Bronchospasm d. Blood-streaked sputum

Answer C. If a biopsy was performed during a bronchoscopy, blood-streaked sputum is expected for several hours. Frank blood indicates hemorrhage. A dry cough may be expected. The client should be assessed for signs of complications, which would include cyanosis, dyspnea, stridor, bronchospasm, hemoptysis, hypotension, tachycardia, and dysrhythmias. Hematuria is unrelated to this procedure.

A nurse is teaching a male client with chronic respiratory failure how to use a metered-dose inhaler correctly. The nurse instructs the client to: a. Inhale quickly b. Inhale through the nose c. Hold the breath after inhalation d. Take two inhalations during one breath

Answer C. Instructions for using a metered-dose inhaler include shaking the canister, holding it right side up, inhaling slowly and evenly through the mouth, delivering one spray per breath, and holding the breath after inhalation.

Pulmonary disease (COPD), which nursing action best promotes adequate gas exchange? a. Encouraging the client to drink three glasses of fluid daily b. Keeping the client in semi-Fowler's position c. Using a high-flow Venturi mask to deliver oxygen as prescribed d. Administering a sedative as prescribed

Answer C. The client with COPD retains carbon dioxide, which inhibits stimulation of breathing by the medullary center in the brain. As a result, low oxygen levels in the blood stimulate respiration, and administering unspecified, unmonitored amounts of oxygen may depress ventilation. To promote adequate gas exchange, the nurse should use a Venturi mask to deliver a specified, controlled amount of oxygen consistently and accurately. Drinking three glasses of fluid daily wouldn't affect gas exchange or be sufficient to liquefy secretions, which are common in COPD. Clients with COPD and respiratory distress should be placed in high Fowler's position and shouldn't receive sedatives or other drugs that may further depress the respiratory center.

A female client is receiving supplemental oxygen. When determining the effectiveness of oxygen therapy, which arterial blood gas value is most important? a. pH b. Bicarbonate (HCO3-) c. Partial pressure of arterial oxygen (PaO2) d. Partial pressure of arterial carbon dioxide (PaCO2)

Answer C. The most significant and direct indicator of the effectiveness of oxygen therapy is the PaO2 value. Based on the PaO2 value, the nurse may adjust the type of oxygen delivery (cannula, venturi mask, or mechanical ventilator), flow rate, and oxygen percentage. The other options reflect the client's ventilation status, not oxygenation.

A nurse is taking pulmonary artery catheter measurements of a male client with acute respiratory distress syndrome. The pulmonary capillary wedge pressure reading is 12mm Hg. The nurse interprets that this readings is: a. High and expected b. Low and unexpected c. Normal and expected d. Uncertain and unexpected

Answer C. The normal pulmonary capillary wedge pressure (PCWP) is 8 to 13 mm Hg, and the client is considered to have high readings if they exceed 18 to 20 mm Hg. The client with acute respiratory distress syndrome has a normal PCWP, which is an expected finding because the edema is in the interstitium of the lung and is noncardiac.

A nurse instructs a female client to use the pursed-lip method of breathing and the client asks the nurse about the purpose of this type of breathing. The nurse responds, knowing that the primary purpose of pursed-lip breathing is to: a. Promote oxygen intake b. Strengthen the diaphragm c. Strengthen the intercostal muscles d. Promote carbon dioxide elimination

Answer D. Pursed-lip breathing facilitates maximal expiration for clients with obstructive lung disease. This type of breathing allows better expiration by increasing airway pressure that keeps air passages open during exhalation. Options A, B, and C are not the purposes of this type of breathing.

For a patient with advance chronic obstructive pulmonary disease (COPD), which nursing action best promotes adequate gas exchange? a. Encouraging the patient to drink three glasses of fluid daily b. Keeping the patient in semi-fowler's position c. Using a high-flow venture mask to deliver oxygen as prescribe d. Administering a sedative, as prescribe

Answer C. The patient with COPD retains carbon dioxide, which inhibits stimulation of breathing by the medullary center in the brain. As a result, low oxygen levels in the blood stimulate respiration, and administering unspecified, unmonitored amounts of oxygen may depress ventilation. To promote adequate gas exchange, the nurse should use a Venturi mask to deliver a specified, controlled amount of oxygen consistently and accurately. Drinking three glasses of fluid daily would not affect gas exchange or be sufficient to liquefy secretions, which are common in COPD. Patients with COPD and respiratory distress should be places in high-Fowler's position and should not receive sedatives or other drugs that may further depress the respiratory center

After undergoing a left pneumonectomy, a female patient has a chest tube in place for drainage. When caring for this patient, the nurse must: a. Monitor fluctuations in the water-seal chamber b. Clamp the chest tube once every shift c. Encourage coughing and deep breathing d. Milk the chest tube every 2 hours

Answer C. When caring for a patient who is recovering from a pneumonectomy, the nurse should encourage coughing and deep breathing to prevent pneumonia in the unaffected lung. Because the lung has been removed, the water-seal chamber should display no fluctuations. Reinflation is not the purpose of chest tube. Chest tube milking is controversial and should be done only to remove blood clots that obstruct the flow of drainage.

A nurse instructs a female client to use the pursed-lip method of breathing and the client asks the nurse about the purpose of this type of breathing. The nurse responds, knowing that the primary purpose of pursed-lip breathing is to: a. Promote oxygen intake. b. Strengthen the diaphragm. c. Strengthen the intercostal muscles. d. Promote carbon dioxide elimination.

Answer D. Pursed-lip breathing facilitates maximal expiration for clients with obstructive lung disease. This type of breathing allows better expiration by increasing airway pressure that keeps air passages open during exhalation. Options A, B, and C are not the purposes of this type of breathing.

A male client is admitted to the health care facility for treatment of chronic obstructive pulmonary disease. Which nursing diagnosis is most important for this client? a. Activity intolerance related to fatigue b. Anxiety related to actual threat to health status c. Risk for infection related to retained secretions d. Impaired gas exchange related to airflow obstruction

Answer D. A patent airway and an adequate breathing pattern are the top priority for any client, making impaired gas exchange related to airflow obstruction the most important nursing diagnosis. The other options also may apply to this client but are less important.

A male patient is admitted to the health care facility for treatment of chronic obstructive pulmonary disease. Which nursing diagnosis is most important for this patient? a. Activity intolerance related to fatigue b. Anxiety related to actual threat to health status c. Risk for infection related to retained secretions d. Impaired gas exchange related to airflow obstruction

Answer D. A patient airway and an adequate breathing pattern are the top priority for any patient, making "impaired gas exchange related to airflow obstruction" the most important nursing diagnosis. The other options also may apply to this patient but less important.

A female client has just returned to a nursing unit following bronchoscopy. A nurse would implement which of the following nursing interventions for this client? a. Administering atropine intravenously b. Administering small doses of midazolam (Versed) c. Encouraging additional fluids for the next 24 hours d. Ensuring the return of the gag reflex before offering food or fluids

Answer D. After bronchoscopy, the nurse keeps the client on NPO status until the gag reflex returns because the preoperative sedation and local anesthesia impair swallowing and the protective laryngeal reflexes for a number of hours. Additional fluids are unnecessary because no contrast dye is used that would need flushing from the system. Atropine and midazolam would be administered before the procedure, not after.

Before administering ephedrine, Nurse Tony assesses the patient's history. Because of ephedrine's central nervous system (CNS) effects, it is not recommended for: a. Patients with an acute asthma attack b. Patients with narcolepsy c. Patients under age 6 d. Elderly patients

Answer D. Ephedrine is not recommended for elderly patients, who are particularly susceptible to CNS reactions (such as confusion and anxiety) and to cardiovascular reactions (such as increased systolic blood pressure, coldness in the extremities, and anginal pain). Ephedrine is used for its bronchodilator effects with acute and chronic asthma and occasionally for its CNS stimulant actions for narcolepsy. It can be administered to children age 2 and older.

Blessy, a community health nurse is conducting an educational session with community members regarding tuberculosis. The nurse tells the group that one of the first symptoms associated with tuberculosis is: a. Dyspnea b. Chest pain c. A bloody, productive cough d. A cough with the expectoration of mucoid sputum

Answer D. One of the first pulmonary symptoms is a slight cough with the expectoration of mucoid sputum. Options A, B, and C are late symptoms and signify cavitation and extensive lung involvement.

A male adult patient on mechanical ventilation is receiving pancuronium bromide (Pavulon), 0.01 mg/kg I.V. as needed. Which assessment finding indicates that the patient needs another pancuronium dose? a. Leg movement b. Finger movement c. Lip movement d. Fighting the ventilator

Answer D. Pancuronium, a nondepolarizing blocking agent, is used for muscle relaxation and paralysis. It assists mechanical ventilation by promoting encdotracheal intubation and paralyzing the patient so that the mechanical ventilator can do its work. Fighting the ventilator is a sign that the patient needs another pancuronium dose. The nurse should administer 0.01 to 0.02 mg/kg I.V. every 20 to 60 minutes. Movement of the legs, or lips has no effect on the ventilator and therefore is not used to determine the need for another dose.

A patient who experienced high fever and chills, a productive cough, chest pain, general malaise, and aching muscles during the past week is admitted to the hospital. The nurse anticipates that diagnostic tests will reveal which disease? A Type A influenza B Pneumonia C Pleurisy with effusion D Streptococcus empyema

B Pneumonia Typical signs and symptoms of pneumonia include high fever, chills, a cough that produces rusty or blood-flecked sputum, sweating, chest pain, and a general feeling of malaise and aching muscles.

A nurse is assessing the respiratory status of a male client who has suffered a fractured rib. The nurse would expect to note which of the following? a. Slow deep respirations b. Rapid deep respirations c. Paradoxical respirations d. Pain, especially with inspiration

Answer D. Rib fractures are a common injury, especially in the older client, and result from a blunt injury or a fall. Typical signs and symptoms include pain and tenderness localized at the fracture site and exacerbated by inspiration and palpation, shallow respirations, splinting or guarding the chest protectively to minimize chest movement, and possible bruising at the fracture site. Paradoxical respirations are seen with flail chest.

At 11 p.m., a male client is admitted to the emergency department. He has a respiratory rate of 44 breaths/minute. He's anxious, and wheezes are audible. The client is immediately given oxygen by face mask and methylprednisolone (Depo-medrol) I.V. At 11:30 p.m., the client's arterial blood oxygen saturation is 86% and he's still wheezing. The nurse should plan to administer: a. alprazolam (Xanax). b. propranolol (Inderal) c. morphine. d. albuterol (Proventil).

Answer D. The client is hypoxemic because of bronchoconstriction as evidenced by wheezes and a subnormal arterial oxygen saturation level. The client's greatest need is bronchodilation, which can be accomplished by administering bronchodilators. Albuterol is a beta2 adrenergic agonist, which causes dilation of the bronchioles. It's given by nebulization or metered-dose inhalation and may be given as often as every 30 to 60 minutes until relief is accomplished. Alprazolam is an anxiolytic and central nervous system depressant, which could suppress the client's breathing. Propranolol is contraindicated in a client who's wheezing because it's a beta2 adrenergic antagonist. Morphine is a respiratory center depressant and is contraindicated in this situation.

A nurse is assessing a female client with multiple trauma who is at risk for developing acute respiratory distress syndrome. The nurse assesses for which earliest sign of acute respiratory distress syndrome? a. Bilateral wheezing b. Inspiratory crackles c. Intercostal retractions d. Increased respiratory rate

Answer D. The earliest detectable sign of acute respiratory distress syndrome is an increased respiratory rate, which can begin from 1 to 96 hours after the initial insult to the body. This is followed by increasing dyspnea, air hunger, retraction of accessory muscles, and cyanosis. Breath sounds may be clear or consist of fine inspiratory crackles or diffuse coarse crackles.

A male adult patient hospitalized for treatment of a pulmonary embolism develops respiratory alkalosis. Which clinical findings commonly accompany respiratory alkalosis? a. Nausea or vomiting b. Abdominal pain or diarrhea c. Hallucinations or tinnitus d. Lightheadedness or paresthesia

Answer D. The patient with respiratory alkalosis may complain of lightheadedness or paresthesia (numbness and tingling in the arms and legs). Nausea, vomiting, abdominal pain, and diarrhea may accompany respiratory acidosis. Hallucinations and tinnitus rare are associated with respiratory alkalosis or any other acid-base imbalance.

A female client with asthma is receiving a theophylline preparation to promote bronchodilation. Because of the risk of drug toxicity, the nurse must monitor the client's serum theophylline level closely. The nurse knows that the therapeutic theophylline concentration falls within which range? a. 1 to 2 mcg/ml b. 2 to 5 mcg/ml c. 5 to 10 mcg/ml d. 10 to 20 mcg/ml

Answer D. The therapeutic serum theophylline concentration ranges from 10 to 20 mcg/ml. Values below 10 mcg/ml aren't therapeutic.

Nurse Ruth assessing a patient for tracheal displacement should know that the trachea will deviate toward the: a. Contralateral side in a simple pneumothorax b. Affected side in a hemothorax c. Affected side in a tension pneumothorax d. Contralateral side in hemothorax

Answer D. The trachea will shift according to the pressure gradients within the thoracic cavity. In tension pneumothorax and hemothorax, accumulation of air or fluid causes a shift away from the injured side. If there is no significant air or fluid accumulation, the trachea will not shift. Tracheal deviation toward the contralateral side in simple pneumothorax is seen when the thoracic contents shift in response to the release of normal thoracic pressure gradients on the injured side.

The position of a conscious client during suctioning is: a. Fowler's b. Supine position c. Side-lying d. Prone

Answer: A. Position a conscious person who has a functional gag reflex in the semi fowler's position with the head turned to one side for oral suctioning or with the neck hyper extended for nasal suctioning. If the client is unconscious place the patient a lateral position facing you.

Presence of overdistended and non-functional alveoli is a condition called: a. Bronchitis b. Emphysema c. Empyema d. Atelectasis

Answer: B. An overdistended and non-functional alveoli is a condition called emphysema. Atelectasis is the collapse of a part or the whole lung. Empyema is the presence of pus in the lung.

Two days after tonsillectomy, Mark's reported that his stool is black. Initially, the nurse should: a. Inform the physician b. Document the findings c. Obtain stool for analysis d. Check the client's vital signs

Answer: B. after tonsillectomy, the client's stool will be black or dark for a few days due to the swallowed blood

Which of the following medications is avoided in sinusitis to prevent the risk of developing nasal polyps? a. Codeine b. Amoxicillin c. ASA d. Anti-infectives

Answer: C. ASA is avoided in sinusitis as it increases the risk of developing nasal polyps.

A Mantoux test signifies exposure to Mycobacterium Tubercle Bacilli. The test is read for how many hours after injection? a. 1 hour b. 12-24 hours c. 48-72 hours d. 2 hours

Answer: C. Mantoux test is read 48-72 hours after injection

A client is about to undergo a chest x-ray. Which of the following should the nurse do first before the procedure? a. Secure a written consent b. Instruct the client not to eat anything at the night before the procedure c. Instruct the client to remove metals from the chest d. Administer atropine sulfate and valium before the procedure

Answer: C. The client is instructed to remove metals from the chest. Consent, NPO post midnight and atropine sulfate administration are not needed for this procedure.

Mark underwent a tonsillectomy procedure. To promote comfort the following interventions should be done by the nurse except: a. Application of ice collar b. Assist the client to a semi-fowler's position with pillow support c. Assess for frequent swallowing of the patient d. Administration of acetaminophen

Answer: C. to promote comfort the nurse must administer acetaminophen to alleviate pain from the operation. Also, ice collar can be applied to decrease pain thus, promoting comforting. Assisting the client to the proper position after tonsillectomy (semi-fowler's) while supporting the position with pillows is also one way to promote comfort. The only choice that does not promote comfort but monitors for hemorrhage is the assessment for frequent swallowing, thus, it is the correct answer (C).

Before the nurse's shift ended, the water seal bottle is observed to have an intermittent suctioning. The nurse should do which of the following? a. Check for an air leak b. Check for kinks in the tube c. Inform the physician immediately d. Make sure that the bottle is at least 2-3 feet below the level of the chest

Answer: D. A water seal bottle is expected to be observed for intermittent bubbling. Therefore, the nurse should only make sure that bottle is at least 2-3 feet below the chest. Checking for an air leak would be the nurse's action if the water seal has continuous bubbling. Kinks should be check for possible obstruction if no bubbling is noted in the water seal bottle.

After thoracentesis the client should be placed on which position? a. Affected side b. Unaffected side c. Prone position d. Supine position

Answer: D. After thoracentesis the client is placed or turned to the unaffected side to prevent leakage of fluid in the thoracic cavity.

Asthma can be caused by extrinsic and intrinsic factors. Presence of these factors triggers the release of the chemical mediators which does not include: a. Serotonin b. Prostaglandin c. Bradykinin d. Adrenaline

Answer: D. Allergy (extrinsic) and inflammation (intrinsic) triggers the release of chemical mediators that causes narrowing of the airways and spasm. These mediators are: • Serotonin • Prostaglandin • Bradykinin • Histamine • Leukotrienes

A client is suspected to have an HIV. The nurse knows that in the Mantoux test result, a client is considered positive with HV if he has an induration of: a. More than 10 mm b. 3mm c. 4 mm d. 5 mm

Answer: D. Mantoux test is considered positive for Mycobacterium Tubercle Bacilli if the induration is 10 mm or more. For HIV positive clients, induration of 5mm is considered positive.

A teen ager is diagnosed to have inflamed tonsils (tonsillitis). The patient's history reveals recurrent tonsillitis episodes for about 6 times of the same year. The most appropriate intervention for the patient is: a. Promoting rest b. Increasing fluid intake c. Warm saline gargle d. Surgery

Answer: D. Surgery is indicated for patients with tonsillitis recurring 5-6 times a year.

The most important action the nurse should do before and after suctioning a client is: a. Placing the client in a supine position b. Making sure that suctioning takes only 10-15 seconds c. Evaluating for clear breath sounds d. Hyperventilating the client with 100% oxygen

Answer: D. The client should be hyperventilated with 100% oxygen before and after suctioning.

The patient recently diagnosed with asthma asks the nurse what type of medication is usually used to manage the disease. Which response by the nurse is correct? A "Antiviral agents are used most often." B "Bronchodilators are the most frequently used medication." C "Antihistamines are the most common drug used for asthma." D "Liquefying agents are the first-line drugs used to treat asthma."

B "Bronchodilators are the most frequently used medication." Bronchodilators are the main course of therapy for treating asthma since the goal is to open air passages. Other medications frequently used for asthma include corticosteroids and mucolytics.

A patient being treated for active pulmonary tuberculosis asks the nurse when he will be permitted to return to work. Which response by the nurse is correct? A "When you have a negative tuberculin test reading, you can to return to work." B "You can return to work after three consecutive negative sputum specimens, 2 to 4 weeks apart." C "You can return to work after you have taken your medication for 6 weeks." D "After you have a BCG (bacille Calmette-Guérin) vaccination you can return to work."

B "You can return to work after three consecutive negative sputum specimens, 2 to 4 weeks apart." When three consecutive sputum cultures are negative, the patient is considered no longer infectious and may resume work and other usual social activities. Until this time, the patient will need to wear a mask when in contact with crowds until the medication has effectively suppressed the infection. Precautions are not needed while at home since the family members have already been exposed by the time of diagnosis.

A patient with asthma is experiencing difficulty breathing, tachypnea, and wheezing. Which medication, administered through an inhaler, should the nurse administer to this patient? A Cromolyn (Intal) B Albuterol (Proventil) C Salmeterol (Serevent) D Formoterol (Foradil)

B Albuterol (Proventil) Albuterol (Proventil) is a fast-acting bronchodilator and can be given during an acute episode of asthma symptoms. Its onset of action is 5 to 10 minutes and duration is 3 to 4 hours. Cromolyn (Intal) is an inhaled anti-inflammatory agent that may take up to 2 weeks to produce a therapeutic effect. Salmeterol (Serevent) and Formoterol (Foradil) are long-acting bronchodilators that should not be used to treat acute episodes of asthma.

The student nurse is preparing a report about COPD. The student would be correct in including which disease(s) in her report? (Select all that apply.) A Pulmonary tuberculosis B Pleurisy with effusion C Chronic bronchitis D Bronchial asthma E Emphysema

C Chronic bronchitis E Emphysema Chronic bronchitis and emphysema are categorized as COPD. They are obstructive pulmonary disorders characterized by problems with moving air into and out of the lungs and progressive dyspnea. Asthma is considered a chronic airflow limitation (CAL) disorder

The nurse is teaching an asthma patient the appropriate way to use an inhaler. Which action indicates the patient requires additional teaching? A She takes a deep breath and holds it for 3 or 4 seconds. B She places the inhaler mouthpiece beyond her lips, into her mouth. C She inhales with her lips tightly sealed to the mouthpiece. D She exhales slowly before inhaling the medication, using purse-lip breathing.

C She inhales with her lips tightly sealed to the mouthpiece. Keeping the lips tightly sealed encourages nasal breathing, which interferes with effectiveness of the inhaler.

Which instruction should the nurse give a patient who is to use an inhaler with a bronchodilating agent? A "Take the drug on an empty stomach." B "Take the drug 15 to 20 minutes before exercising." C "Do not drink water within 10 minutes of taking the drug." D "Do not take the drug after 8 P.M."

D "Do not take the drug after 8 P.M." Bronchodilators often cause tachycardia, palpitations, nervousness, tremors, and insomnia; therefore, these medications should be avoided before bedtime.

A patient with lung cancer is scheduled for intrathoracic surgery to remove the tumor. What teaching point should the nurse include in preoperative teaching? A How to move in bed with the use of an overhead trapeze B To expect restricted fluid intake C Mastering pursed lip breathing D Arm and leg exercises

D Arm and leg exercises It will be important to include arm and leg exercises to avoid thrombophlebitis in the lower extremities and problems with movement of the arm on the operative side. Other preoperative teaching focuses on techniques to increase lung ventilation postoperatively. Information is given about chest tubes, suctioning, mechanical ventilation, and use of an incentive spirometer.

A patient with emphysema may lose weight despite having an adequate caloric intake. What advice should the nurse give the patient regarding ways to maintain an optimal weight? A Continue the same caloric intake but increase fat intake. B Increase activity level to stimulate appetite. C Increase amounts of complex carbohydrates and decrease fats. D Decrease the amount of complex carbohydrates and increase calories, protein, vitamins, and minerals.

D Decrease the amount of complex carbohydrates and increase calories, protein, vitamins, and minerals. Due to the amount of energy expended for breathing, COPD patients often need additional calories, protein, vitamins, and minerals. Patients require more calories but not more fats. Increasing activity will increase oxygen demand and result in further weight loss.

Positioning is an important aspect of the care of a postoperative intrathoracic surgery patient. Which nursing intervention is appropriate for a patient who just returned from having a pneumonectomy? A Always turn this patient onto the left side. B Always turn this patient onto the right side. C Never turn this patient onto the unoperated side. D Never turn this patient onto the operated side.

D Never turn this patient onto the operated side. Which side this patient is turned to depends on which is his operative side. Turning a patient with a pneumonectomy onto his operative side may cause a tension pneumothorax and mediastinal shift, or the bronchial stump where the lung was removed could leak and the patient could drown in accumulated fluid.

The nurse is caring for a patient who is at risk for thrombus formation due to a total hip replacement. Which symptom should alert the nurse to the possibility that a pulmonary embolus (PE) has occurred? A The patient complains of sharp pain in his calf. B The patient becomes bradycardic. C The patient develops hypotension. D The patient complains of dyspnea.

D The patient complains of dyspnea. Dyspnea is the most indicative sign of a PE in this patient. Hypotension is also a sign of PE, but it is not as indicative as dyspnea. Tachycardia rather than bradycardia occurs with PE. The sharp pain in the calf may be a sign of thrombophlebitis.

A home health care nurse visits a client with chronic obstructive pulmonary disease who requires oxygen. Which statement by the client indicates the need for additional teaching about home oxygen use? a) "I make sure my oxygen mask is on tightly, so it won't fall off while I nap." b) "I have a 'No Smoking' sign posted at my front entry-way to remind guests not to smoke." c) "I lubricate my lips and nose with K-Y jelly." d) "I clean my mask with water after every meal."

a) "I make sure my oxygen mask is on tightly, so it won't fall off while I nap." Applying an oxygen mask too tightly can cause skin breakdown. The client should be cautioned against wearing it too tightly. Oxygen therapy is drying to the oral and nasal mucosa, so the client should be encouraged to apply a water-soluble lubricant, such as K-Y jelly, to prevent drying. Smoking is contraindicated wherever oxygen is in use. Posting of a "No Smoking" sign warns people against smoking in the client's house. Cleaning the mask with water 2 or 3 times a day removes secretions and decreases the risk for infection. (less)

A nurse is providing discharge teaching for a client recently exposed to tuberculosis (TB) and started on isoniazid and rifampin. Which of the following statements, if made by the client, indicates understanding of the nurse's instructions? Select all that apply. a) "My urine, tears, and saliva may turn bright orange." b) "I will call my doctor if I develop abdominal pain or vomiting." c) "I should notify my physician if I notice bruising or yellowing of my skin." d) "I will need to take this medication for 6-8 weeks." e) "I will stop taking the medicine immediately if it causes nausea."

a) "My urine, tears, and saliva may turn bright orange." b) "I will call my doctor if I develop abdominal pain or vomiting." c) "I should notify my physician if I notice bruising or yellowing of my skin." The combination of isoniazid and rifampin is a common course of treatment for clients exposed to TB. Abdominal pain and vomiting may indicate inflammation of the liver. Isoniazid and rifampin may cause nausea. Once liver damage is ruled out, this is not a reason to discontinue the medication. Bruising and yellowing of the skin may also indicate liver damage, which can be caused by isoniazid and rifampin and should be reported immediately. These medications should be continued for 6 months to 1 year. Discoloration of body fluids is a side effect of rifa

A client arrives at the emergency department with a flail chest. Data collection reveals that a client has paradoxical chest expansion. When asked by the nursing instructor to best describe this expansion, how does the nursing student respond? a) "The lung movement is outward during expiration and inward during inspiration." b) "It is a decreased respiratory rate." c) "The lung movement is inward during expiration and outward during inspiration." d) "It is a form of tachypnea."

a) "The lung movement is outward during expiration and inward during inspiration." In paradoxical chest expansion, the lungs move outward during expiration and inward during inspiration. The client may exhibit signs of ineffective gas exchange, such as tachypnea (an abnormally fast respiratory rate), secondary to a paradoxical breathing pattern.

A nurse is preparing to reinforce the teaching plan for a client who has recently been diagnosed with squamous cell carcinoma of the left lung. Which statement by the nurse is correct? a) "You have a slow-growing cancer that rarely spreads." b) "Squamous cell cancer is a very rapid-growing cancer." c) "The cancer has generally metastasized by the time the diagnosis is made." d) "In terms of prognosis, you may have only a few months to live."

a) "You have a slow-growing cancer that rarely spreads." Squamous cell carcinoma of the lung is a slow-growing, rarely metastasizing type of cancer. It has the best prognosis of all lung cancer types.

A young adult was told that he had a significant reaction to the Mantoux test. The client asks the nurse what is the meaning of this significant reaction. How does the nurse appropriately respond? a) "You have been exposed to tuberculosis." b) "You are immunocompromised." c) "You had active tuberculosis." d) "You have active tuberculosis."

a) "You have been exposed to tuberculosis." A reaction to the Mantoux test for tuberculosis means that the client has been exposed to the tuberculin bacillus. Further testing needs to be done to determine whether the disease is active or dormant. A positive reaction doesn't mean the client is immunocompromised, but clients who are immunocompromised have a high risk of tuberculosis.

A client is admitted to the intensive care unit with multiple trauma and acute respiratory insufficiency. The physician orders arterial blood gas (ABG) analysis to determine the client's ventilatory and gas exchange status. Because the client's arms are in casts from above his elbow to his fingertips, the clinician takes the ABG sample from the femoral artery. After the sample is drawn, the nurse should apply continuous pressure to the puncture site for: a) 10 minutes. b) 8 minutes. c) 3 minutes. d) 5 minutes.

a) 10 minutes. After a blood sample is drawn from the femoral artery, the nurse should apply continuous pressure to the puncture site for 10 minutes to prevent bleeding. Applying pressure for shorter periods wouldn't allow enough time for clotting to occur. The radial and brachial sites require only 5 minutes of continuous pressure.

After diagnosing a client with pulmonary tuberculosis, the physician tells family members that they must receive isoniazid as prophylaxis against tuberculosis. The client's teenage daughter asks the nurse how long the drug must be taken. What appropriate timeline does the nurse provide for the duration of prophylactic isoniazid therapy? a) 6 to 12 months b) 3 to 5 days c) 1 to 3 weeks d) 2 to 4 months

a) 6 to 12 months Prophylactic isoniazid therapy must continue for 6 to 12 months at a daily dosage of 300 mg. Taking the drug for less than 6 months may not provide adequate protection against tuberculosis.

A child is to receive furosemide (Lasix) 4 mg/kg/day in one daily dose. The child weighs 20 kg. How many milligrams should be administered in each dose? a) 80 mg b) 160 mg c) 20 mg d) 40 mg

a) 80 mg The child should receive 80 mg per dose. Use the following equation: 4 mg/kg × 20 kg = 80 mg.

A nurse is working in a walk-in clinic. She has been alerted that there's an outbreak of tuberculosis (TB). Which client entering the clinic today is most likely to have TB? a) A 43-year-old homeless man with a history of alcoholism b) A 54-year-old businessman c) A 16-year-old female high-school student d) A 33-year-old day-care worker

a) A 43-year-old homeless man with a history of alcoholism

A client with a primary tuberculosis (TB) infection can expect to develop which condition? a) A positive skin test b) Active TB within 2 weeks c) Active TB within 1 month d) A fever requiring hospitalization

a) A positive skin test A primary TB infection occurs when the bacillus has successfully invaded the entire body after entering through the lungs. At this point, the bacilli are walled off and skin tests read positive. However, all but infants and immunosuppressed people will remain asymptomatic. The general population has a 10% risk of developing active TB over their lifetime, often because of a break in the body's immune defenses. The active stage shows the classic symptoms of TB: fever, hemoptysis, and night sweats

Which intervention is appropriate for care of the child with cystic fibrosis? a) Administer chest physiotherapy two to four times per day. b) Decrease exercise and limit physical activity. c) Administer bronchodilator or nebulizer treatments after chest physiotherapy. d) Administer cough suppressants and antihistamines.

a) Administer chest physiotherapy two to four times per day. Chest physiotherapy is recommended two to four times per day to help loosen and move secretions to facilitate expectoration. Exercise and physical activity are recommended to stimulate mucus secretion and to establish a good habitual breathing pattern. Cough suppressants and antihistamines are contraindicated. The goal is for the child to be able to cough and expectorate mucus secretions. Bronchodilator or nebulizer treatments are given before chest physiotherapy to help open the bronchi for easier expectoration.

Which nursing activity requires greater caution when performed on a client with chronic obstructive pulmonary disease (COPD)? a) Administering opioids for pain relief b) Assisting the client with coughing and deep breathing c) Monitoring the client's cardiac rhythm d) Increasing the client's fluid intake

a) Administering opioids for pain relief Opioids suppress the respiratory center in the medulla. Both COPD and pneumonia cause alterations in gas exchange; any further problems with oxygenation could result in respiratory failure and cardiac arrest. Increasing the fluid intake would help to thin the client's secretions. Although the nurse would need to monitor the intake and output and watch for signs of heart failure, this isn't as critical as administering opioids. The cardiac rhythm provides an indication of the client's myocardial oxygenation; it should be a part of the nurse's regular assessment. Helping with coughing and deep breathing should be included in the plan of care. The only caution would be to assess for possible rupture of emphysematous alveolar sacs and pneumothorax

An elderly female client is diagnosed with pulmonary tuberculosis. Upset and tearful, she asks the nurse how long she must be separated from her family. Which nursing diagnosis is appropriate for this client? a) Deficient knowledge b) Impaired social interaction c) Social isolation d) Anxiety

a) Deficient knowledge This client is exhibiting a knowledge deficit because treatment of tuberculosis no longer requires isolation, provided that the client complies with the prescribed medication regimen. Although the client is upset, her question reflects sadness at the prospect of being separated from her family rather than anxiety about the disease. Because she has just been diagnosed and hasn't had a chance to demonstrate compliance, a nursing diagnosis of Social isolation isn't appropriate. A diagnosis of Impaired social interaction usually has a psychiatric or neurologic basis, not a respiratory one such as pulmonary tuberculosis.

A client with a productive cough, chills, and night sweats is suspected of having active tuberculosis (TB). The physician should take which action? a) Admit him to the hospital in respiratory isolation. b) Prescribe isoniazid, and tell him to go home and rest. c) Give a tuberculin skin test, and tell him to come back in 48 hours to have it read. d) Give a prescription for isoniazid, 300 mg daily for 2 weeks, and send him home.

a) Admit him to the hospital in respiratory isolation. This client is showing signs and symptoms of active TB and, because of the productive cough, is highly contagious. He should be admitted to the hospital and placed in respiratory isolation, and three sputum cultures should be obtained to confirm the diagnosis. He would most likely be given isoniazid and two or three other antitubercular antibiotics until the diagnosis is confirmed, and then isolation and treatment would continue if the cultures were positive for TB. After 7 to 10 days, three more consecutive sputum cultures will be obtained. If they're negative, he would be considered noncontagious and may be sent home, although he'll continue to take the antitubercular drugs for 9 to 12 months

Which characteristic distinguishes allergies from colds? a) Allergies trigger constant and consistent bouts of sneezing. b) Colds cause itching of the eyes and nose. c) Allergies are accompanied by fever. d) Skin tests can diagnose a cold.

a) Allergies trigger constant and consistent bouts of sneezing. Allergies elicit consistent bouts of sneezing, are seldom accompanied by fever, and tend to cause itching of the eyes and nose. Skin testing is performed to determine the child's sensitivity to specific allergens. Colds are accompanied by fever and are characterized by sporadic sneezing

A client, confused and short of breath, is brought to the emergency department by a family member. The medical history reveals chronic bronchitis and hypertension. To learn more about the client's current respiratory problem, the physician orders a chest X-ray and arterial blood gas (ABG) analysis. When reviewing the ABG report, the nurse sees many abbreviations and interprets the lowercase "a" in an ABG value to represent which factor? a) Arterial blood b) Acid-base balance c) Alveoli d) Arterial oxygen saturation

a) Arterial blood A lowercase "a" in an ABG value represents arterial blood. For instance, the abbreviation PaO2 refers to the partial pressure of oxygen in arterial blood. The pH value reflects the acid-base balance in arterial blood. The abbreviation SaO2 indicates arterial oxygen saturation. An upper-case "A" represents alveolar conditions; for example, PAO2 indicates the partial pressure of oxygen in the alveoli.

A nurse is caring for a client diagnosed with a deep vein thrombosis who develops dyspnea and restlessness. What is the best action of the nurse? a) Assess the client's oxygen saturation (SaO2) level. b) Notify the health care provider. c) Assist the client to a more comfortable position. d) Document the findings and continue to monitor.

a) Assess the client's oxygen saturation (SaO2) level. The client has symptoms consistent with a pulmonary embolism (PE). Assessing the client's oxygenation status is the priority before contacting the health care provider. Many clients begin treatment for PE on the basis of history and the clinical examination, before definitive diagnostic testing has been completed. The key to timely diagnosis is identifying risk factors for the development of a PE. Documenting the findings and monitoring is not appropriate care. Changing the client's position will not help with diagnosing or treating a PE.

For a client with an endotracheal (ET) tube, which nursing action is most essential? a) Auscultating the lungs for bilateral breath sounds b) Turning the client from side to side every 2 hours c) Monitoring serial blood gas values every 4 hours d) Providing frequent oral hygiene

a) Auscultating the lungs for bilateral breath sounds For a client with an ET tube, the most important nursing action is auscultating the lungs regularly for bilateral breath sounds to ensure proper tube placement and effective oxygen delivery. Although the other options are appropriate for this client, they're secondary to ensuring adequate oxygenation.

A client with end-stage pulmonary hypertension tells his physician that he doesn't want any heroic measures should his heart stop and that he doesn't want to be placed on a ventilator. The physician enters a do-not-resuscitate order into the hospital's computer system. Which ethical principle is a nurse upholding by supporting the client's decision? a) Autonomy b) Beneficence c) Nonmaleficence d) Justice

a) Autonomy Autonomy is the client's right to make his own decisions. This client made the decision to have no heroic measures, so the nurse who supports this is upholding the principle of autonomy. Nonmaleficence is the duty to "do no harm." Beneficence is the quality of doing good. Justice is equated with fairness.

A client with acute asthma showing inspiratory and expiratory wheezes and a decreased forced expiratory volume should be treated immediately with which class of medication? a) Bronchodilators b) Oral steroids c) Beta-adrenergic blockers d) Inhaled steroids

a) Bronchodilators Bronchodilators are the first line of treatment for asthma because bronchoconstriction is the cause of reduced airflow. Inhaled or oral steroids may be given to reduce the inflammation but aren't used for emergency relief. Beta-adrenergic blockers aren't used to treat asthma and can cause bronchoconstriction.

The nurse is caring for a client with a fracture of the right femur caused by a skiing accident. Which is an early sign of fat emboli? a) Confusion b) Abdominal cramping c) Fatty stools d) Numbness in the right foot

a) Confusion Confusion and irritability are signs of hypoxia, which is caused by the fat emboli traveling to the lungs and producing an inflammatory response in the lung tissue. Abdominal cramping may be a sign of abdominal distention and constipation caused by immobility. Fatty stools occur with pancreatitis. Numbness may be secondary to neurovascular impairment.

When a chest tube is accidentally dislodged from a client, which intervention should the nurse perform first? a) Cover the chest tube insertion site opening with petroleum gauze, and apply pressure b) Auscultate the lung fields for breath sounds c) Wipe the chest tube with alcohol and reinsert d) Notify the physician

a) Cover the chest tube insertion site opening with petroleum gauze, and apply pressure If a chest tube is accidentally dislodged, immediately cover the insertion site opening with petroleum gauze and apply pressure to prevent air from entering the chest and causing tension pneumothorax. Next, notify the physician. It isn't appropriate to attempt to reinsert the chest tube. Auscultation of the lungs may be important but isn't the priority.

Which complication is most commonly related to acute otitis media? a) Eardrum perforation b) Hearing loss c) Meningitis d) Tympanosclerosis

a) Eardrum perforation Eardrum perforation is the most common complication of acute otitis media as the exudate accumulates and pressure increases. Hearing loss in most cases is conductive in nature and mild in severity but is less common than eardrum perforation. Hearing tests aren't usually performed during episodes of otitis media. Meningitis and tympanosclerosis are possible but uncommon when adequate antibiotic therapy is implemented.

A client comes to the emergency department reporting sudden onset of diarrhea, anorexia, malaise, cough, headache, and recurrent chills. Based on the client's history and physical findings, the physician suspects Legionnaires' disease. While awaiting diagnostic test results, the client is admitted to the facility and started on antibiotic therapy. Which medication does the nurse expect for the physician to order as it is considered the drug of choice for treating this disease? a) Erythromycin b) Amantadine c) Amphotericin B d) Rifampin

a) Erythromycin Erythromycin is the drug of choice for treating Legionnaires' disease. Rifampin may be added to the regimen if erythromycin alone is ineffective; however, it isn't administered first. Amantadine, an antiviral agent, and amphotericin B, an antifungal agent, are ineffective against Legionnaires' disease, which is caused by bacterial infection.

Which intervention is most appropriate for helping parents to cope with a child newly diagnosed with bronchopulmonary dysplasia? a) Evaluate and assess parents' stress and anxiety levels. b) Help parents identify necessary lifestyle changes. c) Refer them to support groups. d) Teach cardiopulmonary resuscitation.

a) Evaluate and assess parents' stress and anxiety levels. The emotional impact of bronchopulmonary dysplasia is clearly a crisis situation. The parents are experiencing grief and sorrow over the loss of a "healthy" child. The other strategies are more appropriate for long-term intervention.

A ventilation-perfusion scan is frequently performed to diagnose a pulmonary embolism. This test provides what type of information? a) Extent of the occlusion and amount of perfusion lost b) Location of the pulmonary embolism c) Amount of perfusion present in the lung d) Location and size of the pulmonary embolism

a) Extent of the occlusion and amount of perfusion lost The ventilation-perfusion scan provides information on the extent of occlusion caused by the pulmonary embolism and the amount of lung tissue involved in the area not perfused.

Which statement is appropriate for the nurse to make to the parents of a child with cystic fibrosis who are planning to have a second child? a) Genetic counseling is recommended. b) There's a 50% chance that the child will be affected. c) There's a 50% chance that the child will be normal. d) There's a 25% chance that the child will only be a carrier.

a) Genetic counseling is recommended. Genetic counseling should be recommended. Cystic fibrosis is an autosomal-recessive disease. Therefore, there's a 25% chance of the child having the disease, a 25% chance of the child being normal, and a 50% chance of the child being a carrier. (

Clients with chronic illnesses are more likely to get pneumonia when which situation is present? a) Group living b) Severe periodontal disease c) Malnutrition d) Dehydration

a) Group living Clients with chronic illness generally have poor immune systems. Typically, residing in group living situations increases the chance of disease transmission. Adequate fluid intake, adequate nutrition, and proper oral hygiene help maintain normal defenses and can reduce the incidence of getting such diseases as pneumonia.

For a client with impaired gas exchange, which position is best? a) High Fowler's b) Lateral decubitus c) Supine d) Semi-Fowler's

a) High Fowler's For a client with impaired gas exchange, high Fowler's is the best position because it allows maximal chest expansion. If the client can't tolerate high Fowler's position, semi-Fowler's is the next best choice because it increases comfort and allows chest expansion. The lateral decubitus and supine positions don't promote chest expansion

A 48-year-old client is admitted for suspected pulmonary emboli. Upon arrival in the intensive care unit, the client is alert and oriented. He insists on anxiously walking around the room. Which nursing actions take priority for this client? a) Initiate bed rest with the head of the bed elevated at least 45 degrees, administer supplemental oxygen, and monitor the client's respiratory status. b) Auscultate breath sounds, assess the client for chest discomfort, and check his neurovascular status. c) Assess the client's vital signs, respiratory status, and neurovascular status, and initiate bed rest. d) Assess the client's vital signs, chest pain, and breath sounds.

a) Initiate bed rest with the head of the bed elevated at least 45 degrees, administer supplemental oxygen, and monitor the client's respiratory status. Pulmonary emboli are clots that form an obstruction of the pulmonary arterial bed. Danger signs and symptoms include changes in vital signs, pleuritic chest discomfort, tachypnea, dyspnea, and apprehension. The client must be immediately stabilized and further assessed. Continued nursing care focuses on the improvement of respiratory gas exchange, maintenance of optimal cardiac output, reduction of anxiety, and pain relief. Initiating bed rest with the head of the bed elevated to at least 45 degrees and administering supplemental oxygen as prescribed take priority because they decrease the body's oxygen demand and facilitate gas exchange. The other interventions are important but don't take priority.

How does positive end-expiratory pressure (PEEP) improve oxygenation? a) It opens up collapsed alveoli and helps keep them open b) It provides more oxygen to the client c) It opens up bronchioles and allows oxygen to get in the lungs d) It adds pressure to the lung tissue during inhalation

a) It opens up collapsed alveoli and helps keep them open PEEP delivers positive pressure to the lung at the end of expiration. This helps open collapsed alveoli and helps them stay open so gas exchange can occur in these newly opened alveoli, improving oxygenation. The bronchioles don't participate in gas exchange except to act as a conduit for inspired and expired air. The alveolar walls are rigid enough; they generally don't collapse. PEEP doesn't directly add pressure to the lung tissue or provide more oxygen to the client

Occasionally, clients with acute respiratory distress syndrome are placed in the prone position. How does this position help the client? a) It recruits more alveoli b) It makes the client more comfortable c) It prevents skin breakdown d) It improves cardiac output

a) It recruits more alveoli Turning the client to the prone position may recruit new alveoli in the posterior region of the lung and improve oxygenation status. Cardiac output shouldn't be affected by the prone position. Skin breakdown can still occur over the new pressure points. Generally the client is obtunded when this measure is used. If not, he should be well-sedated.

An elderly client with pneumonia has a nursing diagnosis of Ineffective airway clearance. Which intervention would be most appropriate? a) Monitoring the need for suctioning every hour b) Asking the physician for an order to suction c) Suctioning every hour d) Suctioning once per shift

a) Monitoring the need for suctioning every hour Suctioning should be performed only when necessary, based on the client's condition at the time of assessment. Suctioning is a nursing procedure and doesn't require a physician's order.

The right forearm of a client who had a purified protein derivative (PPD) test for tuberculosis (TB) is reddened and raised about 3 mm where the test was given. This PPD would be read as having which result? a) Negative b) Positive c) Indeterminate d) Needs to be redone

a) Negative This test would be classed as negative. A 3-mm raised area would be a positive result if a client had recent close contact with someone diagnosed with or suspected of having infectious TB. In that case follow-up would be done and a chest X-ray should be ordered. The test can be redone in 6 months to see if the client's test results change. If the PPD test is reddened and raised 10 mm or more, it's considered positive according to the Centers for Disease Control and Prevention. Indeterminate isn't a term used to describe results of a PPD test.

A client abruptly sits up in bed, reports having difficulty breathing and has an arterial oxygen saturation of 88%. Which mode of oxygen delivery would most likely reverse the manifestations? a) Nonrebreather mask b) Nasal cannula c) Simple mask d) Face tent

a) Nonrebreather mask A nonrebreather mask can deliver levels of the fraction of inspired oxygen (FIO2) as high as 100%. Other modes — simple mask, face tent, and nasal cannula — deliver lower levels of FIO2.

A nurse has advised a client's family that they shouldn't increase the oxygen flow rate. Which rationale is most correct for not increasing the oxygen level on a client when it isn't needed? a) Oxygen toxicity may reduce the amount of functional alveolar surface area b) Increasing the oxygen level may cause pulmonary barotrauma c) Increased oxygen may decrease carbon dioxide levels and cause apnea d) Extra oxygen may cause the client to breathe too rapidly

a) Oxygen toxicity may reduce the amount of functional alveolar surface area Oxygen toxicity causes direct pulmonary trauma, reducing the amount of alveolar surface area available for gaseous exchange, which results in increased carbon dioxide levels and decreased oxygen uptake. Excessive oxygen therapy may eliminate hypoxic respiratory drive, causing the patient to breathe too slowly or even to stop breathing. Pulmonary barotrauma is caused by high lung pressures, not excessive oxygenation

During a pneumonectomy, the phrenic nerve on the surgical side is typically cut to cause hemidiaphragm paralysis. Why is this done? a) Paralyzing the diaphragm reduces the space left by the pneumonectomy b) Cutting the phrenic nerve is a mistake during surgery c) The client isn't using that lung to breathe any longer d) Paralyzing the diaphragm reduces oxygen demand

a) Paralyzing the diaphragm reduces the space left by the pneumonectomy Because the hemidiaphragm is a muscle that doesn't contract when paralyzed, an uncontracted hemidiaphragm remains in an "up" position, which reduces the space left by the pneumonectomy. Serous fluid has less space to fill, thus reducing the extent and duration of mediastinal shift after surgery. Although it's true that the client no longer needs the hemidiaphragm on the operative side to breathe, this alone wouldn't be sufficient justification for cutting the phrenic nerve. Paralyzing the hemidiaphragm doesn't significantly decrease total-body oxygen demand.

Pulse oximetry gives what type of information about the client? a) Percentage of hemoglobin carrying oxygen b) Amount of oxygen in the blood c) Respiratory rate d) Amount of carbon dioxide in the blood

a) Percentage of hemoglobin carrying oxygen Pulse oximetry determines the percentage of hemoglobin carrying oxygen. This doesn't ensure that the oxygen being carried through the bloodstream is actually being taken up by the tissue. Pulse oximetry doesn't provide information about the amount of oxygen or carbon dioxide in the blood or the client's respiratory rate.

A 3-year-old is admitted to the pediatric unit with pneumonia. He has a productive cough and appears to have difficulty breathing. The parents tell the nurse that the toddler hasn't been eating or drinking much and has been very inactive. Which interventions would be included in the care plan to improve airway clearance? Select all that apply: a) Perform postural drainage. b) Perform chest physiotherapy as ordered. c) Encourage coughing and deep breathing. d) Maintain humidification with a cool mist humidifier. e) Restrict fluid intake. f) Keep the head of the bed flat.

a) Perform postural drainage. b) Perform chest physiotherapy as ordered. c) Encourage coughing and deep breathing. d) Maintain humidification with a cool mist humidifier. Chest physiotherapy and postural drainage work together to break up congestion and then drain secretions. Coughing and deep breathing are also effective to remove congestion. A cool mist humidifier helps loosen thick mucus and relax airway passages. Fluids should be encouraged — not restricted. The child should be placed in semi-Fowler or high Fowler position to facilitate breathing and promote optimal lung expansion.

A child is diagnosed as having right chronic otitis media. After the child returns from surgery for myringotomy and placement of ear tubes, which intervention is appropriate? a) Position the child on the right side. b) Apply gauze dressings. c) Position the child on the left side. d) Apply warm compresses to both ears.

a) Position the child on the right side. The child should be positioned on the right side to facilitate drainage. The left side isn't an area of concern for drainage. Gauze dressings aren't necessary after surgery. Some physicians may prefer a loose cotton wick. Warm compresses may help to facilitate drainage only when used on the affected ear.

A client is brought to the emergency department following an automobile accident. A diagnosis reveals right-sided pneumothorax requiring insertion of a chest tube. What is the appropriate action of the nurse? a) Position the client laying slightly rotated to the left, with the right arm behind the head to expose the axillary area. b) Position the client in a supine position and fully expose the precordium. c) Elevate the head of the bed to a high Fowler's position and remove the client's pillow. d) Assist the client to the prone position with the head turned to the right.

a) Position the client laying slightly rotated to the left, with the right arm behind the head to expose the axillary area. The preferred position for chest tube drain insertion is on the bed, slightly rotated, with the arm on the side of the lesion behind the client's head to expose the axillary area. Alternative positions are sitting upright leaning over an adjacent table with a pillow or in the lateral decubitus position. The other positions are inappropriate for chest tube insertion

The nurse should include which nursing interventions when developing a plan of care for a client who has just undergone a total laryngectomy? Select all that apply. a) Provide meticulous mouth care every 4 hours. b) Keep the tracheostomy cuff fully inflated. c) Encourage oral feedings as soon as possible. d) Keep the client flat in bed. e) Develop an alternative communication method.

a) Provide meticulous mouth care every 4 hours. e) Develop an alternative communication method. A client with a laryngectomy can't speak yet still needs to communicate. Therefore, the nurse should plan to develop an alternative communication method. The client also needs meticulous oral care. After a laryngectomy, edema interferes with the ability to swallow and necessitates tube (enteral) feedings. To prevent injury to the tracheal mucosa, the nurse should deflate the tracheostomy cuff or use the minimal leak technique. To decrease edema, the nurse should place the client in semi-Fowler's position.

Which effect can thoracic kyphoscoliosis have on lung function? a) Restricts lung expansion b) Improves lung expansion c) Reduces alveolar compression during expiration d) Obstructs lung deflation

a) Restricts lung expansion Thoracic kyphoscoliosis causes lung compression, restricts lung expansion, and results in more rapid and shallow respiration.

A client presents with shortness of breath and absent breath sounds on the right side, from the apex to the base. Which condition would best explain this? a) Spontaneous pneumothorax b) Chronic bronchitis c) Pneumonia d) Acute asthma

a) Spontaneous pneumothorax Spontaneous pneumothorax occurs when the client's lung collapses, causing an acute decrease in the amount of functional lung used in oxygenation, resulting in shortness of breath with absent breath sounds. A client with an asthma attack would present with wheezing breath sounds, and bronchitis would be indicated by auscultating rhonchi. Bronchial breath sounds over the area of consolidation would indicate pneumonia.

A client with newly diagnosed asthma is receiving albuterol. When teaching the client about this drug, which signs and symptoms should the nurse explain may occur? Select all that apply. a) Tremors b) Nasal congestion. c) Lethargy d) Restlessness e) Hyperkalemia

a) Tremors d) Restlessness Because of the stimulation of the adrenergic receptors, albuterol may cause restlessness and tremors. The inhaled form of the drug may cause dryness and irritation of the nose and throat, not nasal congestion; insomnia, not lethargy; and hypokalemia (with high doses), not hyperkalemia. Other adverse effects of albuterol along with tremors and restlessness include dizziness, headache, tachycardia, palpitations, hypertension, heartburn, nausea, vomiting, and muscle cramps

The nurse is teaching a client with chronic bronchitis about breathing exercises. Which of the following should the nurse include in the teaching? a) Use diaphragmatic breathing. b) Make inhalation longer than exhalation. c) Exhale through an open mouth. d) Use chest breathing.

a) Use diaphragmatic breathing. In chronic bronchitis the diaphragm is flat and weak. Diaphragmatic breathing helps strengthen the diaphragm and maximizes ventilation. Exhalation should be longer than inhalation to prevent collapse of the bronchioles. The client with chronic bronchitis should exhale through pursed lips to prolong exhalation, keep the bronchioles from collapsing, and prevent air trapping. Diaphragmatic breathing — not chest breathing — increases lung expansion.

Which action by the parents decreases allergens in the home? a) Using foam rubber pillows and synthetic blankets b) Dusting and cleaning the house thoroughly twice a month c) Covering floors with carpeting d) Designating the basement as the play area

a) Using foam rubber pillows and synthetic blankets Bedding should be free from allergens and have nonallergenic covers. Unnecessary rugs should be removed, and floors should be bare and mopped a few times per week to reduce dust. Basements or cellars should be avoided to lessen the child's exposure to molds and mildew. Dusting and cleaning should occur daily or at least weekly.

At discharge, which parental care outcome should be anticipated for a child with bronchopulmonary dysplasia? a) Verbalizes the causes, risks, therapy options, and nursing care b) Only makes safe decisions with professional assistance c) Participates in routine, but not complex, caretaking activities d) Reports increased levels of stress

a) Verbalizes the causes, risks, therapy options, and nursing care The parents should understand the causes, risks, therapy options, and care of their infant by the time of discharge. Asking the parents to verbalize this information is the only way to assess their understanding. The parents should report decreased levels of stress, be capable of making decisions independently, and participate in routine and complex care.

A 2-month-old infant is given a preliminary diagnosis of bronchiolitis. Which symptom would the nurse expect to find? a) Wheezing on auscultation b) No signs of an upper respiratory infection c) Increased appetite d) Bradycardia

a) Wheezing on auscultation In bronchiolitis, the bronchioles become narrowed and edematous, which can cause wheezing. These infants typically have a 2- to 3-day history of an upper respiratory infection and feeding difficulties with loss of appetite due to nasal congestion and increased work of breathing. This combination leads to respiratory distress with tachypnea and tachycardia. (less)

Which X-ray finding would a nurse expect for a child with asthma? a) Pneumothoraces b) Atelectasis c) Infiltrates d) Hemothorax

b) Atelectasis Hyperexpansion, atelectasis, and a flattened diaphragm are typical X-ray findings for a child with asthma. Air becomes trapped behind the narrowed airways and the residual capacity rises, leading to hyperinflation. Hypoxemia results from areas of the lung not being well perfused. A hemothorax isn't a finding related to asthma. Infiltrates and pneumothoraces are uncommon.

After undergoing a left thoracotomy, a client has a chest tube in place. When caring for this client, the nurse should: a) encourage coughing and deep breathing. b) milk the chest tube every 2 hours. c) report fluctuations in the water-seal chamber. d) clamp the chest tube once every shift.

a) encourage coughing and deep breathing. When caring for a client who's recovering from a thoracotomy, the nurse should encourage coughing and deep breathing to prevent pneumonia. Fluctuations in the water-seal chamber are normal. Clamping the chest tube could cause a tension pneumothorax. Chest tube milking is controversial and should be done only to remove blood clots that obstruct the flow of drainage.

A client is diagnosed with a severe airway obstruction. During data collection, which findings does the nurse expects to find? Select all that apply. a) Good air exchange b) Cyanosis c) Clutching the neck d) Inability to speak e) Forceful cough

b) Cyanosis c) Clutching the neck d) Inability to speak A severe airway obstruction commonly manifests as an inability to speak, anxiety, clutching the neck (universal sign), and cyanosis. Forceful cough and good air exchange are manifested with mild airway obstruction

A client with colorectal carcinoma is devastated after learning that the cancer has spread to his liver and lungs and that he has only a 5% chance of surviving for 5 years. Which comment by the nurse would best help the client cope with this news? a) "I've seen clients in your situation who have lived almost 20 years." b) "It must be hard to hear that prognosis. Would it help you to talk to me or the chaplain?" c) "This might be a good time to think about an advance directive in case you run into problems while you're here." d) "Those are just numbers. You have to live each day fully and not worry about dying."

b) "It must be hard to hear that prognosis. Would it help you to talk to me or the chaplain?" This response is most therapeutic because it encourages the client to express feelings and concerns. Options 1 and 4 offer false hope and reflect the nurse's empirical observations, not statistics. Option 3 is inappropriate because an informed person who isn't a member of the health care team should discuss (at the client's request) which level of care the client wishes to receive in case of an emergency.

A nursing student detects bilateral crackles when auscultating a client's lungs. When asked by the nursing instructor, which statement does the student provide about crackles that is true? a) "They are creaking and grating and are usually heard over the problem area on both inspiration and expiration." b) "They are usually heard on inspiration and don't clear with a cough." c) "They are hissing or musical and are usually heard on inspiration and expiration; if severe, they may be heard without a stethoscope." d) "They are usually heard on expiration and may clear with a cough."

b) "They are usually heard on inspiration and don't clear with a cough." Crackles usually are heard on inspiration and do not clear with a cough. They are audible if small airways containing fluid open suddenly. Rhonchi usually are heard on expiration and may clear with a cough. They result from air flowing through narrow, fluid-filled passages. Wheezes are hissing or musical sounds heard on inspiration and expiration; if severe, they are audible without a stethoscope. Wheezes occur when air passes through a narrowed airway such as in asthma. A pleural friction rub causes a creaking, grating noise resembling two pieces of leather rubbing together; it is heard on inspiration and expiration over the problem area.

A client with asthma is receiving a theophylline preparation to promote bronchodilation. Because of the risk of drug toxicity, the nurse must monitor the client's serum theophylline level closely. When reviewing this client's chart, which range will the nurse identify as the therapeutic theophylline concentration? a) 6 to 9 mcg/ml b) 10 to 20 mcg/ml c) 3 to 5 mcg/ml d) 1 to 2 mcg/ml

b) 10 to 20 mcg/ml The therapeutic serum theophylline concentration ranges from 10 to 20 mcg/ml. Values below 10 mcg/ml aren't therapeutic.

A client is receiving oxygen via a nasal cannula at 2 L/minute. What percentage of oxygen concentration is coming through the cannula? a) 30% to 40% b) 23% to 30% c) 50% to 75% d) 40% to 60%

b) 23% to 30% The percentage of oxygen concentration as it passes out of the nasal cannula at 2 L/ minute is 23% to 30%. The oxygen concentration via cannula at 3 to 5 L/minute is 30% to 40%. A simple mask at 6 to 8 L/minute delivers 40% to 60% of oxygen. A partial rebreather mask at 8 to 11 L/minute delivers 50% to 75% of oxygen.

After diagnosing a client with pulmonary tuberculosis, the physician tells family members that they must receive isoniazid as prophylaxis against tuberculosis. The client's teenage daughter asks the nurse how long the drug must be taken. What appropriate timeline does the nurse provide for the duration of prophylactic isoniazid therapy? a) 2 to 4 months b) 6 to 12 months c) 3 to 5 days d) 1 to 3 weeks

b) 6 to 12 months Prophylactic isoniazid therapy must continue for 6 to 12 months at a daily dosage of 300 mg. Taking the drug for less than 6 months may not provide adequate protection against tuberculosis.

A nurse is assigned to care for a client with a chest tube and observes that there's constant bubbling in the water seal chamber of the closed drainage system. Which explanation best describes this observation? a) Constant bubbling indicates that the suction rate is too high b) Constant bubbling indicates that there's a loose connection c) Constant bubbling indicates that the suction rate is too low d) Constant bubbling indicates that the tube is working correctly

b) Constant bubbling indicates that there's a loose connection Constant bubbling in the water seal chamber indicates that there's a leak or loose connection between the client and the water seal chamber. The amount of suction affects the suction control chamber, not the water seal chamber.

Children with chronic otitis media may require surgery for a myringotomy and ear tube placement. Which management strategy explains the purpose of the ear tubes? a) To administer antibiotics b) To drain fluid c) To increase pressure d) To flush the middle ear

b) To drain fluid Ear tubes allow normal fluid to drain (not flush) from the middle ear. They also allow ventilation. The purpose isn't to administer medication. The tubes also allow pressure to equalize in the middle ear.

A 19-year-old client with cystic fibrosis is admitted to the hospital in acute respiratory distress. The client's mother tells a nurse that the client has been unable to get out of bed for the past month. While assessing the client, the nurse notes a stage II pressure ulcer on the client's sacrum. Which action is most important to include in the client's plan of care? a) Turn and reposition the client every 2 hours, monitor the wound, and document findings. b) Accurately document the appearance, size, location, and odor of the wound, and consult a wound care nurse. c) Keep the wound clean and dry, and continue to monitor it. d) Cover the lesion with a sterile gauze pad, and document findings.

b) Accurately document the appearance, size, location, and odor of the wound, and consult a wound care nurse. A client who is on prolonged bed rest is at risk for developing pressure ulcers. The nurse assessing the client must accurately document the appearance, size, location, and odor of the wound. Then the nurse should consult the wound care nurse and notify the physician to establish a treatment plan. The client should also be turned and repositioned every 2 hours, but providing wound care for this client is most important. A stage II pressure ulcer requires intervention; the nurse shouldn't continue to monitor the wound or cover it without establishing a treatment plan.

A client with a pulmonary embolism typically has chest pain and apprehension. Which of the following would be the best treatment method? a) Providing emotional support b) Administering analgesics c) Using guided imagery d) Positioning the client on the left side

b) Administering analgesics After the pulmonary embolism has been diagnosed and the amount of hypoxia determined, chest pain and the accompanying apprehension can be treated with analgesics. The nurse must monitor respiratory status frequently. Guided imagery and providing emotional support can be used as alternatives. Positioning the client on the left side when a pulmonary embolism is suspected may prevent a clot that has extended through the capillaries and into the pulmonary veins from breaking off and trav-eling through the heart into the arterial circulation, leading to a massive stroke.

Hemoptysis may be present in the client with a pulmonary embolism because of which reason? a) Involvement of major blood vessels in the occluded area b) Alveolar damage in the infarcted area c) Loss of lung tissue d) Loss of lung parenchyma

b) Alveolar damage in the infarcted area The infarcted area produces alveolar damage that can lead to the production of bloody sputum, sometimes in large amounts. There's a loss of lung parenchyma and subsequent scar tissue formation and blood vessels may be involved, but these don't cause hemoptysis. (less)

Hemoptysis may be present in the client with a pulmonary embolism because of which reason? a) Involvement of major blood vessels in the occluded area b) Alveolar damage in the infarcted area c) Loss of lung parenchyma d) Loss of lung tissue

b) Alveolar damage in the infarcted area The infarcted area produces alveolar damage that can lead to the production of bloody sputum, sometimes in large amounts. There's a loss of lung parenchyma and subsequent scar tissue formation and blood vessels may be involved, but these don't cause hemoptysis.

A client has a sucking stab wound to the chest. Which action should the nurse take first? a) Prepare a chest tube insertion tray. b) Apply a dressing over the wound and tape it on three sides. c) Prepare to start an I.V. line. d) Draw blood for a hematocrit and hemoglobin level.

b) Apply a dressing over the wound and tape it on three sides. The nurse immediately should apply a dressing over the stab wound and tape it on three sides to allow air to escape and to prevent tension pneumothorax (which is more life-threatening than an open chest wound). Only after covering and taping the wound should the nurse draw blood for laboratory tests, assist with chest tube insertion, and start an I.V. line.

A nurse monitoring a client with a chest tube connected to a water seal drainage device notices constant bubbling in the water seal chamber. What is the appropriate action by the nurse? a) Document the findings and continue to monitor. b) Assess the chest tube connections and drainage device for an air leak. c) Check to make sure the suction is working correctly. d) Notify the health care provider.

b) Assess the chest tube connections and drainage device for an air leak Constant bubbling in the water seal indicates an air leak. To first identify sources of the air leak, the nurse should check and tighten connections and test the tubing for leaks; if the tube is leaking, replace the unit. If the leak is at the insertion site, carefully remove the chest tube dressing and inspect the site to make sure the catheter eyelets have not pulled out beyond the chest wall and replace the dressing. Notify the health care provider of any new, increased, or unexpected air leaks that are not corrected by the above actions. Follow-up with documentation of the magnitude of a client air leak using the air leak meter and appropriate nursing interventions for correction

While assisting a client with a chronic obstructive pulmonary disease (COPD) to ambulate in the hallway, the nurse observes that the client becomes short of breath after walking 100 ft. What is the best action of the nurse? a) Notify the health care provider. b) Assess the client's SpO2 status. c) Encourage the client to continue with ambulation. d) Administer low-flow supplemental O2.

b) Assess the client's SpO2 status. Exercise is an important component to pulmonary rehabilitation for clients with COPD. Clients should be encouraged to exercise despite symptoms of dyspnea, as long as their oxygen saturation levels remain above 90%. The nurse should assess the client's oxygen level before taking any further action such as administering oxygen, allowing ambulation to continue or notifying the health care provider. A drop in SpO2 to below 90% indicates that the client is hypoxemic and needs supplemental oxygen when exercising. At that time the nurse can contact the health care provider for further orders.

A child with thoracic water-seal drainage is on the elevator. The transport aide has placed the drainage system on the stretcher. What action should the nurse on the elevator take first? a) Immediately take the child's respiratory and pulse rates. b) Assist the aide in placing the drainage system lower than the child's chest. c) Clamp the drainage tubing with a hemostat. d) Report the incident to the registered nurse when she returns to the unit.

b) Assist the aide in placing the drainage system lower than the child's chest. The drainage device must be kept below the level of the chest to maintain straight gravity drainage. Placing it on the stretcher may cause a backflow of drainage into the thoracic cavity, which could collapse the partially expanded lung. Reporting the incident is indicated, but the immediate safety of the child takes priority. Clamping the tubing would place the child at risk for a tension pneumothorax. After the drainage system has been properly repositioned, the child's respiratory and pulse rates may be taken

A client must take streptomycin for tuberculosis. Before therapy begins, the nurse should instruct the client to notify the physician if which health concern occurs? a) Impaired color discrimination b) Decreased hearing acuity c) Increased appetite d) Increased urinary frequency

b) Decreased hearing acuity Decreased hearing acuity indicates ototoxicity, a serious adverse effect of streptomycin therapy. The client should notify the physician immediately if it occurs so that streptomycin can be discontinued and an alternative drug can be prescribed. The other options aren't associated with streptomycin. Impaired color discrimination indicates color blindness; increased urinary frequency and increased appetite accompany diabetes mellitus.

A 66-year-old client has marked dyspnea at rest, is thin, and uses accessory muscles to breathe. He's tachypneic, with a prolonged expiratory phase. He has no cough. He leans forward with his arms braced on his knees to support his chest and shoulders for breathing. This client has symptoms of which respiratory disorder? a) Asthma b) Emphysema c) Acute respiratory distress syndrome (ARDS) d) Chronic obstructive bronchitis

b) Emphysema These are classic signs and symptoms of a client with emphysema. Clients with asthma are acutely short of breath during an attack and appear very frightened. Clients with bronchitis are bloated and cyanotic in appearance, and clients with ARDS are acutely short of breath and require emergency care.

A nurse is caring for a client who has a tracheostomy and a temperature of 103° F (39.4° C). Which intervention, if implemented by the nurse, may most likely lower the client's arterial blood oxygen saturation? a) Incentive spirometry b) Endotracheal suctioning c) Encouragement of coughing d) Use of cooling blanket

b) Endotracheal suctioning Endotracheal suctioning removes secretions as well as gases from the airway and lowers the arterial oxygen saturation (SaO2) level. Coughing and incentive spirometry improve oxygenation and should raise or maintain oxygen saturation. Because of superficial vasoconstriction, using a cooling blanket can lower peripheral oxygen saturation readings, but SaO2 levels wouldn't be affected.

A 69-year-old client develops acute shortness of breath and progressive hypoxia requiring mechanical ventilation after repair of a fractured right femur. The hypoxia was probably caused by which condition? a) Atelectasis b) Fat embolism c) Asthma attack d) Bronchitis

b) Fat embolism Long bone fractures are correlated with fat emboli, which cause shortness of breath and hypoxia. It's unlikely the client has developed asthma or bronchitis without a previous history. He could develop atelectasis, but it typically doesn't produce progressive hypoxia

Which reaction is usually exhibited first by the family of an infant who has died from sudden infant death syndrome (SIDS)? a) Requests for the infant's belongings b) Feelings of blame or guilt c) Acceptance of the diagnosis d) Questions regarding the etiology of the diagnosis

b) Feelings of blame or guilt During the first few moments, the parents usually are in shock and have overwhelming feelings of blame or guilt. Acceptance of the diagnosis and questions regarding the etiology may not occur until the parents have had time to see the child. The infant's belongings are usually packaged for the family to take home, but some parents may see this as a painful reminder of their decreased child.

Which nursing intervention is appropriate to correct dehydration for a 2-year-old child with asthma? a) Give cold juice or ice pops. b) Give warm liquids. c) Provide three meals and three snacks. d) Provide small, frequent meals and snacks.

b) Give warm liquids. Liquids are best tolerated if they're warm. Cold liquids may cause bronchospasm and should be avoided. Dehydration should be corrected slowly. Overhydration may increase interstitial pulmonary fluid and exacerbate small airway obstruction. Small, frequent meals should be provided to avoid abdominal distention that may interfere with diaphragm excursion, but these won't correct the dehydration.

Which nursing diagnosis is the priority for an infant with bronchopulmonary dysplasia? a) Effective breast-feeding b) Impaired gas exchange c) Imbalanced nutrition: Less than body requirements d) Risk for imbalanced fluid volume

b) Impaired gas exchange The infant will have Impaired gas exchange related to retention of carbon dioxide and borderline oxygenation secondary to fibrosis of the lungs. Although the infant may require increased caloric intake and may have excess fluid volume, the other nursing diagnoses aren't the priority.

A client with pneumococcal pneumonia is admitted to an acute care facility. The client in the next room is being treated for mycoplasmal pneumonia. Despite the different causes of the various types of pneumonia, the nurse recognizes that all of them share which feature? a) Responsiveness to penicillin b) Inflamed lung tissue c) Sudden onset d) Elevated white blood cell (WBC) count

b) Inflamed lung tissue The common feature of all types of pneumonia is an inflammatory pulmonary response to the offending organism or agent. Although most types of pneumonia have a sudden onset, a few (such as anaerobic bacterial pneumonia and mycoplasmal pneumonia) have an insidious onset. Antibiotic therapy is the primary treatment for most types of pneumonia; however, the antibiotic must be specific for the causative agent, which may not be responsive to penicillin. A few types of pneumonia, such as viral pneumonia, aren't treated with antibiotics. Although pneumonia usually causes an elevated WBC count, some types, such as mycoplasmal pneumonia, don't.

Which pathophysiological mechanism will be expected to develop as a secondary response to pneumonia after development of the primary causative organism? a) Effusion b) Inflammation c) Bronchiectasis d) Atelectasis

b) Inflammation The common feature of all types of pneumonia is an inflammatory pulmonary response to the offending organism or agent. Atelectasis and bronchiectasis indicate a collapse of a portion of the airway that doesn't occur in pneumonia. An effusion is an accumulation of excess pleural fluid in the pleural space, which may be a secondary response to pneumonia.

While examining a child with acute epiglottitis, the nurse should have which item available? a) Tongue depressors b) Intubation equipment c) Cool mist tent d) Viral culture medium

b) Intubation equipment Emergency intubation equipment should be at the bedside to secure the airway if examination precipitates further or complete obstruction occurs. Tongue depressors are contraindicated and may cause the epiglottis to spasm. Cool mist tents and viral culture medium are recommended for the treatment and diagnosis of croup.

A client is prescribed rifampin, 600 mg by mouth daily. The nurse educator is discussing the use of this medication with the unit's nursing staff. Which statements about rifampin would the educator include in the discussion? Select all that apply. a) It acts primarily against resting bacteria. b) It is tuberculocidal, destroying the offending bacteria. c) It is usually given alone. d) Its exact mechanism of action is unknown. e) It is one of the first-line drugs for treating tuberculosis.

b) It is tuberculocidal, destroying the offending bacteria. e) It is one of the first-line drugs for treating tuberculosis. Rifampin is one of the first-line drugs for treating tuberculosis. It may be tuberculocidal or tuberculostatic against the Mycobacterium tuberculosis bacteria. It is usually given with other antitubercular drugs to prevent or delay resistance. The drug impairs ribonucleic acid synthesis of bacteria. It acts against active, not resting, bacteria.

At 8 a.m., the nurse assesses a client who's scheduled for surgery at 10 a.m. During the assessment, the nurse observes dyspnea, a nonproductive cough, and back pain. What should the nurse do next? a) Sign the preoperative checklist for this client b) Make sure that the physician is immediately notified of these findings c) Check the serum electrolyte levels and complete blood count (CBC) d) Check to see that the chest X-ray was done yesterday, as ordered

b) Make sure that the physician is immediately notified of these findings The nurse should make sure that the physician is immediately notified of the findings because dyspnea, a nonproductive cough, and back pain may signal a change in the client's respiratory status. The nurse should check any ordered tests (such as chest X-ray, serum electrolyte levels, and CBC) after notifying the physician because they may help explain the change in the client's condition. The nurse should sign the preoperative checklist after notifying the physician of the client's condition and learning the physician's decision on whether to proceed with surgery.

A client with chronic obstructive pulmonary disease (COPD) is intubated and placed on continuous mechanical ventilation. Which equipment is most important for the nurse to keep at this client's bedside? a) Tracheostomy cleaning kit b) Manual resuscitation bag c) Water-seal chest drainage set-up d) Oxygen analyzer

b) Manual resuscitation bag The client with COPD depends on mechanical ventilation for adequate tissue oxygenation. The nurse must keep a manual resuscitation bag at the bedside to ventilate and oxygenate the client in case the mechanical ventilator malfunctions. Because the client doesn't have chest tubes or a tracheostomy, keeping a water-seal chest drainage set-up or a tracheostomy cleaning kit at the bedside isn't necessary. Although the nurse may keep an oxygen analyzer (pulse oximeter) on hand to evaluate the effectiveness of ventilation, this equipment is less important than the manual resuscitation bag.

A nurse is caring for a 17-year-old female with cystic fibrosis who has been admitted to the hospital for treatment of a recurrent lung infection. The adolescent has many questions about her future and the consequences of her disease. Which statements about the course of cystic fibrosis are accurate? Select all that apply: a) By age 20, the client should be able to decrease the frequency of respiratory treatment. b) Normal sexual relationships can be expected. c) The adolescent is at risk for developing diabetes. d) Pregnancy and childbearing aren't affected. e) Breast development is frequently delayed. f) Only males carry the gene for the disease.

b) Normal sexual relationships can be expected. c) The adolescent is at risk for developing diabetes. e) Breast development is frequently delayed. Cystic fibrosis delays growth and the onset of puberty. Children with cystic fibrosis tend to be smaller-than-average size and develop secondary sexual characteristics later in life. In addition, clients with cystic fibrosis are at risk for developing diabetes mellitus because the pancreatic duct becomes obstructed as pancreatic tissue is damaged. Adolescents with cystic fibrosis can expect to have normal sexual relationships, but fertility may be affected because of changes in mucous membranes. Both males and females carry the gene for cystic fibrosis. Pulmonary disease commonly progresses as the adolescent ages, requiring additional respiratory treatment, not less. (less)

A client is receiving supplemental oxygen. When determining the effectiveness of oxygen therapy, which arterial blood gas value is most important? a) pH b) Partial pressure of arterial oxygen (PaO2) c) Bicarbonate (HCO3-) d) Partial pressure of arterial carbon dioxide (PaCO2)

b) Partial pressure of arterial oxygen (PaO2) The most significant and direct indicator of the effectiveness of oxygen therapy is the PaO2 value. Based on the PaO2 value, the nurse may adjust the type of oxygen delivery (cannula, Venturi mask, or mechanical ventilator), flow rate, and oxygen percentage. The other options reflect the client's ventilation status, not oxygenation.

Bronchopulmonary dysplasia can be classified into four categories. Which characteristic is noted during the early, or first, stage of the disease? a) Hyperexpansion on chest X-ray b) Resemblance to respiratory distress syndrome c) Interstitial fibrosis d) Signs of emphysema

b) Resemblance to respiratory distress syndrome Stage I can be characterized by early interstitial changes and resembles respiratory distress syndrome. Stage III shows signs of the beginning of chronic disease with interstitial edema, signs of emphysema, and pulmonary hypertension. Stage IV shows interstitial fibrosis and hyperexpansion on chest X-ray.

It's highly recommended that clients with asthma, chronic bronchitis, and emphysema have Pneumovax and flu vaccinations for which reason? a) These vaccines produce bronchodilation and improve oxygenation b) Respiratory infections can cause severe hypoxia and possibly death in these clients c) All clients are recommended to have these vaccines d) These vaccines help reduce the tachypnea these clients experience

b) Respiratory infections can cause severe hypoxia and possibly death in these clients It's highly recommended that clients with respiratory disorders be given vaccines to protect against respiratory infection. Infections can cause respiratory failure, and these clients may need to be intubated and mechanically ventilated. The vaccines have no effect on respiratory rate or bronchodilation.

Which organism most commonly causes community-acquired pneumonia in adults? a) Haemophilus influenzae b) Streptococcus pneumoniae c) Klebsiella pneumoniae d) Staphylococcus aureus

b) Streptococcus pneumoniae Pneumococcal or streptococcal pneumonia, caused by S. pneumoniae, is the most common cause of community-acquired pneumonia. H. influenzae is the most common cause of infection in children. Klebsiella species is the most common gram-negative organism found in the hospital setting. S. aureus is the most common cause of hospital-acquired pneumonia.

A client with a pulmonary embolism may need an embolectomy. Which statement most accurately describes the procedure? a) Extracting the embolism from the lung by bronchoscopy b) Surgical removal of the embolism in the pulmonary vasculature c) Removal of an embolism in the lower extremity d) Surgical removal of the embolism source in the pelvis

b) Surgical removal of the embolism in the pulmonary vasculature If the pulmonary embolism is large and doesn't respond to treatment, surgical removal may be necessary to restore perfusion to the area of the lung. This is rarely done because of the associated high mortality risk. It's impossible to remove a pulmonary embolism through bronchoscopy because the defect isn't in the bronchial tree. A thrombectomy can be performed at other sources of clots, but when a pulmonary embolism has already occurred, it would have little effect on oxygenation.

The nurse is developing a teaching plan for a client with asthma. Which teaching point has the highest priority? a) Avoid contact with fur-bearing animals. b) Take prescribed medications as scheduled. c) Change filters on heating and air conditioning units frequently. d) Avoid goose down pillows.

b) Take prescribed medications as scheduled. Although all the measures are appropriate for a client with asthma, taking prescribed medications on time is the most important measure in preventing asthma attacks.

A client with emphysema should receive only enough supplemental oxygen to maintain his PaO2 at 60 mm Hg or higher, or he may lose his hypoxic drive. Which statement is correct about hypoxic drive? a) The client breathes only when his carbon dioxide level dips below a certain point b) The client breathes only when his oxygen levels dip below a certain point c) The client doesn't notice he needs to breathe d) The client breathes only when his oxygen levels climbs above a certain point

b) The client breathes only when his oxygen levels dip below a certain point Clients with emphysema breathe when their oxygen levels drop to a certain level; this is known as the hypoxic drive. Clients with emphysema and chronic obstructive pulmonary disease take a breath when they've reached this low oxygen level. They don't take a breath when their levels of carbon dioxide are higher than normal, as do those with healthy respiratory physiology. If too much oxygen is given, the client has little stimulus to take another breath. His carbon dioxide levels climb, he loses consciousness, and respiratory arrest occurs. (less)

After a motor vehicle accident, a client has a tube inserted that begins to drain a large amount of dark red fluid. Which explanation best describes what caused this type of drainage from the chest tube insertion? a) It is normal for the drainage to be dark red b) The client has experienced a hemothorax instead of a pneumothorax c) An artery was nicked when the chest tube was inserted d) The chest tube was inserted improperly

b) The client has experienced a hemothorax instead of a pneumothorax Because of the traumatic cause of injury, the client has a hemothorax, in which blood collection causes the collapse of the lung. The placement of the chest tube will drain the blood from the space and reexpand the lung. There's a slight chance of nicking an intercostal artery during insertion, but it's fairly unlikely if the person placing the chest tube has been trained. The initial chest X-ray would help confirm whether there was blood in the pleural space or just air.

Upon admission to a long-term care facility, a client is administered a Mantoux test. The nurse reads the test in 48 hours and observes a 5-mm induration. What does this indicate to the nurse? a) The client will develop active tuberculosis. b) The client has produced an immune response to the tuberculosis bacteria. c) The client is immune to tuberculosis. d) The client has active tuberculosis.

b) The client has produced an immune response to the tuberculosis bacteria. Skin testing is based on the antigen/antibody response and will show a positive reaction after an individual is exposed to tuberculosis and has formed antibodies to the tuberculosis bacteria. Thus, a positive Mantoux test indicates the production of an immune response. Exposure doesn't confer immunity. A positive test doesn't confirm that a person has or will develop tuberculosis

On entering the room of a client with chronic obstructive pulmonary disease (COPD), the nurse notices that the client is receiving oxygen at 4 L/minute by way of a nasal cannula. The nurse's actions should be based on which statement? a) The flow rate is too low b) The flow rate is too high c) The client shouldn't receive oxygen d) The flow rate is correct

b) The flow rate is too high The administration of oxygen at 1 to 2 L/ minute by way of a nasal cannula is recommended for clients with COPD: therefore, a rate of 4 L/minute is too high. The normal mechanism that stimulates breathing is a rise in blood carbon dioxide. Clients with COPD retain blood carbon dioxide, so their mechanism for stimulating breathing is a low blood oxygen level. High levels of oxygen may cause hypoventilation and apnea. Oxygen delivered at 1 to 2 L/ minute should aid in oxygenation without causing hypoventilation. Oxygen therapy is the only therapy that has been demonstrated to be life-preserving for patients with COPD

Continuous positive airway pressure can be provided through an oxygen mask to improve oxygenation in hypoxic clients by which method? a) The mask provides continuous air that the client can breathe b) The mask provides pressurized oxygen so the client can breathe more easily c) The mask provides pressurized oxygen at the end of expiration to open collapsed alveoli d) The mask provides 100% oxygen to the client

b) The mask provides pressurized oxygen so the client can breathe more easily The mask provides pressurized oxygen continuously through both inspiration and expiration. By providing a client with pressurized oxygen, the client has less resistance to overcome in taking in his next breath, making it easier to breathe. The mask can be set to deliver any amount of oxygen needed. Pressurized oxygen delivered at the end of expiration is positive end-expiratory pressure, not continuous positive airway pressure.

Which phrase would the nursing student expect to describe the volume of air inspired and expired with a normal breath? a) Forced vital capacity b) Tidal volume c) Residual volume d) Total lung capacity

b) Tidal volume Tidal volume refers to the volume of air inspired and expired with a normal breath. Total lung capacity is the maximal amount of air the lungs and respiratory passages can hold after a forced inspiration. Forced vital capacity is the vital capacity performed with a maximally forced expiration. Residual volume is the maximal amount of air left in the lung after a maximal expiration

A client with chronic obstructive pulmonary disease (COPD) is recovering from a myocardial infarction. Because the client is extremely weak and can't produce an effective cough, the nurse should monitor him closely for: a) oxygen toxicity. b) atelectasis. c) pleural effusion. d) pulmonary edema.

b) atelectasis. In a client with COPD, an ineffective cough impedes secretion removal. This, in turn, causes mucus plugging, which leads to localized airway obstruction — a known cause of atelectasis. An ineffective cough doesn't cause pleural effusion (fluid accumulation in the pleural space). Pulmonary edema usually results from left-sided heart failure, not an ineffective cough. Although many noncardiac conditions may cause pulmonary edema, an ineffective cough isn't one of them. Oxygen toxicity results from prolonged administration of high oxygen concentrations, not an ineffective cough

A 47-year-old male client with an unresolved hemothorax is febrile, with chills and diaphoresis. He has a nonproductive cough and chest pain. His chest tube drainage is turbid. A possible explanation for these findings is: a) Pneumocystis carinii pneumonia. b) empyema. c) infected chest tube wound site. d) lobar pneumonia.

b) empyema. Any condition that produces fluid accumulation or sequestration of fluid with infective properties can lead to empyema (an accumulation of pus in a body cavity, especially the pleural space, as a result of bacterial infection). An infected chest tube site, lobar pneumonia, and P. carinii pneumonia can lead to fever, chills, and sweating associated with infection. In this case, turbid drainage indicates that empyema has developed. Pneumonia typically causes a productive cough and an infected chest tube wound, not turbid drainage. (less)

A client with chronic obstructive lung disease tells the nurse that he feels short of breath. The client's respiratory rate is 36 breaths/minute, and the nurse auscultates diffuse wheezes. His arterial oxygen saturation is 84%. The nurse calls the assigned respiratory therapist to administer a prescribed nebulizer treatment. The therapist says, "I have several more percussions to do on the unit where I am now. As soon as I'm done, I'll come assess the client." The nurse's most appropriate action is to: a) stay with the client until the therapist arrives. b) give the nebulizer treatment herself. c) notify the primary physician immediately. d) administer the treatment by metered-dose inhaler.

b) give the nebulizer treatment herself. The client's needs are preeminent, so the nurse should administer the nebulizer treatment immediately. The nurse can deal with the respiratory therapist's lack of response after the client's condition is stabilized. There is no need to involve the physician in personnel issues. Staying with the client is important, but it isn't a substitute for administering the needed bronchodilator. The order is for a nebulizer treatment. The nurse can't change the route without a new order from the physician.

The client is asking the nurse a question regarding the Mantoux test for tuberculosis. The nurse should base her response on the fact that the: a) area of redness is measured in 3 days and determines whether tuberculosis is present. b) skin test doesn't differentiate between active and dormant tuberculosis infection. c) presence of a wheal at the injection site in 2 days indicates active tuberculosis. d) test stimulates a reddened response in some clients and requires a second test in 3 months.

b) skin test doesn't differentiate between active and dormant tuberculosis infection. The Mantoux test doesn't differentiate between active and dormant infections. If a positive reaction occurs, a sputum smear and culture as well as a chest X-ray are necessary to provide more information. Although the area of redness is measured in 3 days, a second test may be needed; neither test indicates that tuberculosis is active. In the Mantoux test, an induration 5 to 9 mm in diameter indicates a borderline reaction; a larger induration indicates a positive reaction. The presence of a wheal within 2 days doesn't indicate active tuberculosis.

A child is diagnosed with chronic otitis media. Which statement made by the mother indicates an understanding of this condition? a) "I must be sure he gets all of his DTaP vaccinations." b) "I'll give him Tylenol once a day every week." c) "I'll tell my relatives they can't smoke around the baby." d) "I need to be sure the baby is dressed warmly."

c) "I'll tell my relatives they can't smoke around the baby." Eliminating tobacco smoke and other allergens from the environment can help prevent otitis media. No extra clothing is needed. Diphtheria, tetanus, and pertussis aren't causes of otitis media. Tylenol should only be given when indicated for fever or pain.

A client has a tracheostomy but doesn't require continuous mechanical ventilation. When weaning the client from the tracheostomy tube, the nurse initially should plug the opening in the tube for: a) 30 to 40 minutes. b) 45 to 60 minutes. c) 5 to 20 minutes. d) 15 to 60 seconds.

c) 5 to 20 minutes. Initially, the nurse should plug the opening in the tracheostomy tube for 5 to 20 minutes, and then gradually lengthen this interval according to the client's respiratory status. A client who doesn't require continuous mechanical ventilation is already breathing without assistance, at least for short periods. Therefore, plugging the opening of the tube for only 15 to 60 seconds wouldn't be long enough to reveal the client's true tolerance to the procedure. Plugging the opening for more than 20 minutes would increase the risk of acute respiratory distress because the client requires an adjustment period to start breathing normally.

A nurse is caring for a client diagnosed with a deep vein thrombosis who develops dyspnea and restlessness. What is the best action of the nurse? a) Document the findings and continue to monitor. b) Assist the client to a more comfortable position. c) Assess the client's oxygen saturation (SaO2) level. d) Notify the health care provider.

c) Assess the client's oxygen saturation (SaO2) level. The client has symptoms consistent with a pulmonary embolism (PE). Assessing the client's oxygenation status is the priority before contacting the health care provider. Many clients begin treatment for PE on the basis of history and the clinical examination, before definitive diagnostic testing has been completed. The key to timely diagnosis is identifying risk factors for the development of a PE. Documenting the findings and monitoring is not appropriate care. Changing the client's position will not help with diagnosing or treating a PE

Which respiratory disorder is most common in the first 24 to 48 hours after surgery? a) Pneumonia b) Bronchitis c) Atelectasis d) Pneumothorax

c) Atelectasis Atelectasis develops when there's interference with the normal negative pressure that promotes lung expansion. Clients in the postoperative phase typically guard their breathing because of pain and positioning, which causes hypoxia. It's uncommon for any of the other respiratory disorders to develop after surgery

After undergoing a right lower lobectomy for treatment of lung cancer, a 75-year-old client returns to his room with a chest tube in place. Several hours later a nurse finds the client out of bed barely able to speak, with the chest tube removed. Which action should the nurse take immediately? a) Cover the insertion site with a sterile gauze dressing, assist the client back to bed, and monitor his vital signs. b) Assist the client back to bed, assess his respiratory status, and remain with him. c) Cover the insertion site with an occlusive dressing, call for assistance, and remain with the client. d) Assist the client back to bed, assess his vital signs, and notify a physician.

c) Cover the insertion site with an occlusive dressing, call for assistance, and remain with the client. A chest tube facilitates lung re-expansion after surgical intervention in the pleural cavity. If the chest tube becomes dislodged before healing takes place, air enters the pleural cavity, causing the lung to collapse. In this case, the nurse should immediately call for assistance and cover the insertion site with an occlusive dressing. A gauze dressing shouldn't be used because gauze isn't occlusive. The physician should then be notified so that he can reinsert the tube. The nurse should remain with the client and continue to monitor vital signs until the chest tube has been reinserted, and its placement confirmed.

In a client with smoke inhalation, the nurse would expect to hear which breath sound? a) Decreased breath sounds b) Upper airway rhonchi c) Crackles d) Inspiratory and expiratory wheezing

c) Crackles In smoke inhalation, the most frequently heard sounds are crackles throughout the lung fields. Decreased breath sounds or inspiratory and expiratory wheezing is associated with asthma, and rhonchi are heard when there's sputum in the airways.

The oxygen saturation level of a 48-year-old client admitted to the hospital with bronchial pneumonia decreases, and his breathing is shallow. He refuses to perform coughing and deep-breathing exercises, or use an incentive spirometer. Which measures can the nurse take to help improve the client's respiratory status? a) Place the client in semi-Fowler's position and turn him every 2 hours. b) Notify a physician of the assessment findings, and obtain orders to prevent further complications. c) Elevate the head of the bed, and demonstrate and reinforce the importance of incentive spirometry, turning, coughing, and deep breathing. d) Reeducate the client about splinting his chest for turning, coughing, and deep breathing.

c) Elevate the head of the bed, and demonstrate and reinforce the importance of incentive spirometry, turning, coughing, and deep breathing. Elevating the head of the bed facilitates easier breathing. Education and encouragement of breathing techniques and incentive spirometry use may increase compliance and improve gas exchange. Performance should improve with demonstration. Reeducating the client about splinting his chest would be ineffective in improving his breathing. Turning the client every 2 hours helps prevent skin breakdown but won't facilitate breathing. The nurse should notify the physician if the client's condition deteriorates.

While receiving heparin to treat a pulmonary embolus, a client passes bright red urine. What should the nurse do first? a) Start an I.V. infusion of dextrose 5% in water. b) Decrease the heparin infusion rate. c) Expect protamine sulfate to be ordered. d) Monitor the partial thromboplastin time (PTT).

c) Expect protamine sulfate to be ordered. Frank hematuria indicates excessive anticoagulation and bleeding — and heparin overdose. The nurse should discontinue the heparin infusion immediately and prepare to administer protamine sulfate, the antidote for heparin. Decreasing the heparin infusion rate wouldn't prevent further bleeding. Although the nurse should continue to monitor PTT, this action should occur later. An I.V. solution may be administered, but only after protamine has been given.

A child is admitted with a possible tracheal foreign body. Which findings would most likely indicate a foreign body in the trachea? a) Cough, stridor, and changes in phonation b) Cough, asymmetric breath sounds, and wheeze c) Expiratory wheeze and inspiratory stridor d) Cough, dyspnea, and drooling

c) Expiratory wheeze and inspiratory stridor Expiratory and inspiratory noise indicates that the foreign body is in the trachea. Cough, dyspnea, drooling, and gagging indicate supraglottic obstruction. A cough with stridor and changes in phonation occurs if the foreign body is in the larynx. Asym metric breath sounds indicate that the object may be located in the bronchi.

The nurse is caring for a client with pneumonia. The nurse should expect to observe which signs and symptoms? Select all that apply. a) Bradycardia b) Dry cough c) Fever d) Pericardial friction rub e) Use of accessory muscles during respiration f) Crackles or rhonchi

c) Fever e) Use of accessory muscles during respiration f) Crackles or rhonchi The client with pneumonia may have a fever, use accessory muscles for breathing, and exhibit crackles or rhonchi on auscultation. Other signs and symptoms of pneumonia include fever, malaise, pleuritic pain, pleural friction rub, dyspnea, tachypnea, tachycardia, and a cough that produces rusty green or bloody sputum (in pneumococcal pneumonia) or yellow-green sputum (in bronchopneumonia). A dry cough, bradycardia, and a pericardial friction rub are not manifestations of pneumonia.

A client has started a new drug for hypertension. Thirty minutes after he takes the drug, he develops chest tightness and becomes short of breath and tachypneic. He has a decreased level of consciousness. These signs indicate which condition? a) Rheumatoid arthritis b) Asthma attack c) Hypersensitivity to the medication d) Pulmonary embolism

c) Hypersensitivity to the medication These signs indicate a hypersensitivity to the new medication, leading to anaphylaxis and respiratory failure. An asthma attack is characterized by wheezing. A client with pulmonary embolism typically has chest pain with inspiration and hypoxemia. Rheumatoid arthritis doesn't cause respiratory symptoms

A client is admitted to the emergency department with an acute asthma attack. The physician prescribes ephedrine sulfate, 25 mg subcutaneously (subQ). After administration, how soon should the nurse expect ephedrine take effect? a) In 2 hours b) In 3 minutes c) Immediately d) In 1 hour

c) Immediately Ephedrine sulfate's onset of action is immediate when administered I.V., I.M., or subQ. Its onset of action is 15 to 60 minutes when administered orally or nasally. No noncatecholamine agent has an onset of action longer than 60 minutes.

Bronchopulmonary dysplasia can cause increased fluid in the lungs due to disruption of the alveolar-capillary membrane; the child may begin receiving furosemide (Lasix). Which adverse effect is possible? a) Hyperkalemia b) Hypernatremia c) Irregular heart rhythm d) Hypercalcemia

c) Irregular heart rhythm An irregular heart rhythm and muscle cramps are adverse effects of hypokalemia and hypocalcemia and not hypercalcemia or hyperkalemia. Diuretics cause volume depletion by inhibiting reabsorption of sodium and chloride. Hypocalcemia is related to the urinary excretion of calcium. Hypokalemia can occur with excessive fluid loss or as part of contraction alkalosis

A client with chronic obstructive pulmonary disease (COPD) takes anhydrous theophylline, 200 mg by mouth every 8 hours. During a routine clinic visit, the client asks the nurse how the drug works. What mechanism of action of anhydrous theophylline in treating a nonreversible obstructive airway disease such as COPD will the nurse share with the client? a) It stimulates adenosine receptors, causing bronchodilation. b) It alters diaphragm movement, which increases chest expansion and enhances the lung's capacity for gas exchange. c) It makes the central respiratory center more sensitive to carbon dioxide and stimulates the respiratory drive. d) It inhibits the enzyme phosphodiesterase, which decreases the degradation of cyclic adenosine monophosphate, a bronchodilator.

c) It makes the central respiratory center more sensitive to carbon dioxide and stimulates the respiratory drive. Anhydrous theophylline and other methylxanthine agents make the central respiratory center more sensitive to CO2 and stimulate the respiratory drive. Inhibition of phosphodiesterase is the drug's mechanism of action in treating asthma and other reversible obstructive airway diseases — not COPD. Methylxanthine agents inhibit rather than stimulate adenosine receptors. Although these agents reduce diaphragmatic fatigue in clients with chronic bronchitis or emphysema, they don't alter diaphragm movement to increase chest expansion and enhance gas exchange.

In most cases, bronchiolitis is caused by a viral agent, most commonly respiratory syncytial virus (RSV). The nurse should keep in mind which statement regarding RSV infections? a) It's not particularly contagious. b) It's more prevalent in the late summer and early fall months. c) It's most likely to attack the respiratory tract mucosa. d) It's more commonly seen in children older than age 5.

c) It's most likely to attack the respiratory tract mucosa. RSV attacks the respiratory tract mucosa. The virus is most prevalent in the winter and early spring months. Most children develop the infection between ages 2 and 6 months, but RSV may occur during the first 3 years of life. RSV is a highly contagious respiratory virus

Which additional data should immediately be gathered to determine the status of a client with a respiratory rate of 4 breaths/minute? a) Pulse oximetry value and heart sounds b) Breath sounds and reflexes c) Level of consciousness and a pulse oximetry value d) Arterial blood gas (ABG) levels and breath sounds

c) Level of consciousness and a pulse oximetry value First, the nurse should attempt to rouse the client, because this should increase the client's respiratory rate. Then a spot pulse oximetry check should be done and breath sounds should be checked. The physician should be notified immediately of the findings. He'll probably order an ABG to determine specific carbon dioxide and oxygen levels. Heart sounds and reflexes will be checked after these initial actions are completed

A client hospitalized for treatment of a pulmonary embolism develops respiratory alkalosis. While monitoring the client, which clinical findings that commonly accompany respiratory alkalosis would the nurse expect to assess? a) Nausea or vomiting b) Hallucinations or tinnitus c) Light-headedness or paresthesia d) Abdominal pain or diarrhea

c) Light-headedness or paresthesia The client with respiratory alkalosis may report light-headedness or paresthesia (numbness and tingling in the arms and legs). Nausea, vomiting, abdominal pain, and diarrhea may accompany respiratory acidosis. Hallucinations and tinnitus rarely are associated with respiratory alkalosis or any other acid-base imbalance. (

For a client who has a chest tube connected to a closed water-seal drainage system, the nurse should include which action in the plan of care? a) Stripping the chest tube every hour b) Keeping the collection chamber at chest level c) Measuring and documenting the drainage in the collection chamber d) Maintaining continuous bubbling in the water-seal chamber

c) Measuring and documenting the drainage in the collection chamber The nurse should measure and document the amount of chest tube drainage regularly to detect abnormal drainage patterns, such as those that may occur with a hemorrhage (if excessive) or a blockage (if decreased). Continuous bubbling in the water- seal chamber indicates a leak in the closed chest drainage system, which must be corrected. The nurse should keep the collection chamber below chest level to allow fluids to drain into it. The nurse should not strip chest tubes because doing so may traumatize the tissue or dislodge the tube

Which intervention is most appropriate for a child with cystic fibrosis who is having difficulty clearing secretions? a) Provide a high-calorie, high-protein diet at each meal. b) Provide oxygen by nasal cannula at all times. c) Perform chest physiotherapy four times per day. d) Administer pancreatic enzymes with meals.

c) Perform chest physiotherapy four times per day. Chest physiotherapy should be performed to mobilize secretions so they can be more easily cleared. Pancreatic enzymes should be administered with meals to aid in digestion. Administering oxygen may improve oxygenation but won't help clear secretions. A high-calorie, high-protein diet is important for normal growth and development, but it won't aid in clearing secretions.

The parents of a 10-year-old child who was recently diagnosed with asthma ask if the child can continue to play sports. Which response is most appropriate? a) You should limit activities to quiet play. b) It's okay to play some sports but swimming isn't recommended. c) Physical activity and sports are encouraged as long as the asthma is under control. d) Sports don't cause asthma attacks.

c) Physical activity and sports are encouraged as long as the asthma is under control. Participation in sports is encouraged but should be evaluated on an individual basis as long as the asthma is under control. Exercise-induced asthma is an example of the airway hyperactivity common to asthmatics. Exclusion from sports or activities may hamper peer interaction. Swimming is well tolerated because of the type of breathing involved and the moisture in the air.

When preparing an in-service on adult respiratory distress syndrome (ARDS), which sign or symptom will the nurse educator identify as its hallmark? a) Wheezing and shortness of breath b) Copious amounts of mucus c) Progressive hypoxemia despite supplemental oxygen therapy d) Expectoration of foul sputum

c) Progressive hypoxemia despite supplemental oxygen therapy Progressive hypoxemia despite supplemental oxygen therapy is the hallmark of ARDS. Copious amounts of mucus is characteristic of bronchitis, expectoration of foul sputum indicates bronchiectasis, and wheezing and shortness of breath are symptoms of asthma.

The care plan for an infant with bronchopulmonary dysplasia includes the nursing diagnosis of Impaired gas exchange. Which nursing action would be the most appropriate for the nurse to include? a) Provide enteral feedings. b) Provide appropriate age-related activities. c) Provide chest physiotherapy. d) Promote bonding between parents and child.

c) Provide chest physiotherapy. All of these activities are appropriate to include in the care of a child with bronchopulmonary dysplasia; however, providing chest physiotherapy is the nursing action that addresses the nursing diagnosis of Impaired gas exchange.

Which is the best intervention for parents to take if their child is experiencing an episode of "midnight croup," or acute spasmodic laryngitis? a) Give warm liquids. b) Take the child into the bathroom with a warm, running shower. c) Provide humidified air with cool mist. d) Raise the heat on the thermostat.

c) Provide humidified air with cool mist. High humidity with cool mist, such as from a cool mist humidifier, provides the most (and safest form of) relief. Cool liquids would be best for the child. If unable to take liquid, the child needs emergency care. Raising the heat on the thermostat will result in dry, warm air, which may cause secretions to adhere to the airway wall. A warm, running shower provides a mist that may be helpful to moisten and decrease the viscosity of airway secretions and may also decrease laryngeal spasm.

The arterial blood gas analysis of a child with asthma shows a pH of 7.30, PCO2 of 56 mm Hg, and HCO3- of 25 mEq/L. The nurse determines that the child has which condition? a) Metabolic acidosis b) Respiratory alkalosis c) Respiratory acidosis d) Metabolic alkalosis

c) Respiratory acidosis Respiratory acidosis is an acid-base disturbance characterized by excess CO2 in the blood, indicated by a PCO 2 greater than 45 mm Hg. The pH level is usually below the normal range of 7.36 to 7.45. The HCO3- level is normal in the acute stage and elevated in the chronic stage

A 72-year-old client with end-stage chronic obstructive pulmonary disease (COPD) is admitted to the hospital in acute respiratory distress. He refuses endotracheal intubation but requests less invasive treatment interventions. A nurse notes that the client's oxygen saturation is 82%, his pulse is rapid and thready, and his respirations are shallow. How should the nurse intervene? a) Encourage the client to accept the prescribed treatment regimen. b) Inform the client that his health will improve if he chooses endotracheal intubation. c) Support the client's treatment decision and provide care as prescribed. d) Explain to the client that care interventions won't be effective without endotracheal intubation.

c) Support the client's treatment decision and provide care as prescribed. All health care professionals should respect a client's right to make decisions about his health care. Option 2 offers the client false reassurance that endotracheal intubation will improve his health. A client with end-stage COPD is unlikely to recover; his condition may continue to worsen even with endotracheal intubation and mechanical ventilation. The nurse has an obligation to refrain from interfering with the client's autonomy in making health care choices.

A client is brought to the emergency department in acute respiratory distress. After endotracheal (ET) intubation and initiation of mechanical ventilation, the client is transferred to the intensive care unit. Before suctioning the ET tube, the nurse hyperventilates and hyperoxygenates the client. What is the rationale for these interventions? a) They help prevent pneumothorax. b) They help prevent subcutaneous emphysema. c) They help prevent cardiac arrhythmias. d) They help prevent pulmonary edema.

c) They help prevent cardiac arrhythmias. Suctioning of the ET removes oxygen, lowering the partial pressure of arterial oxygen; this, in turn, may induce a cardiac arrhythmia. Hyperventilating and hyperoxygenating the client before and during (or after) suctioning helps prevent this complication. Subcutaneous emphysema occurs when air from the pleural cavity leaks into subcutaneous tissue; it isn't a complication associated with suctioning. Hyperventilation and hyperoxygenation can't prevent a pneumothorax because this condition itself indicates air in the pleural space. Pulmonary edema is associated with cardiac dysfunction, not ET suctioning.

A client diagnosed with active tuberculosis (TB) would be hospitalized primarily for which reason? a) To determine his compliance b) To evaluate his condition c) To prevent spread of the disease d) To determine the need for antibiotic therapy

c) To prevent spread of the disease The client with active TB is highly contagious until three consecutive sputum cultures are negative, so he's put in respiratory isolation in the hospital. Assessment of his physical condition, need for antibiotic therapy, and determinations of compliance aren't considered primary reasons for hospitalization in this case.

Which symptom would the nurse expect to find most commonly in a child with asthma? a) Rales b) Stridor c) Wheezing d) Rhonchi

c) Wheezing Asthma frequently occurs with wheezing and coughing. Airway inflammation and edema increase mucus production. Other signs include dyspnea, tachycardia, and tachypnea. Stridor is heard in croup. Rhonchi and rales are not as common in asthma as is wheezing.

A 24-year-old client comes into the clinic complaining of right-sided chest pain and shortness of breath. He reports that it started suddenly. The nurse's assessment should include: a) electrocardiogram b) chest X-ray c) auscultation of breath sounds d) echocardiogram

c) auscultation of breath sounds Because he's short of breath, auscultation of the lungs will indicate normal or abnormal breath sounds. He may need a chest X-ray and an electrocardiogram, but they require a physician's order. An echocardiogram also requires a physician's order and may be necessary if a pulmonary embolus is suspected.

A client recovering from an acute asthma attack experiences respiratory alkalosis. The nurse measures a respiratory rate of 46 breaths/minute, a heart rate of 110 beats/minute, a blood pressure of 162/90 mm Hg, and a temperature of 98.6° F (37° C). To help correct respiratory alkalosis, the nurse should: a) insert a nasogastric tube (NG) as ordered. b) administer antibiotics as prescribed. c) instruct the client to breathe into a paper bag. d) administer acetaminophen as prescribed.

c) instruct the client to breathe into a paper bag. A client recovering from an acute asthma attack who experiences respiratory alkalosis should breathe into a paper bag to increase arterial carbon dioxide tension and ease anxiety (which may exacerbate the alkalosis). An NG tube would be indicated for a client with metabolic alkalosis secondary to ingestion of toxic substances; nothing indicates that this has occurred. Fever may cause metabolic (not respiratory) alkalosis and would be treated with acetaminophen. A client with sepsis also may have metabolic alkalosis and probably would receive antibiotics; however, this clinical situation doesn't suggest sepsis

A client with Guillain-Barré syndrome develops respiratory acidosis as a result of reduced alveolar ventilation. Which combination of arterial blood gas (ABG) values confirms respiratory acidosis? a) pH, 7.35; PaCO2 40 mm Hg b) pH, 7.40; PaCO2 35 mm Hg c) pH, 7.25; PaCO2 50 mm Hg d) pH, 5.0; PaCO2 30 mm Hg

c) pH, 7.25; PaCO2 50 mm Hg In respiratory acidosis, ABG analysis reveals an arterial pH below 7.35 and partial pressure of arterial carbon dioxide (PaCO2) above 45 mm Hg. Therefore, the combination of a pH value of 7.25 and a PaCO2 value of 50 mm Hg confirms respiratory acidosis. A pH value of 5.0 with a PaCO2 value of 30 mm Hg indicates respiratory alkalosis. The other options represent normal ABG values and reflect normal gas exchange in the lungs

Following the initial care of a client with asthma and impending anaphylaxis from hypersensitivity to a drug, the nurse should take which step next? a) Lay the client flat on the bed b) Obtain serum electrolyte levels c) Administer beta-adrenergic blockers d) Administer bronchodilators

d) Administer bronchodilators Bronchodilators would help open the client's airway and improve his oxygenation status. Beta-adrenergic blockers aren't indicated in the management of asthma because they may cause bronchospasm. Obtaining laboratory values wouldn't be done on an emergency basis, and having the client lie flat in bed could worsen the client's ability to breathe.

A client undergoes a total laryngectomy and tracheostomy formation. On discharge, the nurse should give which instruction to the client and family? a) "Oral intake of fluids should be limited for 1 week only." b) "Clean the tracheostomy tube with alcohol and water." c) "Limit the amount of protein in the diet." d) "Family members should continue to talk to the client."

d) "Family members should continue to talk to the client." Commonly, family members are reluctant to talk to a client who has had a total laryngectomy and can no longer speak. To promote a supportive environment, the nurse should encourage family members to continue normal communication. The nurse should teach the client to clean the tracheostomy tube with hydrogen peroxide and rinse it with sterile saline solution, to consume oral fluids as desired, and to eat protein-rich foods to promote healing

Following a pulmonary embolism, a client is placed on I.V. heparin. The client asks the nurse about the purpose of the heparin. Which statement by the nurse is the correct explanation of the purpose of heparin therapy? a) "Heparin will dissolve any circulating clots." b) "Heparin will prevent pieces of the clot from breaking off and going to your lung." c) "Heparin will dissolve the clot in your lungs." d) "Heparin will slow the development of any more clots."

d) "Heparin will slow the development of any more clots." Heparin is an anticoagulant and is administered to slow thrombus formation. Fibrinolytic medications dissolve clots. Heparin won't prevent clots from embolizing or dissolve circulating clots.

The nurse is caring for a client with pneumonia. As part of prescribed therapy, the client must use a bedside incentive spirometer to promote maximal deep breathing. The nurse checks to make sure the client is using the spirometer properly. During each waking hour, the client should perform a minimum of how many sustained, voluntary inflation maneuvers? a) 1 to 2 b) 3 to 4 c) 5 to 7 d) 8 to 10

d) 8 to 10 The client should perform at least 8 to 10 sustained, voluntary maximal inflation maneuvers with the incentive spirometer during each waking hour. Performing fewer than 8 maneuvers would reduce the respiratory benefits of this therapy.

A client is in danger of respiratory arrest following the administration of an opioid analgesic. An arterial blood gas analysis is obtained. The nurse would expect the PaCO2 to be which value? a) 40 mm Hg b) 15 mm Hg c) 30 mm Hg d) 80 mm Hg

d) 80 mm Hg A client about to go into respiratory arrest will have inefficient ventilation and will be retaining carbon dioxide. The value expected would be around 80 mm Hg. All other values are lower than normal.

A client was infected with tuberculosis (TB) bacillus 10 years ago but never developed the disease. He's now being treated for cancer. The client begins to develop signs of TB. This is known as which type of infection? a) Superinfection b) Primary infection c) Tertiary infection d) Active infection

d) Active infection Some people carry dormant TB infections that may develop into active disease. In addition, primary sites of infection containing TB bacilli may remain latent for years and then activate when the client's resistance is lowered, as when a client is being treated for cancer. There's no such thing as tertiary infection, and superinfection doesn't apply in this case.

Emergency treatment for a client with impending anaphylaxis secondary to hypersensitivity to a drug should include which action first? a) Take vital signs b) Insert an I.V. catheter c) Obtain a complete blood count (CBC) d) Administer oxygen

d) Administer oxygen Giving oxygen would be the best first action in this case. Vital signs then should be checked and the physician immediately notified. If the client doesn't already have an I.V. catheter, one may be inserted now if anaphylactic shock is developing. Obtaining a CBC would not help the emergency situation.

A woman whose husband was recently diagnosed with active pulmonary tuberculosis (TB) is a tuberculin skin test converter but does not show signs of active tuberculosis. What would the nurse include in the management of this client's care? a) Place her in quarantine until sputum cultures are negative. b) Gather a list of persons with whom she has had recent contact. c) Schedule her for annual tuberculin skin testing. d) Advise her to begin prophylactic therapy with isoniazid (INH).

d) Advise her to begin prophylactic therapy with isoniazid (INH). Individuals who are tuberculin skin test converters should begin a 6-month regimen of an antitubercular drug such as INH, and they should never have another skin test. After an individual has a positive tuberculin skin test, subsequent skin tests will cause severe skin reactions but won't provide new information about the client's TB status. The client doesn't have active TB, so she can't transmit, or spread, the bacteria. Therefore, she shouldn't be quarantined or asked for information about recent contacts

A client is diagnosed with a chronic respiratory disorder. After assessing the client's knowledge of the disorder, the nurse prepares a teaching plan. This teaching plan is likely to include which nursing diagnosis? a) Imbalanced nutrition: More than body requirements b) Impaired swallowing c) Unilateral neglect d) Anxiety

d) Anxiety In a client with a respiratory disorder, anxiety worsens such problems as dyspnea and bronchospasm. A client with a chronic respiratory disorder commonly is fatigued from breathing effort or lack of sleep. This client also may have inadequate nutrition, which makes Imbalanced nutrition: More than body requirements an unlikely nursing diagnosis. Impaired swallowing may occur in a client with an acute respiratory disorder, such as an upper airway obstruction, but not in one with a chronic respiratory disorder. Unilateral neglect may be an appropriate nursing diagnosis when neurologic illness or trauma causes lack of awareness of a body part; however, this doesn't occur in a chronic respiratory disorder

The nurse prepares to perform postural drainage. Which method would the nurse use to determine the best position for facilitating drainage of the lungs? a) Arterial blood gas (ABG) levels b) Chest X-ray c) Inspection d) Auscultation

d) Auscultation Breath sounds should be auscultated before doing postural drainage to determine the areas that need draining. After the areas are identified, the nurse can position the client appropriately. Inspection, chest X-rays, and ABG levels are all parameters that give good information about respiratory function, but they aren't necessary to determine lung areas requiring postural drainage

The nurse is caring for a child admitted with acute exacerbation of asthma and receiving oxygen via nasal cannula at 2 L/minute and who needs magnetic resonance imaging (MRI) without contrast. What is the priority nursing action for this client? a) Obtain consent. b) Identify allergies on the client's chart. c) Instruct the child about what to expect from the procedure. d) Avoid having an oxygen cylinder in the MRI area.

d) Avoid having an oxygen cylinder in the MRI area. The priority nursing action is to ensure that the oxygen cylinder is not taken into the MRI area to avoid injury. Instructing the child on the procedure and consent ensures his or her cooperation. Allergy is not a concern with this procedure because the procedure requires imaging only and no dye.

A client with emphysema will be discharged in a few days. During a discharge teaching session, which things should the nurse instruct this client to avoid? Select all that apply. a) Telephone calls b) People c) Exercise d) Dry-heated areas e) Fumes

d) Dry-heated areas e) Fumes To prevent exacerbation of emphysema, the client should avoid fumes because they can irritate the respiratory tract, which increases inflammation and makes breathing more difficult. Dry-heated areas can aggravate the client's symptoms. A client with emphysema should avoid people with respiratory infections — not all people. Even a client with emphysema must obtain some exercise such as simple breathing exercises. Telephone calls may be safer than visits, especially in the winter, because they prevent exposure to people who may have respiratory infections.

Which intervention is the key to increasing the survival rates of clients with lung cancer? a) Smoking cessation b) High-dose chemotherapy c) Early bronchoscopy d) Early detection

d) Early detection

A nurse is caring for a client following a left pneumonectomy. Which is an appropriate nursing intervention? a) Position the client on the left side. b) Position the client on the right side. c) Position the client supine with a small pillow. d) Elevate the head of the bed 45 degrees.

d) Elevate the head of the bed 45 degrees. Elevation of the head helps gas exchange for clients with a pneumonectomy. Side-lying on the unaffected side may cause mediastinal shift. Supine position will not improve oxygenation. The client should not be positioned on the affected side

A 19-year-old client comes to the emergency department with acute asthma. His respiratory rate is 44 breaths/minute, and he appears in acute respiratory distress. Which action should be taken first? a) Apply a cardiac monitor to the client b) Provide emotional support to the client c) Take a full medical history d) Give a bronchodilator by nebulizer

d) Give a bronchodilator by nebulizer The client having an acute asthma attack needs to increase oxygen delivery to the lung and body. Nebulized bronchodilators open airways and increase the amount of oxygen delivered. It may not be necessary to place the client on a cardiac monitor because he's only 19 years old, unless he has a medical history of cardiac problems. First, resolve the acute phase of the attack; then obtain a full medical history to determine the cause of the attack and how to prevent attacks in the future

Administration of which childhood vaccination assists in decreasing a child's incidence of developing epiglottitis? a) Measles vaccine b) Diphtheria vaccine c) Inactivated polio vaccine (IPV) d) Haemophilus influenzae type B (Hib) vaccine

d) Haemophilus influenzae type B (Hib) vaccine Epiglottitis is caused by the bacterial agent H. influenzae. The American Academy of Pediatrics recommends that, beginning at age 2 months, children receive the Hib conjugate vaccine. A decline in the incidence of epiglottitis has been seen as a result of this vaccination regimen. The diphtheria vaccine, measles vaccine, and IPV are preventive for those diseases, not epiglottitis.

The nurse is caring for a client with acute respiratory insufficiency that requires arterial blood gas analysis (ABG). The client has been taking warfarin for several months. What intervention should be provided after the ABG is drawn? a) Hold pressure to the site for 20-30 minutes. b) Hold pressure to the site for 2-5 minutes. c) Hold pressure to the site for 10-20 minutes. d) Hold pressure to the site for 5-10 minutes.

d) Hold pressure to the site for 5-10 minutes. After a blood sample is drawn from the femoral artery, the nurse should apply continuous pressure to the puncture site for 5-10 minutes to prevent bleeding. Applying pressure for shorter periods wouldn't allow enough time for clotting to occur. Longer periods are not necessary unless the patient has a clotting disorder, which is not mentioned for this patient.

Which of the following would be appropriate for a client with arterial blood gas (ABG) values of pH 7.5, PaCO2 26 mm Hg, O2 saturation 96%, HCO3- 24 mEq/L, and PaO2 94 mm Hg? a) Administer a prescribed decongestant. b) Administer prescribed supplemental oxygen. c) Offer the client fluids frequently. d) Instruct the client to breathe into a paper bag.

d) Instruct the client to breathe into a paper bag. The ABG results reveal respiratory alkalosis. The best intervention to raise the PaCO2 level would be to have the client breathe into a paper bag. All of the other options — such as administering a decongestant, offering fluids frequently, and administering supplemental oxygen — wouldn't raise the lowered PaCO2 level.

A client with advanced acquired immunodeficiency syndrome (AIDS) is diagnosed with active tuberculosis. Which of the following regimens would the nurse expect the physician to prescribe? a) Ethambutol, pyrazinamide, and isoniazid b) Isoniazid and rifampin c) Ethambutol, ciprofloxacin, pyrazinamide, and streptomycin d) Isoniazid, rifampin, ethambutol, and pyrazinamide

d) Isoniazid, rifampin, ethambutol, and pyrazinamide A client with advanced AIDS is less able to fight tuberculosis because of decreased cellular immunity. Usually, a 21-day course of treatment is required to identify the specific sensitivity of the tuberculous organism to antibacterial medications. However, multi-drug-resistant tuberculosis is prevalent among clients with AIDS. To avoid inadequate treatment and prevent development of resistant tuberculosis, the client is prescribed a four-drug regimen until specific sensitivities are established; the medication regimen is then adjusted. This client would receive isoniazid (a tuberculostatic), rifampin (which is tuberculocidal), pyrazinamide (which is highly specific for Mycobacterium tuberculosis), and ethambutol. Streptomycin is a second-line drug that isn't routinely given to treat tuberculosis. Ciprofloxacin usually isn't a first-line drug used to treat resistant tuberculosis.

A client reports difficulty breathing and a sharp pain in the right side of the chest. The respiratory rate measures 40 breaths/minute. The nurse should assign highest priority to which goal of care? a) Reducing anxiety b) Maintaining an adequate circulatory volume c) Relieving pain d) Maintaining effective respirations

d) Maintaining effective respirations As suggested by the ABCs of cardiopulmonary resuscitation — airway, breathing, and circulation — the most important goal is to maintain a patent airway and effective respirations, regardless of the client's diagnosis or clinical presentation. Although the other options are pertinent for this client, they're secondary to maintaining effective respirations.

A 69-year-old client comes to the emergency department with a history of productive cough, night sweats, and a 30-lb weight loss over the past 8 months. A diagnosis of tuberculosis is suspected. Which intervention is necessary for this client? a) Place the client in a private room, and wear a mask when entering the room. b) Place the client in a private room with special ventilation, and provide gowns and gloves for those who enter. c) Place the client in a semiprivate room with personal protective equipment. d) Place the client in a private room with negative air pressure, and implement airborne precautions.

d) Place the client in a private room with negative air pressure, and implement airborne precautions. A client with tuberculosis should be placed in a private room that has regularly monitored negative air pressure, and airborne precautions should be implemented. The doors of the room must remain closed, and everyone who comes in close contact with the client must wear a specialized mask.

Which organism is the most common causative agent for bacterial pneumonia? a) Respiratory syncytial virus (RSV) b) Parainfluenza virus c) Mycoplasma d) Pneumococci

d) Pneumococci Pneumococcal pneumonia is the most common causative agent accounting for about 90% of bacterial pneumonia. Mycoplasma is a causative agent for primary atypical pneumonia. Parainfluenza virus and RSV account for viral pneumonia

Presence of which factor would place a child at increased risk for an asthma-related death? a) One admission for asthma last year b) Prior admission to the general pediatric floor c) Use of an inhaler at home d) Prior admission to an intensive care unit for asthma

d) Prior admission to an intensive care unit for asthma Asthma results in varying degrees of respiratory distress. A prior admission to an intensive care unit marks an increased severity and need of immediate therapy. Two or more hospitalizations for asthma, a recent hospitalization or emergency department visit in the past month, or three or more emergency department visits in the past year puts a child at high risk for asthma-related death. Although current use of systemic steroids would also be a risk factor, not all inhalers contain steroids

Which test definitively diagnoses a pulmonary embolism? a) Computed tomography scan b) Arterial blood gas (ABG) analysis c) Ventilation-perfusion scan d) Pulmonary angiogram

d) Pulmonary angiogram Pulmonary angiogram is used to definitively diagnose a pulmonary embolism. A catheter is passed through the circulation to the region of the occlusion; the region can be outlined with an injection of contrast medium and viewed by fluoroscopy. This shows the location of the clot, as well as the extent of the perfusion defect. Computed tomography scan can show the location of infarcted or ischemic tissue. ABG levels can define the amount of hypoxia present. The ventilation-perfusion scan can report whether there's a ventilation-perfusion mismatch present and define the amount of tissue involved.

Which test definitively diagnoses a pulmonary embolism? a) Computed tomography scan b) Ventilation-perfusion scan c) Arterial blood gas (ABG) analysis d) Pulmonary angiogram

d) Pulmonary angiogram Pulmonary angiogram is used to definitively diagnose a pulmonary embolism. A catheter is passed through the circulation to the region of the occlusion; the region can be outlined with an injection of contrast medium and viewed by fluoroscopy. This shows the location of the clot, as well as the extent of the perfusion defect. Computed tomography scan can show the location of infarcted or ischemic tissue. ABG levels can define the amount of hypoxia present. The ventilation-perfusion scan can report whether there's a ventilation-perfusion mismatch present and define the amount of tissue involved.

During the recovery stages of croup, the nurse should explain which intervention to parents? a) Allow the child to go to the playground. b) Provide three nutritious meals per day. c) Limit oral fluid intake. d) Recognize signs of respiratory distress.

d) Recognize signs of respiratory distress. Although most children with croup recover without complications, the parents should be able to recognize signs and symptoms of respiratory distress and know how to access emergency services. Oral fluids should be encouraged because fluids help to thin secretions. Although nutrition is important, frequent small nutritious snacks are usually more appealing than an entire meal. Children should have optimal rest and engage in quiet play. A comfortable environment free of noxious stimuli lessens respiratory distress.

After undergoing a thoracotomy, a client is receiving epidural analgesia. Which data collection finding indicates that the client has developed the most serious complication of epidural analgesia? a) Heightened alertness b) Numbness and tingling of the extremities c) Increased heart rate d) Respiratory depression

d) Respiratory depression Respiratory depression is the most serious complication of epidural analgesia. Other potential complications include hypotension, decreased sensation and movement of the extremities, allergic reactions, and urine retention. Typically, epidural analgesia causes central nervous system depression (indicated by drowsiness) as well as a decreased heart rate and blood pressure.

The nurse administers albuterol, as prescribed, to a client with emphysema. Which finding indicates that the drug is producing a therapeutic effect? a) Urine output of 40 ml/hour b) Heart rate of 100 beats/minute c) Dilated and reactive pupils d) Respiratory rate of 22 breaths/minute

d) Respiratory rate of 22 breaths/minute In a client with emphysema, albuterol is used as a bronchodilator. A respiratory rate of 22 breaths/minute indicates that the drug has achieved its therapeutic effect because fewer respirations are required to achieve oxygenation. Albuterol has no effect on pupil reaction or urine output. It may cause a change in the heart rate, but this is an adverse, not therapeutic, effect.

Which test is the definitive means of diagnosing tuberculosis (TB) in children? a) Chest X-ray b) Urine culture c) Tuberculin test d) Sputum specimen

d) Sputum specimen A sputum culture is the definitive test for TB in children. The tuberculin test is the most accurate but not necessarily the most reliable test for TB in children. X-rays usually appear normal in children with TB. Stool culture, not urine culture, and gastric washings will show positive results on acid-fast smears but aren't specific for Mycobacterium tuberculosis.

A client with a pulmonary embolism may have an umbrella-like filter placed in the vena cava for which reason? a) The filter contains anticoagulants that are slowly released, dissolving any clots b) The filter prevents further clot formation c) The filter collects clots so they don't go to the lung d) The filter break up clots into insignificantly small pieces

d) The filter break up clots into insignificantly small pieces The umbrella-like filter is placed in a client at high risk for the formation of more clots that could potentially become pulmonary emboli. The filter breaks the clots into small pieces that won't significantly occlude the pulmonary vasculature. The filter doesn't release anticoagulants and doesn't prevent further clot formation. The filter doesn't collect the clots, because if it did, it would have to be emptied periodically, causing the client to require surgery in the future.

A nursing student is reviewing the respiratory system for an upcoming examination. Which term will the student review that describes the amount of air inspired and expired with each breath? a) Residual volume b) Vital capacity c) Dead-space volume d) Tidal volume

d) Tidal volume Tidal volume is the amount of air inspired and expired with each breath. Residual volume is the amount of air remaining in the lungs after forcibly exhaling. Vital capacity is the maximum amount of air that can be moved out of the lungs after maximal inspiration and expiration. Dead-space volume is the amount of air remaining in the upper airways that never reaches the alveoli. In pathologic conditions, dead space may also exist in the lower airways.

A 21-year-old client with cystic fibrosis develops pneumonia. To decrease the viscosity of respiratory secretions, the physician prescribes acetylcysteine. Before administering the first dose, the nurse checks the client's history for asthma. Acetylcysteine must be used cautiously in a client with asthma because: a) it's a respiratory depressant. b) it's a respiratory stimulant. c) it inhibits the cough reflex. d) it may induce bronchospasm.

d) it may induce bronchospasm. Acetylcysteine must be used cautiously in a client with asthma because it may induce bronchospasm. The drug isn't a respiratory depressant or stimulant. It's a mucolytic agent that decreases the viscosity of respiratory secretions by altering the molecular composition of mucus. Acetylcysteine doesn't inhibit the cough reflex.


Set pelajaran terkait

Rektion 4, s. sorgen - s. wundern / abhängen - beginnen

View Set

Chapter 18 Privacy and Consumer Protection

View Set

PERFUSION THE CONCEPT OF PERFUSION

View Set

Chapter 06: Current Digital Forensics Tools

View Set

A&P CH 13 - Anatomy of the nervous system

View Set

Chapter 52: Renal and Urinary Medications

View Set